SlideShare a Scribd company logo
1 of 100
MedicalResearch.com
Exclusive Interviews with Medical Research and
Health Care Researchers from Major and Specialty Medical
Research Journals and Meetings
Editor: Marie Benz, MD
info@medicalresearch.com
January 22 2015
For Informational Purposes Only: Not for Specific Medical Advice.
Medical Disclaimer | Terms and Conditions
• The contents of the MedicalResearch.com Site, such as text, graphics, images, and
other material contained on the MedicalResearch.com Site ("Content") are for
informational purposes only. The Content is not intended to be a substitute for
professional medical advice, diagnosis, or treatment. Always seek the advice of
your physician or other qualified health provider with any questions you may have
regarding a medical condition. Never disregard professional medical advice or
delay in seeking it because of something you have read on the Hemodialysis.com
Site!
• If you think you may have a medical emergency, call your doctor or 911
immediately. MedicalResearch.com does not recommend or endorse any specific
tests, physicians, products, procedures, opinions, or other information that may be
mentioned on the Site. Reliance on any information provided by
MedicalResearch.com or other Eminent Domains Inc (EDI) websites, EDI
employees, others appearing on the Site at the invitation of MedicalResearch.com
or EDI, or other visitors to the Site is solely at your own risk.
• The Site may contain health- or medical-related materials that are sexually explicit.
If you find these materials offensive, you may not want to use our Site. The Site
and the Content are provided on an "as is" basis.
Read more interviews on
MedicalResearch.com
Text Messaging Can Boost Medical Research Clinical Trial Recruitment
MedicalResearch.com Interview with:
Moe Alsumidaie MBA MSF
President & Chief Scientific Officer
Annex Clinical
• MedicalResearch: What is the background for this study?
• Response: SUMMARY:
• A real-world case study measuring the impact of Short Messaging System (SMS) or “Text
Messaging” on clinical trial patient recruitment using an interactive two-way patient engagement
platform by Mosio, Inc., which provides clinical research services designed to increase patient
recruitment, engagement and retention, found that use of text messaging alone can be an effective
means of patient engagement that results in clinical trial patient enrollment.
• Patient recruitment, retention and medication adherence continue to be challenges in conducting
effective clinical trials. While clinical trials often rely on email recruitment, recent studies suggest
that only 22% of emails are read.1 Alternatively, 98% of text messages are read1 and 90% of text
messages are read within the first three minutes of receipt.2
• Recent research has evaluated the impact of Short Messaging System (SMS) or “Text Messaging” in
healthcare settings, such as appointment reminders and medication adherence. Results have
demonstrated that SMS intervention significantly improved patient behavioral outcomes: patients
who received SMS reminders were more likely to show up to appointments on time,3 and patients
who received SMS reminders were more adherent to medications.4 However, only limited research
is available on the effect of SMS on clinical trial subject enrollment.
• Johnson County Clin-Trials (JCCT), a clinical research facility that specializes in executing 10-15
vaccine clinical trials per year, was facing issues with enrolling patients rapidly in a tight time frame
using email. To access a more effective strategy to better engage patients, JCCT employed two-way
SMS/text messaging solutions, and this study assessed the impact of SMS/text messaging on
patient recruitment and enrollment.
Read the rest of the interviews on MedicalResearch.com
Content NOT an endorsement of efficacy and NOT intended as specific medical advice.
Text Messaging Can Boost Medical Research Clinical Trial Recruitment
MedicalResearch.com Interview with:
Moe Alsumidaie MBA MSF
President & Chief Scientific Officer
Annex Clinical
• MedicalResearch: What are the main findings?
• Response: Overall, SMS/text messaging alone produced an immediate response that resulted
in more optimized clinical trial enrollment outcomes in comparison to JCCT’s previous
experiences using email. JCCT remarked that the response was five times the response
compared to e-mail and that enrollment results exceeded the trial sponsor’s expectations.
• Use of two-way SMS/text messaging achieved a 1% increase in research subjects enrolled for
every 1.5% increase in text messages sent. Over an 8-week enrollment period, a total of
1,541 text messages sent resulted in screening of 795 patients and enrollment of 265
patients.
Read the rest of the interviews on MedicalResearch.com
Content NOT an endorsement of efficacy and NOT intended as specific medical advice.
Text Messaging Can Boost Medical Research Clinical Trial Recruitment
MedicalResearch.com Interview with:
Moe Alsumidaie MBA MSF
President & Chief Scientific Officer
Annex Clinical
• MedicalResearch: What should clinicians and patients take away from your report?
• Response: This case study suggests that two-way text messaging through a strategic and interactive
patient engagement platform is an effective tool to captivate potential study participants by
reaching them on the mobile devices they carry. JCCT found that patients responded positively to
receiving SMS/Text Messages.
• In this case, we have found that two-way SMS/Text Messaging solutions enable researchers to
enroll patients in clinical trials. It is important to emphasize that IT systems utilization in clinical
trials require validated systems that are HIPAA and 21 CFR Part 11 compliant; the two-way SMS
system we used was validated for clinical trial use.
• It is also important to note that FCC regulations require that any SMS/text message recipient opt in
to receiving SMS from the deliverer. Many study sites have patient email lists; I recommend that
study sites consider building their own SMS/text message database, so that they can send mobile
communications to patients and comply with FCC regulations. Taking the effort now can save study
sites time in the long run, while boosting enrollment performance, and available technologies can
facilitate the process.
• For example, the Mosio platform used by JCCT in the case study includes technology and tools to
assist with developing communications content that can convert patients to opt in to receiving
SMS/text messages, such as opt-in web widgets, opt-in wording templates in consent forms, and
opt-in email blast templates to send out to existing patient databases.
• Further, I want to indicate that many patients tend to respond to SMS/text messages by texting
back with questions and more information. Mosio’s technological platform is capable of having
interactive and automated conversations with patients, which can be used to qualify patients while
enhancing patient engagement and reducing the work burden on study sites.
Read the rest of the interviews on MedicalResearch.com
Content NOT an endorsement of efficacy and NOT intended as specific medical advice.
Text Messaging Can Boost Medical Research Clinical Trial Recruitment
MedicalResearch.com Interview with:
Moe Alsumidaie MBA MSF
President & Chief Scientific Officer
Annex Clinical
• MedicalResearch: What recommendations do you have for future research as a result of
this study?
• Response: This real-world case study, conducted during an actual vaccine clinical trial,
strongly suggests that a bilateral two-way SMS/text messaging platform can boost trial
recruitment, with implications for cost savings, study timing and quality of results. Future
prospective studies should compare SMS/text messaging with other common recruiting
approaches via random subject assignment to groups to quantify levels of response with
each approach and its resulting effectiveness and cost efficiency.
• New approaches will be key in meeting trial retention, compliance and adherence challenges.
In rigorous investigational product trials, non-adherence rates of 20%-30% require a 50%
increase in sample size to maintain equivalent statistical power, while a 50% non-adherence
rate requires a 200% increase in sample size.5
• Conducting additional studies to evaluate the impact of SMS/Text Messages on medication
adherence and other areas such as patient retention, study compliance, appointment
reminders, and patient reported outcomes data collection will bring us closer to finding
useful and cost effective solutions in clinical trials.
Read the rest of the interviews on MedicalResearch.com
Content NOT an endorsement of efficacy and NOT intended as specific medical advice.
Text Messaging Can Boost Medical Research Clinical Trial Recruitment
MedicalResearch.com Interview with:
Moe Alsumidaie MBA MSF
President & Chief Scientific Officer
Annex Clinical
• References:
• Frost & Sullivan 2010, Epsilon 2009
• http://www.tatango.com/blog/90-of-text-messages-are-read-within-3-minutes/
• Use of mobile telephone short message service as a reminder: the effect on patient
attendance, Sumanth Prasad and Richa Anand, International Dental Journal 2012; 62: 21-26
• Sahar Khonsari, et al., Effect of a reminder system using an automated short messaging
service on medication adherence following acute coronary syndrome. Eur J Cardiovasc Nurs.
published 02, February 2014
• Smith, Dorothy PharmD, Patient Nonadherencein Clinical Trials: Could There Be a Link to Post
Marketing Patient Safety?, Consumer Health Information Corporation, October2011
• Citation:
• Text Messaging Enhances Clinical Trial Enrollment with Mobile Solutions from Mosio
Applied Clinical Trials
• Publish date: Nov 3, 2014
Moe Alsumidaie
Read the rest of the interviews on MedicalResearch.com
Content NOT an endorsement of efficacy and NOT intended as specific medical advice.
Endocrine Society Guidelines For Weight Loss Medications in Obesity
MedicalResearch.com interview with:
Caroline M. Apovian, MD
Chair of the Endocrine Society task force that developed “Pharmacological Management of Obesity: An Endocrine Society Clinical Practice
Guideline” Boston University School of Medicine Boston Medical Center
• MedicalResearch: What is the background for this report?
• Dr. Apovian: The Food and Drug Administration has approved four new anti-obesity drugs –
lorcaserin, phentermine/topiramate, naltrexone/bupropion and liraglutide – in the past two
years. To help clinicians navigate this changing landscape, the Endocrine Society developed its
Clinical Practice Guideline to provide strategies for prescribing drugs to manage obesity and
promote weight loss.
• MedicalResearch: What are the main findings?
• Dr. Apovian: In the Clinical Practice Guideline, the Endocrine Society recommends that diet,
exercise and behavioral modifications be part of all obesity management approaches. Other
tools such as weight loss medications and bariatric surgery can be combined with behavioral
changes to reduce food intake and increase physical activity, in appropriate patients. Patients
who have been unable to successfully lose weight and maintain a goal weight may be
candidates for prescription medication if they meet the criteria on the drug’s label as well as
BMI criteria (BMI greater than or equal to 30 or greater than or equal to 27 with at least one
comorbidity).
Read the rest of the interviews on MedicalResearch.com
Content NOT an endorsement of efficacy and NOT intended as specific medical advice.
Endocrine Society Guidelines For Weight Loss Medications in Obesity
MedicalResearch.com interview with:
Caroline M. Apovian, MD
Chair of the Endocrine Society task force that developed “Pharmacological Management of Obesity: An Endocrine Society Clinical Practice
Guideline” Boston University School of Medicine Boston Medical Center
• Other recommendations from the CPG include:
• If a patient responds well to a weight loss medication and loses 5 percent or more of their
body weight after three months, the medication can be continued. If the medication is
ineffective or the patient experiences side effects, the prescription should be stopped and an
alternative medication or approach considered.
• Since some diabetes medications are associated with weight gain, people with diabetes who
are obese or overweight should be given medications that promote weight loss or have no
effect on weight as first- and second-line treatments. Doctors should discuss medications’
potential effects on weight with patients.
• Certain types of medication – angiotensin converting enzyme inhibitors, angiotensin receptor
blockers and calcium channel blockers – should be used as a first-line treatment for high
blood pressure in obese people. These are effective blood pressure treatments that are less
likely to contribute to weight gain than an alternative medication, beta-adrenergic blockers.
• When patients need medications that can have an impact on weight such as antidepressants,
antipsychotic drugs and medications for treating epilepsy, they should be fully informed and
provided with estimates of each option’s anticipated effect on weight. Doctors and patients
should engage in a shared-decision making process to evaluate the options.
• In patients with uncontrolled high blood pressure or a history of heart disease, the
medications phentermine and diethylpropion should not be used.
Read the rest of the interviews on MedicalResearch.com
Content NOT an endorsement of efficacy and NOT intended as specific medical advice.
Endocrine Society Guidelines For Weight Loss Medications in Obesity
MedicalResearch.com interview with:
Caroline M. Apovian, MD
Chair of the Endocrine Society task force that developed “Pharmacological Management of Obesity: An Endocrine Society Clinical Practice
Guideline” Boston University School of Medicine Boston Medical Center
• MedicalResearch: What should clinicians and patients take away from your report?
• Dr. Apovian: Lifestyle changes should always be a central part of any weight loss strategy.
Medications do not work by themselves, but they can help people maintain a healthy diet by
reducing the appetite. Adding a medication to a lifestyle modification program is likely to
result in greater weight loss.
• Citation:
• Pharmacological Management of Obesity: An Endocrine Society Clinical Practice Guideline
• Caroline M. Apovian, Louis J. Aronne, Daniel H. Bessesen, Marie E. McDonnell, M. Hassan
Murad, Uberto Pagotto, Donna H. Ryan, and Christopher D. Still
• DOI: http://dx.doi.org/10.1210/jc.2014-3415
Received: September 03, 2014 Accepted: December 08, 2014
Published Online: January 15, 2015
Read the rest of the interviews on MedicalResearch.com
Content NOT an endorsement of efficacy and NOT intended as specific medical advice.
ICD-9 Codes Do Not Identify All Strokes in Atrial Fibrillation Patients
MedicalResearch.com Interview with:
Jonathan Thigpen, PharmD
Assistant Professor Clinical and Administrative Sciences
Notre Dame of Maryland University School of Pharmacy
• Medical Research: What is the background for this study? What are the main findings?
Dr. Thigpen: This effort assessed the accuracy of International Classification of Disease 9th Edition
(ICD-9) stroke codes in identifying valid stroke events in a cohort of atrial fibrillation (AF) patients.
The initial electronic search yielded 1,812 events across three stroke centers (Boston Medical
Center, Geisinger Health System, and University of Alabama). All ICD-9 identified stroke events were
vetted through manual chart review with final adjudication by a stroke neurologist. Atrial fibrillation
was verified by evidence via electrocardiogram at stroke admission, 6 months prior to, or 90 days
after stroke admission.
• In addition to assessing the accuracy of the stroke codes alone, we also assessed the accuracy of
stroke and Atrial fibrillation codes combined as well as the accuracy of stroke codes when seeking
for stroke associated with Atrial fibrillation. These additional steps give readers insight as to the
accuracy and reliability of using ICD-9 codes alone to create a stroke plus AF cohort. We feel that
this effort is extremely important given the increasing reliance on ICD-9 codes as a means of
identifying stroke events and covariates in research, especially research using administrative data.
The positive predictive value (PPV) of stroke codes alone was 94.2%. PPVs did not differ across clinical
site or by type of event (ischemic vs. intracranial hemorrhage). PPV of stroke codes did differ by event
coding position (primary vs. other; 97.2% vs. 83.7%) and by ischemic stroke code (433 vs. 434; 85.2% vs.
94.4%). When combined with validation of Atrial fibrillation codes, the PPV of stroke codes decreased to
82.2%. After excluding ischemic stroke due to a different mechanism (eg, vascular procedure, tumor,
sepsis) the PPV dropped further to 72.8%. As a separate exercise, manual review confirmed 33 (7.2%)
ischemic strokes in 458 events coded as “without infarction”.
Read the rest of the interviews on MedicalResearch.com
Content NOT an endorsement of efficacy and NOT intended as specific medical advice.
ICD-9 Codes Do Not Identify All Strokes in Atrial Fibrillation Patients
MedicalResearch.com Interview with:
Jonathan Thigpen, PharmD
Assistant Professor Clinical and Administrative Sciences
Notre Dame of Maryland University School of Pharmacy
• Medical Research: What should clinicians and patients take away from your report?
• Dr. Thigpen: These results indicate that ICD-9 stroke codes alone have limited use in
identifying acute strokes in the setting of active Atrial fibrillation. Manual verification of
stroke is needed to confirm stroke events in the setting of AF so as to reduce potential bias.
• Medical Research: What recommendations do you have for future research as a result of
this study?
• Dr. Thigpen: Future research could build on our study in the following ways.
• Firstly, there is limited knowledge concerning the accuracy of ICD-10 stroke codes.
Secondly, our study (and other previous literature) indicates several screening methods that
tend to lead to higher coding accuracies (ie, only using stroke codes in the primary position).
Further optimizing ICD-9 screening methods so as to limit the inaccuracies of stroke codes
will potentially help better establish ICD stroke codes as a reliable tool for case and covariate
ascertainment.
• Citation:
• Validity of International Classification of Disease Codes to Identify Ischemic Stroke and
Intracranial Hemorrhage Among Individuals With Associated Diagnosis of Atrial Fibrillation
• AHA publication:Jonathan L. Thigpen, Chrisly Dillon, Kristen B. Forster, Lori Henault, Emily K.
Quinn, Yorghos Tripodis, Peter B. Berger, Elaine M. Hylek, and Nita A. Limdi
• Circ Cardiovasc Qual Outcomes. 2015;CIRCOUTCOMES.113.000371published online before
print January 13 2015, doi:10.1161/CIRCOUTCOMES.113.000371
Read the rest of the interviews on MedicalResearch.com
Content NOT an endorsement of efficacy and NOT intended as specific medical advice.
Walking Program Insufficient To Reduce Risk of Falls in Elderly
MedicalResearch.com Interview with:
Alexander Voukelatos BSc, BA, MA(psych), PhD Healthy Populations Program Manager Health Promotion
Sydney Local Health District and Conjoint Lecturer
School of Public Health and Community Medicine University of NSW
• Medical Research: What is the background for this study?
• Response: Falls in older people has been a significant public health issue in high income
countries for several decades now. We know that if current trends continue, given that more
people will be living for longer, falls will be an even bigger issue in the not too distant. Falls
are not an inevitable part of ageing, and in fact many falls can be prevented relatively simply
by increasing physical activity.
• For over 15 years we’ve known that physical activity is one of the most effective ways of
reducing the risk of falls in older people living in the community; since the publication of the
first Cochrane review on Interventions for preventing falls in the elderly by Gillespie and
colleagues. [1] I know that Health Departments here, in Australia, and in New Zealand – as I
suspect has been the case in many high income countries – have invested a lot of resources
over the past few decades into reducing falls-related hospital admissions in older people.
Much of this going into promoting and funding physical activity programs for older people.
• However, this investment has had very little if any impact on falls-related hospital admissions
in older people. There may be several reasons we haven’t seen any difference in these rates.
In New South Wales – Australia’s most populous state – we know that physical activity rates
amongst older people have actually risen by about 15% between 1998 and 2005,[2] the most
popular activity by far being walking,[3] yet we haven’t see any corresponding change in falls-
related hospitalization rates. Perhaps there has not been enough time for these programs to
have made an impact on hospitalization rates, or perhaps the change in physical activity
levels is insufficient to make an impact on these rates.
Read the rest of the interviews on MedicalResearch.com
Content NOT an endorsement of efficacy and NOT intended as specific medical advice.
Walking Program Insufficient To Reduce Risk of Falls in Elderly
MedicalResearch.com Interview with:
Alexander Voukelatos BSc, BA, MA(psych), PhD Healthy Populations Program Manager Health Promotion
Sydney Local Health District and Conjoint Lecturer
School of Public Health and Community Medicine University of NSW
• Another possibility could be that while we’ve seen an increase in physical activity in older people
perhaps its not the kind of physical activity that results in a reduction in falls. Sherrington and
colleagues [4] reviewed effective physical activity interventions for preventing falls in older people
and found they had several elements in common:
a) the physical activity included balance challenging exercise i.e. exercises taking participants to the
limits of their stability,
b) at least 50 hours of accumulated activity was needed, and
c) no walking was included in the exercises. So we know not all types of physical activity will be
equally effective in reducing the risk of falling. There is some disagreement in the literature about
walking.
• There are several studies that included walking as part of the intervention and showed a reduction
in falls in older people. Other studies supported the conclusions made by Sherrington that walking
is not associated with a reduction in falls. All of these studies included walking as a component of
an intervention which makes it difficult to figure out what effect walking specifically has on falls
rates.
• This is were our study comes in. We wanted to investigate the effectiveness of a walking program
on falls in older people, specifically sedentary older people, who we presumed would get the most
benefit from becoming more physically active.
• We developed a walking program specifically for sedentary older people, that they could do
themselves in their own time, at their preferred locale. The aim of the program was to get
participants walking for at least 150 minutes per week at a brisk pace. The program comprised of
four parts: the first part focused on increasing the frequency of walks, the second part focused on
getting participants to walk for at least 150 minutes per week, followed by walking for 150 minutes
at a brisk pace, while the final part focused on supporting participants in maintaining their walking
levels and incorporating walking as part of their daily activities.
Read the rest of the interviews on MedicalResearch.com
Content NOT an endorsement of efficacy and NOT intended as specific medical advice.
Walking Program Insufficient To Reduce Risk of Falls in Elderly
MedicalResearch.com Interview with:
Alexander Voukelatos BSc, BA, MA(psych), PhD Healthy Populations Program Manager Health Promotion
Sydney Local Health District and Conjoint Lecturer
School of Public Health and Community Medicine University of NSW
• Medical Research: What are the main findings?
• Response: Overall, we found no evidence that walking is associated with a reduction (or
increase) in falls rates. The walking program did however increase participants’ levels of
general physical activity as well as walking levels.
However, on closer inspection there were some interesting trends. Note that by trends I
mean there was no statistical support for the following conclusions but there was some
consistency in the data that suggests there maybe something going on (something for future
research). The data indicated that for participants aged 65-74 years walking had non-
significant decrease in falls, whereas for participants aged 75 years or more walking had a
non-significant increase in the the risk of falls. The key words here are non-significant. This
study was not powered to examine the data stratified by age group. However these trends
may be explained if we consider age-group a proxy for frailty (of course this is not conclusive
as there will be some 65-74 year olds who are frail, and indeed some people over 75 years
who are not frail) For the more frail participants (i.e. 75+ year olds) walking actually increased
their risk of falls by exposing them to environmental falls risk factors – eg. uneven surfaces,
slippery paths – which they were too frail to negotiate. On the other hand the less frail
participants (65-74 year olds) even though they were exposed to more environmental falls
hazards they were able to negotiate the hazards and in time reduce their risk of falling. These
two opposing effects may have cancelled each other out thus resulting in the null effect for
the overall study. This is purely speculative however.
Read the rest of the interviews on MedicalResearch.com
Content NOT an endorsement of efficacy and NOT intended as specific medical advice.
Walking Program Insufficient To Reduce Risk of Falls in Elderly
MedicalResearch.com Interview with:
Alexander Voukelatos BSc, BA, MA(psych), PhD Healthy Populations Program Manager Health Promotion
Sydney Local Health District and Conjoint Lecturer
School of Public Health and Community Medicine University of NSW
• Medical Research: What should clinicians and patients take away from your report?
• Response: The take away message from this study is that walking is insufficient activity to
reduce the risk of falls. If a clinician or anyone else is concerned about someone’s risk of
falling walking is probably not going to do much to address this.
• However, walking is still a highly effective and very important activity for addressing many
chronic diseases in older people. And walking can still play an important role in reducing falls
as a way of increasing activity levels for sedentary older people who, once they become more
active, can then participate in proven falls prevention activities such as tai chi.
• I would also recommend that clinicians use their judgment in prescribing walking for people
over 75 years of age.
Read the rest of the interviews on MedicalResearch.com
Content NOT an endorsement of efficacy and NOT intended as specific medical advice.
Walking Program Insufficient To Reduce Risk of Falls in Elderly
MedicalResearch.com Interview with:
Alexander Voukelatos BSc, BA, MA(psych), PhD Healthy Populations Program Manager Health Promotion
Sydney Local Health District and Conjoint Lecturer
School of Public Health and Community Medicine University of NSW
• Medical Research: What recommendations do you have for future research as a result of
this study?
• Response: This study shows that the relationship between walking and falls is not straight
forward. Future studies might consider planning a stratified analysis based on age-groups, or
level of frailty to see if age is a modifying variable. Also future studies could use different
types of walking such as obstacle walking or hiking/bushwalking which present many falls
hazards to walker – the idea being that if the walker can learn to safely negotiate these high-
level falls hazards then that will confer a beneficial effect in negotiating more moderate falls
hazards typically found in their local environment.
• [1] Gillespie LD, Gillespie WJ, Cumming R, Lamb S, Rowe BH. Interventions for preventing falls
in the elderly (Cochrane Review). The Cochrane Library 1997, Issue 4.
[2] Chau J, Smith B, Chey T, Merom D, Bauman A. Trends in population levels of sufficient
physical activity in NSW, 1998 to 2005: Summary report. Sydney, NSW: NSW Centre for
Physical Activity and Health (CPAH); January 2007. Report No. CPAH06-001b
[3] Merom D, Pye V, Macniven R, Van der Ploeg H, Milat A, Sherrington C, Lord S, Bauman A.
Prevalence and correlates of participation in fall prevention exercise/physical activity by older
adults. Prev Med 2012; 55(5): 613-7.
[4] Sherrington C, Tiedemann A, Fairhall N, Close JCT, Lord SR. Exercise to prevent falls in
older adults: an updated meta-analysis and best practice recommendations. NSW Public
Health Bull 2011, 22 (3-4): 78-83.
Read the rest of the interviews on MedicalResearch.com
Content NOT an endorsement of efficacy and NOT intended as specific medical advice.
Walking Program Insufficient To Reduce Risk of Falls in Elderly
MedicalResearch.com Interview with:
Alexander Voukelatos BSc, BA, MA(psych), PhD Healthy Populations Program Manager Health Promotion
Sydney Local Health District and Conjoint Lecturer
School of Public Health and Community Medicine University of NSW
• Citation:
• The impact of a home-based walking programme on falls in older people: the Easy Steps
randomised controlled trial
Alexander Voukelatos, Dafna Merom, Catherine Sherrington, Chris Rissel, Robert G.
Cumming, and Stephen R. Lord
Age Ageing first published online January 8, 2015 doi:10.1093/ageing/afu186
Read the rest of the interviews on MedicalResearch.com
Content NOT an endorsement of efficacy and NOT intended as specific medical advice.
Eczema and Atopic Dermatitis Linked To Increased Cardiovascular Risk
MedicalResearch.com Interview with:
Dr. Jonathan L. Silverberg MD PhD MPH
Assistant Professor in Dermatology, Medical Social Sciences and Preventive Medicine
Northwestern University, Chicago, Illinois
• Medical Research: What is the background for this study? What are the main findings?
Dr. Silverberg: There is a growing body of literature supporting an association between
psoriasis and increased cardiovascular risk. We hypothesized that these associations are not
specific to psoriasis. Rather, they likely occur in other chronic inflammatory skin disorders,
namely eczema. We studied two large-scale US population-based studies and found that
adults with eczema were more likely to smoke cigarettes, drink alcohol and were less
physically active. In turn, they also have higher rates of obesity, high blood pressure,
prediabetes and type 2 diabetes and high cholesterol. Of note, eczema was associated with
these disorders even after controlling for smoking, alcohol consumption and physical activity.
This suggests that chronic inflammation and/or other factors related to eczema may also
drive increased cardiovascular risk.
• Medical Research: What should clinicians and patients take away from your report?
• Dr. Silverberg: There are several take home points:
• First, smoking, alcohol and physical activity are all modifiable cardiovascular risk factors.
Adults with eczema should be screened for these behaviors and clinical interventions should
be employed early and often to reduce their excess cardiovascular risk.
Second, better systemic treatments are needed to address the systemic inflammation of
moderate to severe eczema. Clinicians should avoid using systemic steroids, which
predictably increase weight gain, blood pressure and blood sugar.
Read the rest of the interviews on MedicalResearch.com
Content NOT an endorsement of efficacy and NOT intended as specific medical advice.
Eczema and Atopic Dermatitis Linked To Increased Cardiovascular Risk
MedicalResearch.com Interview with:
Dr. Jonathan L. Silverberg MD PhD MPH
Assistant Professor in Dermatology, Medical Social Sciences and Preventive Medicine
Northwestern University, Chicago, Illinois
• Medical Research: What recommendations do you have for future research as a result of
this study?
• Dr. Silverberg: Future studies are needed to identify the mechanisms of increased
cardiovascular risk in atopic dermatitis and the most effective clinical approaches to reducing
this risk. Further, more research is needed for the development of safer and more effective
systemic treatments of atopic dermatitis.
• Citation:
• Eczema and cardiovascular risk factors in 2 US adult population studies
• Silverberg JI, Greenland P
J Allergy Clin Immunol. 2015 Jan 8. pii: S0091-6749(14)01677-7. doi:
10.1016/j.jaci.2014.11.023. [Epub ahead of print]
Read the rest of the interviews on MedicalResearch.com
Content NOT an endorsement of efficacy and NOT intended as specific medical advice.
Expedited Partner Therapy May Decrease Population Levels of Gonorrhea and Chlamydia
MedicalResearch.com Interview with:
Matthew Golden MD, MPH
Director, PHSKC HIV/STD Program
Professor of Medicine, University of Washington Harborview Medical Center
• Medical Research: What is the background for this study? What are the main findings?
Dr. Golden: Gonorrhea and chlamydial infection are the most common reportable infections
in the United States and, in women, are associated with pelvic inflammatory disease, ectopic
pregnancy, infertility and chronic pelvic pain. One way to decrease the number of cases of
gonorrhea and chlamydia is to increase our success in treating the sex partners of persons
diagnosed with these infections. Expedited partner therapy (EPT) – treating partners without
requiring them to first undergo a medical evaluation – is one way to increase partner
treatment. This usually involves giving people medication to give to their partners. Prior
randomized trials have found that EPT decreases patients’ risk of becoming reinfected.
• We conducted a community-level randomized trial to evaluate whether making free
Expedited partner therapy available to medical providers would increase the use of Expedited
Partner Therapy and decrease gonorrhea and chlamydial infections at the population level.
We found that a public health program that made Expedited partner therapy widely available
could dramatically increase medical providers use of EPT. Although our final result was not
statistically significant, our findings suggest that the program likely decreased both gonorrhea
and chlamydial infection by about 10% at the population-level.
Read the rest of the interviews on MedicalResearch.com
Content NOT an endorsement of efficacy and NOT intended as specific medical advice.
Expedited Partner Therapy May Decrease Population Levels of Gonorrhea and Chlamydia
MedicalResearch.com Interview with:
Matthew Golden MD, MPH
Director, PHSKC HIV/STD Program
Professor of Medicine, University of Washington Harborview Medical Center
• Medical Research: What should clinicians and patients take away from your report?
• Dr. Golden: Expedited partner therapy, which was previously shown to be an effective
intervention in increasing partner treatment and decreasing patients’ risk of reinfection, can
also have a population-level effect.
• Medical Research: What recommendations do you have for future research as a result of
this study?
• Dr. Golden: Health departments need to develop and test ways to better promote Expedited
partner therapy use.
• Citation:
• Uptake and Population-Level Impact of Expedited Partner Therapy (EPT) on Chlamydia
trachomatis And Neisseria gonorrhoeae : The Washington State Community-Level
Randomized Trial of EPT.
• Matthew R. Golden et al
Published: January 15, 2015 DOI: 10.1371/journal.pmed.1001777
Read the rest of the interviews on MedicalResearch.com
Content NOT an endorsement of efficacy and NOT intended as specific medical advice.
Mortality Risks High In Elderly Patients After Carotid Artery Stenting
MedicalResearch.com Interview with:
Soko Setoguchi, MD DrPH Assistant Professor of Medicine
Harvard Medical School and Harvard School of Public Health
Director of Safety and Outcome Research in Cardiology
Associate Physician in the Division of Pharmacoepidemiology and Pharmacoeconomics Brigham and Women’s Hospital
• Medical Research: What is the background for this study? What are the main findings?
Dr. Setoguchi: Medicare made a decision to cover Carotid Artery Stenting (CAS) in 2005 after
publication of SAPPHIRE, which demonstrated the efficacy of Carotid Artery Stenting (CAS) vs
Carotid endarterectomy (CEA) in high risk patients for CEA. Despite the data showing
increased carotid artery stenting dissemination following the 2005 National Coverage
Determination, peri-procedural and long-term outcomes have not been described among
Medicare beneficiaries, who are quite different from trial patients, older and with more
comorbidities in general population.
• Understanding the outcomes in these population is particularly important in the light of more
recent study, the Carotid Revascularization Endarterectomy versus Stenting Trial (CREST),
which established CAS as a safe and efficacious alternative to CEA among non-high-surgical
risk patients that also expanded the clinical indication of carotid artery stenting.
• Another motivation to study ‘real world outcomes in the general population is expected
differences in the proficiency of physicians peforming stenting in trial setting vs. real world
practice setting. SAPPHIRE and CREST physicians were enrolled only after having
demonstrated CAS proficiency with low complication rates whereas hands-on experience and
patient outcomes among real-world physicians and hospitals is likely to be more diverse.
• We found that unadjusted mortality risks over study period of 5 years with a mean of 2 years
of follow-up in our population was 32%. Much higher mortality risks observed among certain
subgroups with older age, symptomatic patients and non-elective hospitalizations.
Read the rest of the interviews on MedicalResearch.com
Content NOT an endorsement of efficacy and NOT intended as specific medical advice.
Mortality Risks High In Elderly Patients After Carotid Artery Stenting
MedicalResearch.com Interview with:
Soko Setoguchi, MD DrPH Assistant Professor of Medicine
Harvard Medical School and Harvard School of Public Health
Director of Safety and Outcome Research in Cardiology
Associate Physician in the Division of Pharmacoepidemiology and Pharmacoeconomics Brigham and Women’s Hospital
• Medical Research: What should clinicians and patients take away from your report?
• Dr. Setoguchi: Mortality risks in older Medicare patients who underwent Carotid Artery
Stenting were high. The benefit of Carotid Artery Stenting in older patients, especially those
who are symptomatic from carotid stenosis and who may undergo urgent procedure, is likely
diminished. Therefore, careful clinical assessment of risk benefit should guide the choice of
therapy in older patients.
• Factors such as patient’s age, symptomatic status and urgent hospitalization that we found
associated with higher mortality can guide physicians and patient to make decision to
consider/perform Carotid Artery Stenting. However, such guidance should not overwrite a
decision based on careful clinical assessments about the prognosis of patient and patient
preferences.
Read the rest of the interviews on MedicalResearch.com
Content NOT an endorsement of efficacy and NOT intended as specific medical advice.
Mortality Risks High In Elderly Patients After Carotid Artery Stenting
MedicalResearch.com Interview with:
Soko Setoguchi, MD DrPH Assistant Professor of Medicine
Harvard Medical School and Harvard School of Public Health
Director of Safety and Outcome Research in Cardiology
Associate Physician in the Division of Pharmacoepidemiology and Pharmacoeconomics Brigham and Women’s Hospital
• Medical Research: What recommendations do you have for future research as a result of
this study?
• Dr. Setoguchi: Our study did not have comparable data to SAPPHIRE or CREST to directly
apply the same proficiency criteria used in these studies. The modified criteria we applied
likely underestimated the number of providers who would have been included SAPHIRE or
CREST. However, our data demonstrated that the real-world providers are more diverse in
their proficiency than those who were enrolled in these trials. More studies with high quality
data are needed to carefully assess provider’s proficiency in real-world settings and provide
guidance on the necessary level of proficiency for better outcomes after Carotid Artery
Stenting.
• Citation:
• Jalbert JJ, Nguyen LL, Gerhard-Herman MD, et al. Outcomes After Carotid Artery Stenting in
Medicare Beneficiaries, 2005 to 2009. JAMA Neurol. Published online January 12, 2015.
doi:10.1001/jamaneurol.2014.3638.
Read the rest of the interviews on MedicalResearch.com
Content NOT an endorsement of efficacy and NOT intended as specific medical advice.
Periodontitis Linked To Larger Myocardial Infarct Size
MedicalResearch.com Interview with:
Dr. Francisco Mesa
Department of Periodontics,
School of Dentistry, University of Granada, Spain
• Medical Research: What is the background for this study? What are the main findings?
• Dr. Mesa: The size of an acute myocardial infarct (AMI) is one of the determinants of its
severity, i.e., the degree of myocardial necrosis. This necrosis is indicated by peak troponin I
levels in the blood. Among the acute myocardial infarct patients in our study, mediated
regression analysis demonstrated that troponin I levels were higher, i.e., the infarct size was
larger, in those with chronic periodontitis.
• Medical Research: What should clinicians and patients take away from your report?
• Dr. Mesa: The primary message for dentists is that their contribution to the healthcare of
patients is not limited to oral problems alone, adding value to their profession. Their
knowledge and action can have important systemic repercussions, implying the need for
maximum rigor in the diagnosis and treatment of chronic periodontitis. Around 75-80% of
adult cases of chronic periodontitis can be treated and controlled by the general dentist. A
key message for patients would be to maintain correct oral hygiene habits and pursue what
could be called a “Cardio-Perio-Healthy Life Style”, given the large number of risk factors
shared by cardiovascular and periodontal diseases.
Read the rest of the interviews on MedicalResearch.com
Content NOT an endorsement of efficacy and NOT intended as specific medical advice.
Periodontitis Linked To Larger Myocardial Infarct Size
MedicalResearch.com Interview with:
Dr. Francisco Mesa
Department of Periodontics,
School of Dentistry, University of Granada, Spain
• Medical Research: What recommendations do you have for future research as a result of
this study?
• Dr. Mesa: Periodontics has acquired a dimension that could not have been imagined a few
years ago, when the loss of teeth was the sole complication of periodontal disease. It is now
considered to be a risk factor for systemic diseases (e.g., AMI and low-weight or premature
births) and to represent a major public health problem. This important research line in
Periodontal Medicine will continue to contribute scientific evidence on the influence of
periodontitis on different diseases. Further research is warranted on the inclusion of
periodontitis in risk scores for AMI and other pathologies associated with endothelial
dysfunction, such as erectile dysfunction or dementia. Cardiologists are increasingly aware of
the relationship of chronic periodontitis with atherosclerosis and AMI. In 2012, the European
Society of Cardiology introduced periodontitis into its cardiovascular disease prevention
guidelines, and the American Heart Association has acknowledged its influence on
cardiovascular risk, although it has called for more follow-up studies to elucidate the causal
nature of this relationship.
• Citation:
• Marfil-Álvarez, F. Mesa, A. Arrebola-Moreno, J.A. Ramírez-Hernández, A. Magán-Fernández, F.
O’Valle, P. Galindo-Moreno, and A. Catena
• Acute Myocardial Infarct Size Is Related to Periodontitis Extent and Severity J DENT RES
October 2014 93: 993-998, first published on August 19, 2014
doi:10.1177/002203451454822
Read the rest of the interviews on MedicalResearch.com
Content NOT an endorsement of efficacy and NOT intended as specific medical advice.
New Drug For Hyperkalemia Selectively Traps Potassium
MedicalResearch.com Interview with:
David K. Packham, M.B., B.S., M.D
Royal Melbourne Hospital
Melbourne Renal Research Group VIC 3073, Australia,
• Medical Research: What is the background for this study? What are the main findings?
Dr. Packham: ZS-9 represents a new mechanism of action for addressing hyperkalemia.
Unlike traditional nonspecific organic polymer cationexchangers, ZS-9 is a non-absorbed,
inorganic crystalline potassium-selective cation exchanger that traps excess potassium in the
gastrointestinal tract. It has been evaluated in three prospective, randomized, double-blind,
placebo-controlled studies with over 1100 patients to date, representing the largest ever
clinical development program for hyperkalemia.
• ZS-003 was the first of two pivotal Phase 3 studies that evaluated the safety and efficacy of
ZS-9 in patients with hyperkalemia. In ZS-003, treatment of patients with an oral suspension
of ZS-9 (2.5, 5, or 10 grams, three times a day) resulted in statistically significant and clinically
meaningful reductions in serum potassium, compared with placebo, during the “acute
phase” (first 48 hours), with 99 percent of patients achieving normal potassium levels with
the highest 10 gram dose. During the next 12 days of the trial (the “maintenance phase”), ZS-
9 (5 or 10 grams) given once daily could maintain the corrected potassium levels achieved
during the acute phase. In contrast, patients who were randomized back to placebo after
achieving normal potassium reverted back to hyperkalemia. The tolerability profile has been
favorable, with adverse event rates from ZS-9 similar to that of placebo.
Read the rest of the interviews on MedicalResearch.com
Content NOT an endorsement of efficacy and NOT intended as specific medical advice.
New Drug For Hyperkalemia Selectively Traps Potassium
MedicalResearch.com Interview with:
David K. Packham, M.B., B.S., M.D
Royal Melbourne Hospital
Melbourne Renal Research Group VIC 3073, Australia,
• Medical Research: What should clinicians and patients take away from your report?
• Dr. Packham: Hyperkalemia increases the risk of mortality and limits use of life-saving
renin angiotensin-aldosterone inhibitors (RAASi) in patients with heart failure (HF), chronic
kidney disease (CKD), and diabetes mellitus (DM). Current therapies for hyperkalemia include
loop diurectics and nonspecific organic polymer resins (eg, sodium polystyrene sulfonate);
however, these have inconsistent, questionable efficacy and carry significant gastrointestinal
intolerability. From our studies, ZS-9 has been observed to rapidly and predictably lower and
maintain normal serum potassium levels, while demonstrating a favorable safety and
tolerability profile. ZS-9 offers potential promise for a safe and effective new therapy which
will enable life saving cardio- and renoprotective RAAS therapies in patients with heart
failure, CKD, and DM. Furthermore, other observations suggest that ZS-9 has may improve
serum bicarbonate, thus demonstrating potential to improve metabolic acidosis, a key factor
in the progression of kidney disease.
Read the rest of the interviews on MedicalResearch.com
Content NOT an endorsement of efficacy and NOT intended as specific medical advice.
New Drug For Hyperkalemia Selectively Traps Potassium
MedicalResearch.com Interview with:
David K. Packham, M.B., B.S., M.D
Royal Melbourne Hospital
Melbourne Renal Research Group VIC 3073, Australia,
• Medical Research: What recommendations do you have for future research as a result of this study?
• Dr. Packham: There are 4 potential areas for future research under consideration:
• Emergency department setting for acute therapy of hyperkalemia: Subgroup analyses of our studies have
consistently shown that the potassium lowering effect of ZS-9 is rapid (within an hour) and has greatest
potency among patients with the severest cases of hyperkalemia (ie, serum potassium ≥6mEq/l). Based on
our observations in these studies, we believe that ZS-9 has potential to be an alternative therapy to
emergent dialysis in emergency or hospital settings.
• Long term safety and efficacy: ZS-005 is an ongoing study that is evaluating ZS-9 as long-term (≥12-month)
therapy for patients who require chronic treatment for hyperkalemia.
• Dialysis patients: The ability to address hyperkalemia on non-dialysis days, reduce the pre-dialysis
potassium levels to enable lower potassium baths, and liberate diet are exciting considerations for a non-
polymer based therapy, such as ZS-9, which does not adversely interact with polymer phosphate binders.
• Transplant patients: Anti-rejection medications carry the risk of hyperkalemia and threaten the ability to
optimize transplant medications. Maintenance of normal potassium in transplant patients with ZS-9 may
permit optimal use of transplant medications.
• ACEi, ARB, and Mineralocorticoid Dose Optimization: Hyperkalemia is often the rate-limiting side effect
of these important RAAS medications. Clinical studies of ZS-9 have demonstrated that its effect is
independent of concomitant use of RAAS therapies. Dose optimization may be permitted for such
therapies that are no longer limited because of hyperkalemia.
• Citation:
• Sodium Zirconium Cyclosilicate in Hyperkalemia
• David K. Packham, M.B., B.S., M.D., Henrik S. Rasmussen, M.D., Ph.D., Philip T. Lavin, Ph.D., Mohamed A.
El-Shahawy, M.D., M.P.H., Simon D. Roger, M.D., Geoffrey Block, M.D., Wajeh Qunibi, M.D., Pablo Pergola,
M.D., Ph.D., and Bhupinder Singh, M.D.
• N Engl J Med 2015; 372:222-231 January 15, 2015 DOI: 10.1056/NEJMoa1411487
Read the rest of the interviews on MedicalResearch.com
Content NOT an endorsement of efficacy and NOT intended as specific medical advice.
New Cancer Fighting Strategy Attacks Telomere Pathway In Difficult Tumors
MedicalResearch.com Interview with:
Lee Zou, Ph.D.
Professor of Pathology, Harvard Medical School The Jim & Ann Orr MGH Research Scholar
Associate Scientific Director, MGH Cancer Center
• Medical Research: What is the background for this study? What are the main findings?
Dr. Lee Zou: Cancer cells must rely on telomerase or the alternative lengthening of telomere
(ALT) pathway to maintain telomeres and bypass replicative senescence. The ALT pathway is
active in about 10-15% of human cancers, and it is particularly prevalent in specific cancer
types, such as osteosarcoma, glioblastoma, and neuroendocrine pancreatic tumors. ALT is a
recombination-mediated process. Whether the reliance of cancer cells on alternative
lengthening of telomere can be exploited therapeutically was not known.
• In our study, we discovered that the ATR kinase is a key regulator of alternative lengthening
of telomere. We found that ATR inhibitors disrupt ALT effectively. Furthermore, we found that
ATR inhibitors selectively kill ALT-positive cancer cells in a panel of caner cell lines. These
findings have suggested the first rational therapeutic strategy for the treatment of ALT-
positive cancer.
Read the rest of the interviews on MedicalResearch.com
Content NOT an endorsement of efficacy and NOT intended as specific medical advice.
New Cancer Fighting Strategy Attacks Telomere Pathway In Difficult Tumors
MedicalResearch.com Interview with:
Lee Zou, Ph.D.
Professor of Pathology, Harvard Medical School The Jim & Ann Orr MGH Research Scholar
Associate Scientific Director, MGH Cancer Center
• Medical Research: What should clinicians and patients take away from your report?
• Dr. Lee Zou: Many of the ALT-positive cancers are difficult-to-treat cancers. Our findings have
provided clinicians with a new therapeutic strategy to explore in preclinical and clinical
studies. In particular, several ATR inhibitors have been recently developed and are entering
clinical trials. I hope that our findings will motivate clinician to investigate the efficacy of
alternative lengthening of telomere inhibitors in the treatment of ALT-positive cancers.
• Our findings have provided cancer patients with new hope. However, it is important to note
that further preclinical and clinical work is still needed to validate and refine this new
therapeutic strategy.
Read the rest of the interviews on MedicalResearch.com
Content NOT an endorsement of efficacy and NOT intended as specific medical advice.
New Cancer Fighting Strategy Attacks Telomere Pathway In Difficult Tumors
MedicalResearch.com Interview with:
Lee Zou, Ph.D.
Professor of Pathology, Harvard Medical School The Jim & Ann Orr MGH Research Scholar
Associate Scientific Director, MGH Cancer Center
• Medical Research: What recommendations do you have for future research as a result of
this study?
• Dr. Lee Zou: I hope that our findings will motivate clinicians and patients to explore this new
therapeutic strategy is clinical trials. Our work also suggest that DNA recombination could be
a good therapeutic target in certain cancers. For future research, it would be interesting to
identify additional cancers that are dependent on DNA recombination and to target them
with ATR inhibitors.
• Citation:
• Science. 2015 Jan 16;347(6219):273-7. doi: 10.1126/science.1257216.
• Alternative lengthening of telomeres renders cancer cells hypersensitive to ATR inhibitors.
• Flynn RL1, Cox KE2, Jeitany M3, Wakimoto H4, Bryll AR2, Ganem NJ2, Bersani F5, Pineda JR3,
Suvà ML6, Benes CH7, Haber DA5, Boussin FD3, Zou L8.
Read the rest of the interviews on MedicalResearch.com
Content NOT an endorsement of efficacy and NOT intended as specific medical advice.
Breast Cancer Patients May Benefit From Detailed Nutrition Monitoring
MedicalResearch.com Interview with:
Cecilia Cesa Schiavon
Department of Nutrition, Federal University of Santa Catarina
Florianópolis, Santa Catarina, Brazil
• Medical Research: What is the background for this study? What are the main findings?
Response: The study was based on a nutritional intervention for patients undergoing
treatment for breast cancer. The intervention took place right after the surgical procedure
and lasted about a year, until the end of chemotherapy. The patients were submitted to a
special methodology of intervention, aimed at increasing fruit and vegetable intake and
reducing red and processed meat, following the World Cancer Research Fund and the
American Institute for Cancer Research in the document entitled Food, Nutrition, Physical
Activity and the prevention of Cancer: A Global Perspective”.
• The main findings show that women undergoing breast cancer treatment may benefit from
immediate, individualized, and detailed nutrition monitoring through appropriate nutrition
education.
• Medical Research: What should clinicians and patients take away from your report?
• Response: I think that clinicians and patients can consider the possibility of improving the
quality of the diet of patients, even during breast cancer treatment. And it is also a big
motivation to know that a healthy diet can contribute to a reduction of the side effects of
chemotherapy, and in consequence this can help reduce the risk of breast cancer recurrence.
Read the rest of the interviews on MedicalResearch.com
Content NOT an endorsement of efficacy and NOT intended as specific medical advice.
Breast Cancer Patients May Benefit From Detailed Nutrition Monitoring
MedicalResearch.com Interview with:
Cecilia Cesa Schiavon
Department of Nutrition, Federal University of Santa Catarina
Florianópolis, Santa Catarina, Brazil
• Medical Research: What recommendations do you have for future research as a result of
this study?
• Response: We suggest a study with more time and more resources to have a larger sample
size and to include patients from other health centers.
• Additionally, the ideal situation would be to be able to supply the patients with fruits and
vegetables to guarantee easy access and, consequently, monitor the consumption more
closely.
• Citation:
• J Nutr Educ Behav. 2015 Jan-Feb;47(1):2-9. doi: 10.1016/j.jneb.2014.09.005.
• Nutrition education intervention for women with breast cancer: effect on nutritional
factors and oxidative stress.
• Schiavon CC1, Vieira FG1, Ceccatto V1, de Liz S1, Cardoso AL1, Sabel C2, Gonzalez-Chica DA1, da
Silva EL2, Galvan D1, Crippa CG3, Di Pietro PF4.
Read the rest of the interviews on MedicalResearch.com
Content NOT an endorsement of efficacy and NOT intended as specific medical advice.
Healthy Diet Lowers Diabetes Risk in All Ethnic Groups
MedicalResearch.com Interview with:
Jinnie J. Rhee
Department of Epidemiology, Harvard School of Public Health,
Boston, MA
• Medical Research: What is the background for this study? What are the main findings?
Response: The goal of this study was to see if the dietary determinants of type 2 diabetes
observed in predominantly white populations were similar to those in other racial and ethnic
groups. We created a dietary diabetes risk reduction score using eight different dietary
factors found to be associated with risk of type 2 diabetes, where a higher score indicates a
healthier overall diet (A higher score included low intakes trans fat, sugar-sweetened
beverages, and red and processed meats; lower glycemic index; and higher intakes of cereal
fiber, nuts, and coffee; and higher polyunsaturated to saturated fat ratio). We found a
protective association of similar magnitude between a healthy overall diet and type 2
diabetes risk in all racial and ethnic groups. However, in terms of the actual number of
preventable cases, a healthier diet conferred even greater benefit for minority women
because they were initially at higher risk than white women.
• This study is significant because diabetes is a rapidly growing epidemic in most parts of the
world, but most previous studies of diet and diabetes have been conducted in populations of
European origin. This analysis was very powerful because it combined two large populations
with a total of 156,030 women who were followed for up to 28 years with many repeated
assessments of diet. This allowed us to conduct detailed analyses within specific racial and
ethnic groups.
Read the rest of the interviews on MedicalResearch.com
Content NOT an endorsement of efficacy and NOT intended as specific medical advice.
Healthy Diet Lowers Diabetes Risk in All Ethnic Groups
MedicalResearch.com Interview with:
Jinnie J. Rhee
Department of Epidemiology, Harvard School of Public Health,
Boston, MA
• Medical Research: What should clinicians and patients take away from your report?
• Response: These findings suggest that a healthy overall diet can play an important role in the
prevention of type 2 diabetes, particularly in high-risk groups. The take-home message is
that regardless of one’s race and ethnicity, a healthy diet can play an important role in
preventing diabetes, and in fact, the benefit may be greater for minority women who already
have elevated risks of diabetes. A healthier diet, especially that of lower glycemic foods and
lower intakes of sugar sweetened beverages and red and processed meats, and higher
intakes of cereal fiber and coffee should be encouraged.
Read the rest of the interviews on MedicalResearch.com
Content NOT an endorsement of efficacy and NOT intended as specific medical advice.
Healthy Diet Lowers Diabetes Risk in All Ethnic Groups
MedicalResearch.com Interview with:
Jinnie J. Rhee
Department of Epidemiology, Harvard School of Public Health,
Boston, MA
• Medical Research: What recommendations do you have for future research as a result of
this study?
• Response: This study marks a big step forward in diabetes research because it shows that a
dietary pattern found to be beneficial in a population mainly European origin can be
applicable to most of the world’s population of women. Future research should focus on
how this knowledge can be applied into practice. This includes finding ways to address
health disparities that are influenced by race, income, and other socioeconomic factors
• Citation:
• Jinnie J. Rhee, Josiemer Mattei, Michael D. Hughes, Frank B. Hu, and Walter C. Willett
• Dietary Diabetes Risk Reduction Score, Race and Ethnicity, and Risk of Type 2 Diabetes in
Women Diabetes Care published ahead of print January 15, 2015, doi:10.2337/dc14-1986
Read the rest of the interviews on MedicalResearch.com
Content NOT an endorsement of efficacy and NOT intended as specific medical advice.
Buddy System May You Swap Bad Habits For Good Ones
MedicalResearch.com Interview with:
Jane Wardle
University College London
• Medical Research: What is the background for this study? What are the main findings?
Dr. Wardle: Previous studies have shown that couples tend to have similar health behaviours
to one another, but no studies had compared having a partner who takes up a healthy
behaviour (e.g. quits smoking) with having one whose behaviour is consistently healthy (e.g.
never smoked). Nor have there been other studies in the older age group – our participants
were over 60 on average. We used data from 3722 couples participating in the English
Longitudinal Study of Ageing (ELSA) to explore this issue for three behaviours: smoking,
physical activity, and weight loss. For each behaviour, we found that when one partner
changed their behaviour, the other partner was more likely to make a positive change, and
the effect was stronger than having a partner whose behaviour was consistently healthy (i.e.
never smoked/always exercised).
• Medical Research: What should clinicians and patients take away from your report?
• Dr. Wardle: We know from decades of research that information alone is not enough to get
everyone to take up a healthy lifestyle. Changing together may make things easier. Perhaps
clinicians could encourage people to bring their spouse in too. Your nearest and dearest may
be your best buddy – but where that’s not an option, perhaps another buddy system could
help people to swap their bad habits for healthier ones.
Read the rest of the interviews on MedicalResearch.com
Content NOT an endorsement of efficacy and NOT intended as specific medical advice.
Buddy System May You Swap Bad Habits For Good Ones
MedicalResearch.com Interview with:
Jane Wardle
University College London
• Medical Research: What recommendations do you have for future research as a result of
this study?
• Dr. Wardle: This study is observational – in that we compared couples where the partner
changed with couples where they didn’t. Future research needs to test this in a trial – can we
harness the partner effect in treatment. And can we identify what is important about the
partner effect to translate to other potential buddies?
• Citation:
• Sarah E. Jackson, Andrew Steptoe, Jane Wardle. The Influence of Partner’s Behavior on
Health Behavior Change. JAMA Internal Medicine, 2015;
DOI: 10.1001/jamainternmed.2014.7554
Read the rest of the interviews on MedicalResearch.com
Content NOT an endorsement of efficacy and NOT intended as specific medical advice.
Hardening of Aorta Linked To Poor Cognitive Performance
MedicalResearch.com Interview with:
Jose Gutierrez MD, MPH Assistant Professor of Neurology
Division of Stroke and Cerebrovascular Disease
Columbia University Medical Center NY, NY
• Medical Research: What is the background for this study? What are the main findings?
Dr. Gutierrez: There is growing interest in the effects of vascular health in cognition. The prevailing
thought is that vascular disease leads to worse cognition due to direct structural damage of the
brain, as in the case of brain infarcts, microhemorrhages or white matter hyperintensities, which
are themselves associated with traditional cardiovascular risk factors such as hypertension,
diabetes, smoking etc. Arterial stiffness, particularly of the aorta, has gained interest among
researchers as predictors of vascular disease and worse cognition, but it is not clear whether
arterial stiffness in the absence of traditional definition of vascular disease may be associated with
worse cognition.
• We investigated in a representative sample of the US among adults 60 years or older who
underwent cognitive testing with the Digit Symbol Subtraction test and who also had other
measures of vascular disease, including blood workup, blood pressure measurement and Pulse
pressure. We hypothesized that indirect measures of arterial stiffness such as ABI > 1.3 or pulse
pressure would be associated with worse cognition, even among those without any clinical vascular
disease or traditional vascular risk factors. We Included 2573 US adults in the sample, segregated
those with any self-reported vascular disease or vascular risk factors and we found that among
those without vascular disease or risk factors, an ABI > 1.3 and increased intra-visit blood pressure
variability were predictors of worse cognitive performance compared with those without these
indicators. Among participants with both indirect markers of arterial stiffness, their cognitive
performance was worse that having only one of them suggesting additive effects of these two
variables.
Read the rest of the interviews on MedicalResearch.com
Content NOT an endorsement of efficacy and NOT intended as specific medical advice.
Hardening of Aorta Linked To Poor Cognitive Performance
MedicalResearch.com Interview with:
Jose Gutierrez MD, MPH Assistant Professor of Neurology
Division of Stroke and Cerebrovascular Disease
Columbia University Medical Center NY, NY
• Medical Research: What should clinicians and patients take away from your report?
• Dr. Gutierrez: This results could be used by clinicians to screen patients without apparent
vascular disease for arterial stiffness, and eventually, if positive, refer them for more accurate
measure of arterial stiffness such as carotid-femoral pulse wave velocity. Increasing awareness of
the vascular component of cognition.
• Medical Research: What recommendations do you have for future research as a result of this
study?
• Dr. Gutierrez: We need to know whether medications that can alter aortic stiffness may be
beneficial to preserves cognition among these individuals. Physical exercise appears to have a
protective effects against cognitive decline, so it would be important to see whether exercise
decreases aortic stiffness, thus mediating the effects on cognition. It would be desirable to
investigate further if certain antihypertensives may be preferable in patient with increased aortic
stiffness. Ideally, a patient should come to our office and be screen for vascular disease. If positive,
then aggressively controls their vascular risk factors. If negative, then proceed with measure of
aortic stiffness that may lead to individualized therapies among those not typically consider to have
vascular disease. Evidently, all these algorithm should be tested in clinical trials.
• Citation:
• Gutierrez J, Marshall RS, Lazar RM. Indirect Measures of Arterial Stiffness and Cognitive
Performance in Individuals Without Traditional Vascular Risk Factors or Disease. JAMA Neurol.
Published online January 19, 2015. doi:10.1001/jamaneurol.2014.3873.
• Email *
Read the rest of the interviews on MedicalResearch.com
Content NOT an endorsement of efficacy and NOT intended as specific medical advice.
Is Strict Salt Limit Necessary For Older Adults?
MedicalResearch.com Interview with:
Andreas Kalogeropoulos, MD MPH PhD
Assistant Professor of Medicine (Cardiology)
Emory University School of Medicine Emory Clinical Cardiovascular Research Institute Atlanta GA 30322
• Medical Research: What is the background for this study? What are the main findings?
• Dr. Kalogeropoulos: There is ongoing debate on how low should we go when it
comes to dietary sodium (salt) restriction recommendations. In this study, we examined the
association between self-reported dietary sodium intake and 10-year risk for death,
cardiovascular disease, and heart failure in approximately 2,600 adults 71-80 years
old. The subjects (women: 51.2%; white: 61.7%; black: 38.3%) were participants of the
community-based Health, Aging, and Body Composition Study, which is sponsored by NIH
and focuses on aging processes, i.e. was not specifically designed to address the issue of
dietary salt intake. Also, it is important to note that salt intake was self-reported (not
objectively measured) using a food frequency questionnaire, which underestimates salt
intake. Keeping these limitations in mind, we did not observe a significant association
between self-reported sodium intake and 10-year mortality, cardiovascular disease, and
heart failure. Ten-year mortality was lower in the group reporting 1500–2300 mg daily
sodium intake (30.7%) compared to those reporting daily intake less than 1500 mg (33.8%) or
over 2300 mg (35.2%); however, this difference was not statistically significant. The 10-year
event rates for cardiovascular disease (28.5%, 28.2%, and 29.7%) and heart failure (15.7%,
14.3%, and 15.5%) were also comparable across the <1500-mg, 1500-2300-mg, and >2300-
mg dietary sodium intake groups.
Read the rest of the interviews on MedicalResearch.com
Content NOT an endorsement of efficacy and NOT intended as specific medical advice.
Is Strict Salt Limit Necessary For Older Adults?
MedicalResearch.com Interview with:
Andreas Kalogeropoulos, MD MPH PhD
Assistant Professor of Medicine (Cardiology)
Emory University School of Medicine Emory Clinical Cardiovascular Research Institute Atlanta GA 30322
• Medical Research: What should clinicians and patients take away from your report?
• Dr. Kalogeropoulos: The current CDC guidelines for dietary sodium intake, i.e. <2300 mg for
the general population (corresponding to 6 grams or a teaspoon of salt) with <1500 mg
reserved for high risk groups, including older adults, are probably the best approach until
more data are available. Although our findings do not support the strict (<1500 mg daily)
sodium restriction recommendation for the general population of older adults, there was no
signal of harm either. However, a more conservative approach to sodium restriction (that is,
targeting <2300 mg/d) might be appropriate for older adults until more data become
available. Of note, our findings do not apply to older adults with pre-existing cardiovascular
disease or heart failure.
• Patients should not interpret our findings as a “license” to consume more salt.
First, average dietary salt intake is already too high in most Western or
westernized communities and well above the average reported intake in our study. Second,
there is no question whether moderating salt intake provides health benefits. We know that
going from high-salt (e.g. 12 g or two teaspoons a day) to low-salt (6 g or one teaspoon a day)
diet is good for health. We are now trying to fine-tune the most appropriate level of “low”
that provides the most benefit, i.e., would that be 6 g a day or lower? Third, high salt intake
(i.e., far more than a teaspoon daily) was associated with a trend towards increased mortality
in our study, too; we just did not have enough participants in this category to be statistically
confident about these trends. Therefore, patients should follow the current guidelines and
consult with their healthcare provider as the best approach.
Read the rest of the interviews on MedicalResearch.com
Content NOT an endorsement of efficacy and NOT intended as specific medical advice.
Is Strict Salt Limit Necessary For Older Adults?
MedicalResearch.com Interview with:
Andreas Kalogeropoulos, MD MPH PhD
Assistant Professor of Medicine (Cardiology)
Emory University School of Medicine Emory Clinical Cardiovascular Research Institute Atlanta GA 30322
• Medical Research: What recommendations do you have for future research as a result of
this study?
• Dr. Kalogeropoulos: Our study is not the first to cast doubt on the strict (<1500 mg daily)
sodium intake recommendation — although it is the first one to look at older adults
specifically. Considering the special case of older adults, in whom comorbidities, inadequate
caloric intake, and medication interactions are additional concerns with very low sodium
intake, we probably need to test the effect of sodium restriction on outcomes explicitly in this
special population before implementing a generalized recommendation for very low (<1500
mg/d) sodium intake. Of note, the Institute of Medicine has highlighted the lack of data on
the appropriate level of sodium restriction for older adults and other special populations,
including patients with pre-existing heart failure. In the latter, a limited number of studies to
date has not corroborated the traditional recommendation for sodium restriction,
highlighting the need for more rigorous studies and more definitive evidence in special
populations.
• Citation:
• Kalogeropoulos AP, Georgiopoulou VV, Murphy RA, et al. Dietary Sodium Content, Mortality,
and Risk for Cardiovascular Events in Older Adults: The Health, Aging, and Body Composition
(Health ABC) Study. JAMA Intern Med. Published online January 19, 2015.
doi:10.1001/jamainternmed.2014.6278.
Read the rest of the interviews on MedicalResearch.com
Content NOT an endorsement of efficacy and NOT intended as specific medical advice.
VA Study Examines Variability of Costs and Mortality of Stent Surgery
MedicalResearch.com Interview with:
Dr. P. Michael Ho, MD PhD
Denver Veteran Affairs Medical Center,
University of Colorado, Denver, Section of Cardiology Denver, Colorado 80220.
• Medical Research: What is the background for this study? What are the main findings?
• Dr. Ho: There is increasing interest in measuring health care value, particularly as the healthcare
system moves towards accountable care. Value in health care focuses on measuring outcomes
achieved relative to costs for a cycle of care. Attaining high value care – good clinical outcomes at
low costs – is of interest to patients, providers, health systems, and payers. To date, value
assessments have not been operationalized and applied to specific patient populations. We focused
on percutaneous coronary intervention (PCI) because it is an important aspect of care for patients
with ischemic heart disease, is commonly performed and is a costly procedure. In this study, we
evaluated 1-year risk-adjusted mortality and 1-year risk-standardized costs of care for all patients
who underwent PCI in the VA healthcare system from 2008 to 2010.
• We found that median one-year unadjusted hospital mortality rate was 6.13% (interquartile range
4.51% to 7.34% across hospitals). Four hospitals were significantly above the one-year risk
standardized median mortality rate, with median mortality ratios ranging from 1.23 to 1.28; no
hospitals were significantly below median mortality. Median 1-year total unadjusted hospital costs
were $46,302 (IQR of $37,291 to $57,886) per patient. There were 16 hospitals above and 19
hospitals below the risk standardized average cost, with risk standardized ratios ranging from 0.45
to 2.09 reflecting much larger magnitude of variability in costs compared to mortality. These
findings suggest that there are opportunities to improve PCI healthcare by reducing costs without
compromising outcomes. This approach of evaluating outcomes and costs together may be a model
for other health systems and accountable care organizations interested in operationalizing value
measurement.
Read the rest of the interviews on MedicalResearch.com
Content NOT an endorsement of efficacy and NOT intended as specific medical advice.
VA Study Examines Variability of Costs and Mortality of Stent Surgery
MedicalResearch.com Interview with:
Dr. P. Michael Ho, MD PhD
Denver Veteran Affairs Medical Center,
University of Colorado, Denver, Section of Cardiology Denver, Colorado 80220.
• Medical Research: What should clinicians and patients take away from your report?
• Dr. Ho: These findings are more relevant to hospitals, health care systems, and policy makers. Most of the
variation across hospitals of longitudinal PCI care occurs as a result of differences in costs with much less
variation in patient outcomes. This suggests that value of PCI care for the VA may be enhanced through
reducing unnecessary variation in costs of care, without compromising patient outcomes.
• Medical Research: What recommendations do you have for future research as a result of this study?
• Dr. Ho: Future research will be needed to identify specific reasons for the cost variations identified, one of
which may be regional differences in practice patterns. For inpatient care, there may be structural factors
related to staffing of cath labs and/or supply costs (e.g., coronary stents) which are negotiated by
individual hospitals. For outpatient care, there will be opportunities to explore differences in follow-up
care, some of which may be related to intensity of care provided, frequency of cardiac testing and/or need
for non-cardiac related care. We are planning on conducting qualitative interviews at hospitals to identify
reasons for the cost variations found in this study.
• Citation:
• 1-Year Risk-Adjusted Mortality and Costs of Percutaneous Coronary Intervention in the Veterans Health
Administration: Insights From the VA CART Program
• P. Michael Ho, MD, PhD, Colin I. O’Donnell, MS, Steven M. Bradley, MD, MPH Gary K. Grunwald, PhD
Christian Helfrich, PhD Michael Chapko, PhD Chuan-Fen Liu, PhD Thomas M. Maddox, MD, MSc Thomas T.
Tsai, MD, MSc Robert L. Jesse, MD, PhD, Stephan D. Fihn, MD, MPH John S. Rumsfeld, MD, PhD
• Journal of the American College of Cardiology
Volume 65, Issue 3, 27 January 2015, Pages 236–242
Read the rest of the interviews on MedicalResearch.com
Content NOT an endorsement of efficacy and NOT intended as specific medical advice.
Sitting Time Linked To Increased Heart Disease and Cancer Mortality
MedicalResearch.com Interview with:
David Alter, MD, PhD FRCPC Senior Scientist
Toronto Rehabilitation Institute-University Health Network and Institute for Clinical Evaluative Sciences
Research Director, Cardiac Rehabilitation and Secondary Prevention Program Toronto Rehabilitation Institute
Medical Research: What is the background for this study? What are the main findings?
Dr. Alter: We knew going into the study that exercise was an important lifestyle factor that
improved health. We also knew from studies that sedentary time was associated with deleterious
health-effects. What we didn’t know was whether the health-outcome effects of sedentary time
and exercise were really one and the same (i.e., albeit opposite ends of the same spectrum) or
alternatively, whether the health effects of each were independent of one another. We explored
over 9000 published studies to quantify the health-outcome effects associated with sedentary
behaviour and extracted only those which took into account both sedentary time and exercise.
We found a consistent association between sedentary time and a host of health outcomes
independent of exercise. Specifically, after controlling for an individual’s exercising behaviour,
sitting-time was associated with a 15-20% higher risk of death, heart-disease, death from heart
disease, cancer-incidence, and death from cancer. Sitting time was also independently associated
with a marked (i.e., 90% increase) in the risk for diabetes after controlling for exercise. In short,
sedentary times and exercise are each independently associated with health outcomes. We
hypothesize that the two may have different mechanism, and may require different therapeutic
strategies. But, the health-outcome implications of both are each important in their own right.
Read the rest of the interviews on MedicalResearch.com
Content NOT an endorsement of efficacy and NOT intended as specific medical advice.
Sitting Time Linked To Increased Heart Disease and Cancer Mortality
MedicalResearch.com Interview with:
David Alter, MD, PhD FRCPC Senior Scientist
Toronto Rehabilitation Institute-University Health Network and Institute for Clinical Evaluative Sciences
Research Director, Cardiac Rehabilitation and Secondary Prevention Program Toronto Rehabilitation Institute
• Medical Research: What should clinicians and patients take away from your report?
• Dr. Alter: A call to action. Several indeed:
• First, from a public-health perspective, the signal between sedentary time and deleterious health-
outcomes is one that is consistent for multiple diseases and health outcomes. Traditionally, public health
and health promotion messaging has focused on exercise alone (among other healthy lifestyle behaviours
like diet and smoking-cessation etc.). However, we believe that our study justifies the need for a
modification in public-health and health promotion messaging – We must speak about the benefits of
both “Exercise” and “Avoidance of excessive sitting” – A two-pronged message – both are important, but
distinct…. “An hour per day of exercise does not give the right of passage to sit for the remaining 23” – It’s
time we incorporate the sedentary message into public health.
• Second, while we need more scientifically-rigorous research on how best to decrease sitting times, there
are things we can do as patients and health care providers in the meantime as the science of sitting times
evolves. “Breaking cultural norms” is a challenge. However, we need to begin by providing greater
specificity and granularity to our recommendations. The generic “sit less” “exercise more” is often lost on
us. We know this already. What we often struggle with, however, is the “how”. While admittedly
anecdotal, I’ve had success by setting out a step-by-step plan for my patients. I set forth tangible and
achievable tasks that patients can follow which promote self-monitoring, goal-set, and feedback.
• How do I do this for my patients? I begin by having patients monitor their sleeping times, sitting times,
and exercise times per day using either a simple app or log-book diary I provide to them. Generally, one
week of monitoring serves as a starting point. This task does two things: It engages patients into
monitoring their sitting time behaviours to draw awareness to the issue, and serves as baseline set of
measures against which I compare when monitoring progress. . Then, I have patients set a goal – “how
many fewer minutes per day should you sit over the next week?” And, “how will you achieve this?”
Read the rest of the interviews on MedicalResearch.com
Content NOT an endorsement of efficacy and NOT intended as specific medical advice.
Sitting Time Linked To Increased Heart Disease and Cancer Mortality
MedicalResearch.com Interview with:
David Alter, MD, PhD FRCPC Senior Scientist
Toronto Rehabilitation Institute-University Health Network and Institute for Clinical Evaluative Sciences
Research Director, Cardiac Rehabilitation and Secondary Prevention Program Toronto Rehabilitation Institute
• Some practical tips? Begin by having the patient setting a timer at work to remind them on
the half-hour, to get up and stretch from their sitting position and stroll for 2 minutes; There
are a host of other activities…..For example, every commercial break on TV, do the same;
watch the last 5-10 minutes of a show standing than sitting….great for sports….Have patients
multitask during their leisure time. For example, I am currently typing this email while on my
elliptical…spelling errors aside, it feels like a “healthy way to message” in more ways than
one. Finally, I have patients return where I will provide them with feedback. What health
benefits did they achieve over the last week or two by sitting less? This, in my experience, is
VERY IMPORTANT. What patients (and providers) don’t realize is how much can be achieved
with just little changes. For example, we burn 2 times as many calories standing as we do
sitting. For an average 70 kg male, that is an incremental difference of 70 Kcal/hour. Walking
will burn even more than standing- Doing the math, you realize that you can burn quite a
number of calories per day through these simple strategies. Cumulatively over a week,
month, or year, the numbers add up.
• There are many potential approaches when feeding back such information – – the aim is for
patients to internalize the health-benefits of this behaviour change. There are all sorts of
software apps and tools that can help… And more will undoubtedly emerge in the coming
years.
• The bottom line: reducing sitting time is a distinct priority from our 30-45 minutes per day
requirement for moderate to vigorous exercise – -both need to be done. Both need to be
promoted. Both should be thought of as “health hygiene”.
Read the rest of the interviews on MedicalResearch.com
Content NOT an endorsement of efficacy and NOT intended as specific medical advice.
Sitting Time Linked To Increased Heart Disease and Cancer Mortality
MedicalResearch.com Interview with:
David Alter, MD, PhD FRCPC Senior Scientist
Toronto Rehabilitation Institute-University Health Network and Institute for Clinical Evaluative Sciences
Research Director, Cardiac Rehabilitation and Secondary Prevention Program Toronto Rehabilitation Institute
• Medical Research: What recommendations do you have for future research as a result of this study?
• Dr. Alter: More research is needed. Indeed, we have barely scratched the surface here. This underscores
another call to action – – that to our research community. First, our study demonstrated that the
measurement of sitting times varied from study to study. Definitions were all over the map in the studies
explored. Therefore, we need a more uniform and consistent definition so that we are “speaking the same
language” when embarking on further research. Second, we need to better understand the mechanisms
by which sedentary behaviours lead to poorer health, and how this differs from exercise and it’s health
promoting attributes. Plausible mechanisms are needed to help build the credibility of the message – –
especially for clinicians and scientists. Teasing out the mechanisms will also give way to new approaches
and solutions. So, we need to look “underneath the hood” and dig deeper into mechanisms. Third, we
need to better understand how best to “intervene” in order to change the sedentary behaviours of
individuals and what effects if any, such therapeutic interventions have on health outcomes. Solutions
need be integrative and interwoven within an individual’s 12 hour waking day period- – In so doing, we
also need to find better and more innovative ways of quantifying the health-benefits of sitting-less – –
These will not be as aesthetic as weight loss. Nor might they feel as intensive as exercise. We need to
incorporate health metrics which connect with patients in a way that motivates them from pushing on and
sustaining their reductions in sitting times – – after all, solutions may be quite simple and intuitive – but
will they resonate with patients? Finally, our approach to solutions may differ from individual to
individual For example, our study suggested that the deleterious effects of sitting time were most
pronounced among those who did not participate in any regular exercise. On the flip side, the adverse
health effects of sitting may diminished in magnitude among those who do participate avidly in regular
exercise. This suggests that our research priorities may necessitate a better understanding of the
population – – who are the sedentary non-avid exercisers in the population? Do they need different
approaches to their otherwise sedentary but avid-exercise counterparts?
Read the rest of the interviews on MedicalResearch.com
Content NOT an endorsement of efficacy and NOT intended as specific medical advice.
Sitting Time Linked To Increased Heart Disease and Cancer Mortality
MedicalResearch.com Interview with:
David Alter, MD, PhD FRCPC Senior Scientist
Toronto Rehabilitation Institute-University Health Network and Institute for Clinical Evaluative Sciences
Research Director, Cardiac Rehabilitation and Secondary Prevention Program Toronto Rehabilitation Institute
• In short, there is a lot more work to be done. We are hoping that this study serves as a call to
action for all!
• Citation:
• Biswas A, Oh PI, Faulkner GE, Bajaj RR, Silver MA, Mitchell MS, et al. Sedentary Time and Its
Association With Risk for Disease Incidence, Mortality, and Hospitalization in Adults: A
Systematic Review and Meta-analysis. Ann Intern Med. 2015;162:123-132. doi:10.7326/M14-
1651
Read the rest of the interviews on MedicalResearch.com
Content NOT an endorsement of efficacy and NOT intended as specific medical advice.
High-Intensity Statin Therapy Underutilized After Heart Attack
MedicalResearch.com Interview with:
Dr. Robert S. Rosenson, MD Professor, Cardiology
Icahn School of Medicine at Mount Sinai Cardiovascular Institute
New York, New York 10029
• Medical Research: What is the background for this study? What are the main findings?
Dr. Rosenson: High intensity statin therapy is evidence-based and guideline directed for
patients with acute coronary syndromes. In a 5 percent random sample of Medicare
patients, we investigated the utilization of high vs low-moderate dosage statin in older
adjusts who were admitted with an acute myocardial infarction of severe myocardial
ischemia requiring hospitalization for a revascularization procedure (PCI or CABG).
• We report that only 27 percent of hospitalized patients received high-intensity statin therapy
based on their first outpatient fill for a statin medication. The most important determinant
for the utilization of statin therapy is the dosage of the statin previously prescribed as an
outpatient. When patients were started on a high-intensity statin, the continued use
diminished in the ensuing year
• Medical Research: What should clinicians and patients take away from your report?
• Dr. Rosenson: Evidence-based guidelines recommend in hospital initiation of high-intensity
statin for patients with acute coronary syndromes, and this therapy should be continued
long-term. It is important to understand that we do not treat these patients to a LDL
cholesterol target as statins have other salutary effects that extend beyond cholesterol
lowering.
Read the rest of the interviews on MedicalResearch.com
Content NOT an endorsement of efficacy and NOT intended as specific medical advice.
High-Intensity Statin Therapy Underutilized After Heart Attack
MedicalResearch.com Interview with:
Dr. Robert S. Rosenson, MD Professor, Cardiology
Icahn School of Medicine at Mount Sinai Cardiovascular Institute
New York, New York 10029
• Medical Research: What recommendations do you have for future research as a result of
this study?
• Dr. Rosenson: We have plans to evaluate treatment patterns and the impact on
cardiovascular events.
• Citation:
• Underutilization of High-Intensity Statin Therapy After Hospitalization for Coronary Heart
Disease
• Robert S. Rosenson, MD , Shia T. Kent, PhD Todd M. Brown, MD Michael E. Farkouh, MD
Emily B. Levitan, PhD†, Huifeng Yun, MD, PhD Pradeep Sharma, MS Monika M. Safford, MD
Meredith Kilgore, PhD Paul Muntner, PhD Vera Bittner, MD
• Journal of the American College of Cardiology
• Volume 65, Issue 3, 27 January 2015, Pages 270–277
Read the rest of the interviews on MedicalResearch.com
Content NOT an endorsement of efficacy and NOT intended as specific medical advice.
Key Clock Genes Linked To Alcohol Use and Abuse
MedicalResearch.com Interview with:
Timo Partonen MD, Research Professor
National Institute for Health and Welfare
Helsinki, Finland
• Medical Research: What is the background for this study? What are the main findings?
Response: Alcohol-use disorders are often comorbid conditions with mood and anxiety disorders.
Clinical studies have demonstrated that there are abnormalities in circadian rhythms and intrinsic
clocks in patients with alcohol-use disorders. Circadian clock gene variants are therefore a fruitful
target of interest.
• The main findings are that variants of key clock genes, namely those of ARNTL, ARNTL2, PER1 and
PER2, have association with alcohol consumption, with alcohol abuse, or with alcohol dependence.
It is of interest that variants of a fifth clock gene of key importance, that is those of CLOCK, are
associated with alcohol-use disorders only if comorbid with depressive disorders.
• Medical Research: What should clinicians and patients take away from your report?
• Response: It is too early to give any implication for clinical practice on the basis of these findings.
So, the current take-home message to clinicians or patients is that research on the topic has
advanced, but clinical applications still remain to be seen.
• Medical Research: What recommendations do you have for future research as a result of this
study?
• Response: Characterization of the functional relevance of these genetic variants and the
mechanistic basis of these associations need to be elucidated in order to have the first insight of
the potential, if any, small-molecule medication options for treatment of alcohol-use disorders.
• Citation:
• Clock Genes in Human Alcohol Abuse and Comorbid Conditions
• Timo Partonen Alcohol Available online 16 January 2015
Read the rest of the interviews on MedicalResearch.com
Content NOT an endorsement of efficacy and NOT intended as specific medical advice.
Punitive Approach To Drug Dependence May Be Counterproductive
MedicalResearch.com Interview with:
Katharine A. Neill PhD
Alfred C. Glassell III Postdoctoral Fellow in Drug Policy Baker Institute, Rice University
Houston, TX 77005
• Medical Research: What is the background for this study? What are the main findings?
Dr. Neill: This study is a historical review of drug policy in 20th century United States. It examines
drug policy decisions and consequences through a socio-political lens and argues that the
prominence of the law-and-order approach to dealing with drug offenders–that emphasizes
punishment and incarceration over prevention and treatment–is a result of the construction of
drug offenders as social deviants that threaten society. This punitive model has been especially
harmful because it has occurred to the detriment of harm reduction approaches to drug use that
have greater potential to negate the negative individual and public health consequences of drug
use.
• Medical Research: What should clinicians and patients take away from your report?
• Dr. Neill: For clinicians who see drug dependence as a disease, the negative constructions of drug
users is a big reason why harm reduction efforts–which may include education and prevention
campaigns, expanded treatment services for individuals seeking help, prevention of dangerous drug
use such as driving under the influence, and safe addiction maintenance for individuals who do not
or cannot stop using via needle-exchange programs–have not received the same kind of funding as
law enforcement approaches. Not only do powerful public safety interests have an interest in
maintaining a punishment-over-treatment focus, but the general public’s perceptions of drug users
as deviant and dangerous makes the funding of harm reduction efforts politically unpopular,
especially programs such as needle-exchange. I suspect that many patients who have sought help
for their drug dependence have had direct experiences with this kind of stigma.
Read the rest of the interviews on MedicalResearch.com
Content NOT an endorsement of efficacy and NOT intended as specific medical advice.
Punitive Approach To Drug Dependence May Be Counterproductive
MedicalResearch.com Interview with:
Katharine A. Neill PhD
Alfred C. Glassell III Postdoctoral Fellow in Drug Policy Baker Institute, Rice University
Houston, TX 77005
• Medical Research: What recommendations do you have for future research as a result of
this study?
• Dr. Neill: Several things have to happen to refocus attention on drug dependence as a disease
that requires medical treatment rather than a deviant behavior that deserves punishment.
Changing public perceptions of drug offenders is critical. This is happening in small ways
already. Drug war fatigue and a new crop of prescription opiate and heroin users that is
whiter and more female than previous users has facilitated more calls for drug treatment and
less focus on incarceration. But it is important that drug dependence is viewed as a disease
regardless of who the users are. Research can help with this; studies that look at the
effectiveness of drug treatment at preventing relapses, the public health benefits of needle-
exchange programs, and the negative consequences of incarceration on drug users’ future
drug use and recidivism rates, are all examples of important issues that can be explored
more. It is also critical to have education campaigns that bring awareness to the public about
the true nature of drug addiction.
• Citation:
• World Medical & Health Policy Vol 6 Issue 4
• Katharine A. Neill Article first published online: 21 DEC 2014
• DOI: 10.1002/wmh3.123
Read the rest of the interviews on MedicalResearch.com
Content NOT an endorsement of efficacy and NOT intended as specific medical advice.
MedicalResearch.com:  Medical Research Exclusive Interviews Jan 23 2015
MedicalResearch.com:  Medical Research Exclusive Interviews Jan 23 2015
MedicalResearch.com:  Medical Research Exclusive Interviews Jan 23 2015
MedicalResearch.com:  Medical Research Exclusive Interviews Jan 23 2015
MedicalResearch.com:  Medical Research Exclusive Interviews Jan 23 2015
MedicalResearch.com:  Medical Research Exclusive Interviews Jan 23 2015
MedicalResearch.com:  Medical Research Exclusive Interviews Jan 23 2015
MedicalResearch.com:  Medical Research Exclusive Interviews Jan 23 2015
MedicalResearch.com:  Medical Research Exclusive Interviews Jan 23 2015
MedicalResearch.com:  Medical Research Exclusive Interviews Jan 23 2015
MedicalResearch.com:  Medical Research Exclusive Interviews Jan 23 2015
MedicalResearch.com:  Medical Research Exclusive Interviews Jan 23 2015
MedicalResearch.com:  Medical Research Exclusive Interviews Jan 23 2015
MedicalResearch.com:  Medical Research Exclusive Interviews Jan 23 2015
MedicalResearch.com:  Medical Research Exclusive Interviews Jan 23 2015
MedicalResearch.com:  Medical Research Exclusive Interviews Jan 23 2015
MedicalResearch.com:  Medical Research Exclusive Interviews Jan 23 2015
MedicalResearch.com:  Medical Research Exclusive Interviews Jan 23 2015
MedicalResearch.com:  Medical Research Exclusive Interviews Jan 23 2015
MedicalResearch.com:  Medical Research Exclusive Interviews Jan 23 2015
MedicalResearch.com:  Medical Research Exclusive Interviews Jan 23 2015
MedicalResearch.com:  Medical Research Exclusive Interviews Jan 23 2015
MedicalResearch.com:  Medical Research Exclusive Interviews Jan 23 2015
MedicalResearch.com:  Medical Research Exclusive Interviews Jan 23 2015
MedicalResearch.com:  Medical Research Exclusive Interviews Jan 23 2015
MedicalResearch.com:  Medical Research Exclusive Interviews Jan 23 2015
MedicalResearch.com:  Medical Research Exclusive Interviews Jan 23 2015
MedicalResearch.com:  Medical Research Exclusive Interviews Jan 23 2015
MedicalResearch.com:  Medical Research Exclusive Interviews Jan 23 2015
MedicalResearch.com:  Medical Research Exclusive Interviews Jan 23 2015
MedicalResearch.com:  Medical Research Exclusive Interviews Jan 23 2015
MedicalResearch.com:  Medical Research Exclusive Interviews Jan 23 2015
MedicalResearch.com:  Medical Research Exclusive Interviews Jan 23 2015
MedicalResearch.com:  Medical Research Exclusive Interviews Jan 23 2015
MedicalResearch.com:  Medical Research Exclusive Interviews Jan 23 2015
MedicalResearch.com:  Medical Research Exclusive Interviews Jan 23 2015
MedicalResearch.com:  Medical Research Exclusive Interviews Jan 23 2015
MedicalResearch.com:  Medical Research Exclusive Interviews Jan 23 2015
MedicalResearch.com:  Medical Research Exclusive Interviews Jan 23 2015
MedicalResearch.com:  Medical Research Exclusive Interviews Jan 23 2015
MedicalResearch.com:  Medical Research Exclusive Interviews Jan 23 2015
MedicalResearch.com:  Medical Research Exclusive Interviews Jan 23 2015
MedicalResearch.com:  Medical Research Exclusive Interviews Jan 23 2015

More Related Content

What's hot

Healthcare Communications Study Among Physicians: Medical Monitor 2013
Healthcare Communications Study Among Physicians: Medical Monitor 2013Healthcare Communications Study Among Physicians: Medical Monitor 2013
Healthcare Communications Study Among Physicians: Medical Monitor 2013Joshua Spiegel
 
Recruitment & Retention: Breaking Down the Barriers to eConsent Adoption
Recruitment & Retention: Breaking Down the Barriers to eConsent AdoptionRecruitment & Retention: Breaking Down the Barriers to eConsent Adoption
Recruitment & Retention: Breaking Down the Barriers to eConsent AdoptionCRF Health
 
Mosio's Clinical Trial Patient Recruitment and Retention Ebook (First Edition)
Mosio's Clinical Trial Patient Recruitment and Retention Ebook (First Edition)Mosio's Clinical Trial Patient Recruitment and Retention Ebook (First Edition)
Mosio's Clinical Trial Patient Recruitment and Retention Ebook (First Edition)Mosio
 
Patient recruitment into clinical trials presentation
Patient recruitment into clinical trials presentationPatient recruitment into clinical trials presentation
Patient recruitment into clinical trials presentationBeshr Nammouz
 
CROs Driving Changes in Patient Recruitment
CROs Driving Changes in Patient RecruitmentCROs Driving Changes in Patient Recruitment
CROs Driving Changes in Patient RecruitmentJohn Reites
 
Methods of patient recruitment
Methods of patient recruitmentMethods of patient recruitment
Methods of patient recruitmentswati2084
 
Hospitalist M&A Landscape – Winter 2018 – 2019
Hospitalist M&A Landscape – Winter 2018 – 2019Hospitalist M&A Landscape – Winter 2018 – 2019
Hospitalist M&A Landscape – Winter 2018 – 2019Duff & Phelps
 
Clinical trial recruitment overview
Clinical trial recruitment overviewClinical trial recruitment overview
Clinical trial recruitment overviewUsama Malik
 
Best Practices in Clinical Study Recruitment
Best Practices in Clinical Study RecruitmentBest Practices in Clinical Study Recruitment
Best Practices in Clinical Study RecruitmentCTSI at UCSF
 
Use of translation apps and websites in health care settings
Use of translation apps and websites in health care settingsUse of translation apps and websites in health care settings
Use of translation apps and websites in health care settingsBen Harris-Roxas
 
Tricks of the Trade: Patient Recruitment & Retention for Different Study Types
Tricks of the Trade: Patient Recruitment & Retention for Different Study TypesTricks of the Trade: Patient Recruitment & Retention for Different Study Types
Tricks of the Trade: Patient Recruitment & Retention for Different Study TypesImperial CRS
 
Analysis of The Influence Patient Safety, Service Quality, Marketing Mix, Tow...
Analysis of The Influence Patient Safety, Service Quality, Marketing Mix, Tow...Analysis of The Influence Patient Safety, Service Quality, Marketing Mix, Tow...
Analysis of The Influence Patient Safety, Service Quality, Marketing Mix, Tow...inventionjournals
 
Recruitment Metrics from a Direct-to-Patient Approach to Enroll Patients in a...
Recruitment Metrics from a Direct-to-Patient Approach to Enroll Patients in a...Recruitment Metrics from a Direct-to-Patient Approach to Enroll Patients in a...
Recruitment Metrics from a Direct-to-Patient Approach to Enroll Patients in a...John Reites
 
Innovations conference 2014 erica wales does an online anti-cancer medicati...
Innovations conference 2014   erica wales does an online anti-cancer medicati...Innovations conference 2014   erica wales does an online anti-cancer medicati...
Innovations conference 2014 erica wales does an online anti-cancer medicati...Cancer Institute NSW
 
PFCC Methodology Meets "Lean," Part I
PFCC Methodology Meets "Lean,"  Part IPFCC Methodology Meets "Lean,"  Part I
PFCC Methodology Meets "Lean," Part IEngagingPatients
 
Recruitment & Retention Plan: A Sample Strategy Presentation
Recruitment & Retention Plan: A Sample Strategy PresentationRecruitment & Retention Plan: A Sample Strategy Presentation
Recruitment & Retention Plan: A Sample Strategy PresentationMarwah Zagzoug, PhD
 

What's hot (20)

Healthcare Communications Study Among Physicians: Medical Monitor 2013
Healthcare Communications Study Among Physicians: Medical Monitor 2013Healthcare Communications Study Among Physicians: Medical Monitor 2013
Healthcare Communications Study Among Physicians: Medical Monitor 2013
 
Recruitment & Retention: Breaking Down the Barriers to eConsent Adoption
Recruitment & Retention: Breaking Down the Barriers to eConsent AdoptionRecruitment & Retention: Breaking Down the Barriers to eConsent Adoption
Recruitment & Retention: Breaking Down the Barriers to eConsent Adoption
 
Mosio's Clinical Trial Patient Recruitment and Retention Ebook (First Edition)
Mosio's Clinical Trial Patient Recruitment and Retention Ebook (First Edition)Mosio's Clinical Trial Patient Recruitment and Retention Ebook (First Edition)
Mosio's Clinical Trial Patient Recruitment and Retention Ebook (First Edition)
 
Patient recruitment into clinical trials presentation
Patient recruitment into clinical trials presentationPatient recruitment into clinical trials presentation
Patient recruitment into clinical trials presentation
 
CROs Driving Changes in Patient Recruitment
CROs Driving Changes in Patient RecruitmentCROs Driving Changes in Patient Recruitment
CROs Driving Changes in Patient Recruitment
 
Methods of patient recruitment
Methods of patient recruitmentMethods of patient recruitment
Methods of patient recruitment
 
Hospitalist M&A Landscape – Winter 2018 – 2019
Hospitalist M&A Landscape – Winter 2018 – 2019Hospitalist M&A Landscape – Winter 2018 – 2019
Hospitalist M&A Landscape – Winter 2018 – 2019
 
Clinical trial recruitment overview
Clinical trial recruitment overviewClinical trial recruitment overview
Clinical trial recruitment overview
 
Clinical Trial Recruitment
Clinical Trial RecruitmentClinical Trial Recruitment
Clinical Trial Recruitment
 
Survey Analyses for Implementing an Electronic Information System to Enhance ...
Survey Analyses for Implementing an Electronic Information System to Enhance ...Survey Analyses for Implementing an Electronic Information System to Enhance ...
Survey Analyses for Implementing an Electronic Information System to Enhance ...
 
Clorox Report
Clorox ReportClorox Report
Clorox Report
 
Best Practices in Clinical Study Recruitment
Best Practices in Clinical Study RecruitmentBest Practices in Clinical Study Recruitment
Best Practices in Clinical Study Recruitment
 
Use of translation apps and websites in health care settings
Use of translation apps and websites in health care settingsUse of translation apps and websites in health care settings
Use of translation apps and websites in health care settings
 
Tricks of the Trade: Patient Recruitment & Retention for Different Study Types
Tricks of the Trade: Patient Recruitment & Retention for Different Study TypesTricks of the Trade: Patient Recruitment & Retention for Different Study Types
Tricks of the Trade: Patient Recruitment & Retention for Different Study Types
 
Analysis of The Influence Patient Safety, Service Quality, Marketing Mix, Tow...
Analysis of The Influence Patient Safety, Service Quality, Marketing Mix, Tow...Analysis of The Influence Patient Safety, Service Quality, Marketing Mix, Tow...
Analysis of The Influence Patient Safety, Service Quality, Marketing Mix, Tow...
 
Recruitment Metrics from a Direct-to-Patient Approach to Enroll Patients in a...
Recruitment Metrics from a Direct-to-Patient Approach to Enroll Patients in a...Recruitment Metrics from a Direct-to-Patient Approach to Enroll Patients in a...
Recruitment Metrics from a Direct-to-Patient Approach to Enroll Patients in a...
 
Innovations conference 2014 erica wales does an online anti-cancer medicati...
Innovations conference 2014   erica wales does an online anti-cancer medicati...Innovations conference 2014   erica wales does an online anti-cancer medicati...
Innovations conference 2014 erica wales does an online anti-cancer medicati...
 
PFCC Methodology Meets "Lean," Part I
PFCC Methodology Meets "Lean,"  Part IPFCC Methodology Meets "Lean,"  Part I
PFCC Methodology Meets "Lean," Part I
 
AsokanRatheesh
AsokanRatheeshAsokanRatheesh
AsokanRatheesh
 
Recruitment & Retention Plan: A Sample Strategy Presentation
Recruitment & Retention Plan: A Sample Strategy PresentationRecruitment & Retention Plan: A Sample Strategy Presentation
Recruitment & Retention Plan: A Sample Strategy Presentation
 

Viewers also liked

MedicalResearch.com: Medical Research Exclusive Interviews December 31 2014
MedicalResearch.com:  Medical Research Exclusive Interviews December 31 2014MedicalResearch.com:  Medical Research Exclusive Interviews December 31 2014
MedicalResearch.com: Medical Research Exclusive Interviews December 31 2014Marie Benz MD FAAD
 
MedicalResearch.com: Medical Research Exclusive Interviews April 8 2015
MedicalResearch.com:  Medical Research Exclusive Interviews April 8  2015MedicalResearch.com:  Medical Research Exclusive Interviews April 8  2015
MedicalResearch.com: Medical Research Exclusive Interviews April 8 2015Marie Benz MD FAAD
 
Medical research slideshare_feb-24_2015
Medical research slideshare_feb-24_2015Medical research slideshare_feb-24_2015
Medical research slideshare_feb-24_2015Marie Benz MD FAAD
 
MedicalResearch.com: Medical Research Exclusive Interviews February 10 2015
MedicalResearch.com:  Medical Research Exclusive Interviews February 10  2015MedicalResearch.com:  Medical Research Exclusive Interviews February 10  2015
MedicalResearch.com: Medical Research Exclusive Interviews February 10 2015Marie Benz MD FAAD
 
MedicalResearch.com: Medical Research Exclusive Interviews December 21 2014
MedicalResearch.com:  Medical Research Exclusive Interviews December 21 2014MedicalResearch.com:  Medical Research Exclusive Interviews December 21 2014
MedicalResearch.com: Medical Research Exclusive Interviews December 21 2014Marie Benz MD FAAD
 
MedicalResearch.com: Medical Research Exclusive Interviews January 28 2015
MedicalResearch.com:  Medical Research Exclusive Interviews January 28 2015MedicalResearch.com:  Medical Research Exclusive Interviews January 28 2015
MedicalResearch.com: Medical Research Exclusive Interviews January 28 2015Marie Benz MD FAAD
 
MedicalResearch.com: Medical Research Exclusive Interviews February 17 2015
MedicalResearch.com:  Medical Research Exclusive Interviews February 17  2015MedicalResearch.com:  Medical Research Exclusive Interviews February 17  2015
MedicalResearch.com: Medical Research Exclusive Interviews February 17 2015Marie Benz MD FAAD
 
MedicalResearch.com: Medical Research Exclusive Interviews January 15 2014
MedicalResearch.com:  Medical Research Exclusive Interviews January 15  2014MedicalResearch.com:  Medical Research Exclusive Interviews January 15  2014
MedicalResearch.com: Medical Research Exclusive Interviews January 15 2014Marie Benz MD FAAD
 
MedicalResearch.com: Medical Research Exclusive Interviews March 24 2015
MedicalResearch.com:  Medical Research Exclusive Interviews March 24 2015MedicalResearch.com:  Medical Research Exclusive Interviews March 24 2015
MedicalResearch.com: Medical Research Exclusive Interviews March 24 2015Marie Benz MD FAAD
 
MedicalResearch.com: Medical Research Exclusive Interviews April 1 2015
MedicalResearch.com:  Medical Research Exclusive Interviews April 1  2015MedicalResearch.com:  Medical Research Exclusive Interviews April 1  2015
MedicalResearch.com: Medical Research Exclusive Interviews April 1 2015Marie Benz MD FAAD
 
Medical research slideshare_march 12_2015
Medical research slideshare_march 12_2015 Medical research slideshare_march 12_2015
Medical research slideshare_march 12_2015 Marie Benz MD FAAD
 
MedicalResearch.com: Medical Research Exclusive Interviews May 18 2015
MedicalResearch.com:  Medical Research Exclusive Interviews May 18 2015MedicalResearch.com:  Medical Research Exclusive Interviews May 18 2015
MedicalResearch.com: Medical Research Exclusive Interviews May 18 2015Marie Benz MD FAAD
 
Medical research slideshare_june_24_2015
Medical research slideshare_june_24_2015 Medical research slideshare_june_24_2015
Medical research slideshare_june_24_2015 Marie Benz MD FAAD
 
MedicalResearch.com: Medical Research Exclusive Interviews June 9 2015
MedicalResearch.com:  Medical Research Exclusive Interviews June 9 2015MedicalResearch.com:  Medical Research Exclusive Interviews June 9 2015
MedicalResearch.com: Medical Research Exclusive Interviews June 9 2015Marie Benz MD FAAD
 
MedicalResearch.com: Medical Research Exclusive Interviews July 29 2015
MedicalResearch.com:  Medical Research Exclusive Interviews July 29 2015MedicalResearch.com:  Medical Research Exclusive Interviews July 29 2015
MedicalResearch.com: Medical Research Exclusive Interviews July 29 2015Marie Benz MD FAAD
 
MedicalResearch.com: Medical Research Exclusive Interviews April 20 2015
MedicalResearch.com:  Medical Research Exclusive Interviews April 20  2015MedicalResearch.com:  Medical Research Exclusive Interviews April 20  2015
MedicalResearch.com: Medical Research Exclusive Interviews April 20 2015Marie Benz MD FAAD
 
MedicalResearch.com: Medical Research Exclusive Interviews July 20 2015
MedicalResearch.com:  Medical Research Exclusive Interviews July 20  2015MedicalResearch.com:  Medical Research Exclusive Interviews July 20  2015
MedicalResearch.com: Medical Research Exclusive Interviews July 20 2015Marie Benz MD FAAD
 
MedicalResearch.com: Medical Research Exclusive Interviews June 6 2015
MedicalResearch.com:  Medical Research Exclusive Interviews June 6 2015MedicalResearch.com:  Medical Research Exclusive Interviews June 6 2015
MedicalResearch.com: Medical Research Exclusive Interviews June 6 2015Marie Benz MD FAAD
 
MedicalResearch.com: Medical Research Exclusive Interviews June 26 2015
MedicalResearch.com:  Medical Research Exclusive Interviews June 26 2015MedicalResearch.com:  Medical Research Exclusive Interviews June 26 2015
MedicalResearch.com: Medical Research Exclusive Interviews June 26 2015Marie Benz MD FAAD
 

Viewers also liked (19)

MedicalResearch.com: Medical Research Exclusive Interviews December 31 2014
MedicalResearch.com:  Medical Research Exclusive Interviews December 31 2014MedicalResearch.com:  Medical Research Exclusive Interviews December 31 2014
MedicalResearch.com: Medical Research Exclusive Interviews December 31 2014
 
MedicalResearch.com: Medical Research Exclusive Interviews April 8 2015
MedicalResearch.com:  Medical Research Exclusive Interviews April 8  2015MedicalResearch.com:  Medical Research Exclusive Interviews April 8  2015
MedicalResearch.com: Medical Research Exclusive Interviews April 8 2015
 
Medical research slideshare_feb-24_2015
Medical research slideshare_feb-24_2015Medical research slideshare_feb-24_2015
Medical research slideshare_feb-24_2015
 
MedicalResearch.com: Medical Research Exclusive Interviews February 10 2015
MedicalResearch.com:  Medical Research Exclusive Interviews February 10  2015MedicalResearch.com:  Medical Research Exclusive Interviews February 10  2015
MedicalResearch.com: Medical Research Exclusive Interviews February 10 2015
 
MedicalResearch.com: Medical Research Exclusive Interviews December 21 2014
MedicalResearch.com:  Medical Research Exclusive Interviews December 21 2014MedicalResearch.com:  Medical Research Exclusive Interviews December 21 2014
MedicalResearch.com: Medical Research Exclusive Interviews December 21 2014
 
MedicalResearch.com: Medical Research Exclusive Interviews January 28 2015
MedicalResearch.com:  Medical Research Exclusive Interviews January 28 2015MedicalResearch.com:  Medical Research Exclusive Interviews January 28 2015
MedicalResearch.com: Medical Research Exclusive Interviews January 28 2015
 
MedicalResearch.com: Medical Research Exclusive Interviews February 17 2015
MedicalResearch.com:  Medical Research Exclusive Interviews February 17  2015MedicalResearch.com:  Medical Research Exclusive Interviews February 17  2015
MedicalResearch.com: Medical Research Exclusive Interviews February 17 2015
 
MedicalResearch.com: Medical Research Exclusive Interviews January 15 2014
MedicalResearch.com:  Medical Research Exclusive Interviews January 15  2014MedicalResearch.com:  Medical Research Exclusive Interviews January 15  2014
MedicalResearch.com: Medical Research Exclusive Interviews January 15 2014
 
MedicalResearch.com: Medical Research Exclusive Interviews March 24 2015
MedicalResearch.com:  Medical Research Exclusive Interviews March 24 2015MedicalResearch.com:  Medical Research Exclusive Interviews March 24 2015
MedicalResearch.com: Medical Research Exclusive Interviews March 24 2015
 
MedicalResearch.com: Medical Research Exclusive Interviews April 1 2015
MedicalResearch.com:  Medical Research Exclusive Interviews April 1  2015MedicalResearch.com:  Medical Research Exclusive Interviews April 1  2015
MedicalResearch.com: Medical Research Exclusive Interviews April 1 2015
 
Medical research slideshare_march 12_2015
Medical research slideshare_march 12_2015 Medical research slideshare_march 12_2015
Medical research slideshare_march 12_2015
 
MedicalResearch.com: Medical Research Exclusive Interviews May 18 2015
MedicalResearch.com:  Medical Research Exclusive Interviews May 18 2015MedicalResearch.com:  Medical Research Exclusive Interviews May 18 2015
MedicalResearch.com: Medical Research Exclusive Interviews May 18 2015
 
Medical research slideshare_june_24_2015
Medical research slideshare_june_24_2015 Medical research slideshare_june_24_2015
Medical research slideshare_june_24_2015
 
MedicalResearch.com: Medical Research Exclusive Interviews June 9 2015
MedicalResearch.com:  Medical Research Exclusive Interviews June 9 2015MedicalResearch.com:  Medical Research Exclusive Interviews June 9 2015
MedicalResearch.com: Medical Research Exclusive Interviews June 9 2015
 
MedicalResearch.com: Medical Research Exclusive Interviews July 29 2015
MedicalResearch.com:  Medical Research Exclusive Interviews July 29 2015MedicalResearch.com:  Medical Research Exclusive Interviews July 29 2015
MedicalResearch.com: Medical Research Exclusive Interviews July 29 2015
 
MedicalResearch.com: Medical Research Exclusive Interviews April 20 2015
MedicalResearch.com:  Medical Research Exclusive Interviews April 20  2015MedicalResearch.com:  Medical Research Exclusive Interviews April 20  2015
MedicalResearch.com: Medical Research Exclusive Interviews April 20 2015
 
MedicalResearch.com: Medical Research Exclusive Interviews July 20 2015
MedicalResearch.com:  Medical Research Exclusive Interviews July 20  2015MedicalResearch.com:  Medical Research Exclusive Interviews July 20  2015
MedicalResearch.com: Medical Research Exclusive Interviews July 20 2015
 
MedicalResearch.com: Medical Research Exclusive Interviews June 6 2015
MedicalResearch.com:  Medical Research Exclusive Interviews June 6 2015MedicalResearch.com:  Medical Research Exclusive Interviews June 6 2015
MedicalResearch.com: Medical Research Exclusive Interviews June 6 2015
 
MedicalResearch.com: Medical Research Exclusive Interviews June 26 2015
MedicalResearch.com:  Medical Research Exclusive Interviews June 26 2015MedicalResearch.com:  Medical Research Exclusive Interviews June 26 2015
MedicalResearch.com: Medical Research Exclusive Interviews June 26 2015
 

Similar to MedicalResearch.com: Medical Research Exclusive Interviews Jan 23 2015

A study on effect of cme programs in improving doctors medical practice
A study on effect of cme programs in improving doctors medical practiceA study on effect of cme programs in improving doctors medical practice
A study on effect of cme programs in improving doctors medical practiceBhavik Amin
 
Electronic Media Use in Academic Medical Center Patient Recruitment
Electronic Media Use in Academic Medical Center Patient RecruitmentElectronic Media Use in Academic Medical Center Patient Recruitment
Electronic Media Use in Academic Medical Center Patient RecruitmentIndustry Standard Research
 
How to improve clinical trial recruitment
How to improve clinical trial recruitment How to improve clinical trial recruitment
How to improve clinical trial recruitment Sollers College
 
1 front cover-preface and table of contents
1 front cover-preface and table of contents1 front cover-preface and table of contents
1 front cover-preface and table of contentsPharmacyTechnicianExam
 
MEMOTEXT - University of Toronto Masters in Health Informatics guest lecture ...
MEMOTEXT - University of Toronto Masters in Health Informatics guest lecture ...MEMOTEXT - University of Toronto Masters in Health Informatics guest lecture ...
MEMOTEXT - University of Toronto Masters in Health Informatics guest lecture ...MEMOTEXT Corporation
 
Results of an Online Survey of Stakeholders Regarding Barriers and Solutions ...
Results of an Online Survey of Stakeholders Regarding Barriers and Solutions ...Results of an Online Survey of Stakeholders Regarding Barriers and Solutions ...
Results of an Online Survey of Stakeholders Regarding Barriers and Solutions ...John Reites
 
A dissertation report on analysis of patient satisfaction max polyclinic by ...
A  dissertation report on analysis of patient satisfaction max polyclinic by ...A  dissertation report on analysis of patient satisfaction max polyclinic by ...
A dissertation report on analysis of patient satisfaction max polyclinic by ...Mohammed Yaser Hussain
 
mComply_Brochure_v1.0c.pdf
mComply_Brochure_v1.0c.pdfmComply_Brochure_v1.0c.pdf
mComply_Brochure_v1.0c.pdfSynegys
 
A Leading Patient Experience Survey Platform by MedStatix - White Labeled for...
A Leading Patient Experience Survey Platform by MedStatix - White Labeled for...A Leading Patient Experience Survey Platform by MedStatix - White Labeled for...
A Leading Patient Experience Survey Platform by MedStatix - White Labeled for...MedStatix, LLC
 
Cme model of dmims (du) wardha the 10 point action program for learning and q...
Cme model of dmims (du) wardha the 10 point action program for learning and q...Cme model of dmims (du) wardha the 10 point action program for learning and q...
Cme model of dmims (du) wardha the 10 point action program for learning and q...Alexander Decker
 
Implement a Direct-to-Patient Approach to Increase Patient Engagement and Ret...
Implement a Direct-to-Patient Approach to Increase Patient Engagement and Ret...Implement a Direct-to-Patient Approach to Increase Patient Engagement and Ret...
Implement a Direct-to-Patient Approach to Increase Patient Engagement and Ret...John Reites
 
Health Informatics- Module 4-Chapter 3.pptx
Health Informatics- Module 4-Chapter 3.pptxHealth Informatics- Module 4-Chapter 3.pptx
Health Informatics- Module 4-Chapter 3.pptxArti Parab Academics
 
Importance of research in the feild of medical science
Importance of research in the feild of medical scienceImportance of research in the feild of medical science
Importance of research in the feild of medical scienceIram Anwar
 
A SURVEY ON FACTORS INFLUENCING QUALITY MANAGEMENT WITH REFERENCE TO NURSING ...
A SURVEY ON FACTORS INFLUENCING QUALITY MANAGEMENT WITH REFERENCE TO NURSING ...A SURVEY ON FACTORS INFLUENCING QUALITY MANAGEMENT WITH REFERENCE TO NURSING ...
A SURVEY ON FACTORS INFLUENCING QUALITY MANAGEMENT WITH REFERENCE TO NURSING ...IAEME Publication
 
Patient satisfaction survey.docx
Patient satisfaction survey.docxPatient satisfaction survey.docx
Patient satisfaction survey.docxDaniel320418
 
Example Dissertation Proposal Defense Power Point Slide
Example Dissertation Proposal Defense Power Point SlideExample Dissertation Proposal Defense Power Point Slide
Example Dissertation Proposal Defense Power Point SlideDr. Vince Bridges
 
Your Patients Are Here: Where to Recruit & How to Retain Highly Engaged Patients
Your Patients Are Here: Where to Recruit & How to Retain Highly Engaged PatientsYour Patients Are Here: Where to Recruit & How to Retain Highly Engaged Patients
Your Patients Are Here: Where to Recruit & How to Retain Highly Engaged PatientsImperial CRS
 
MARKETING PRINCIPLESMKTG 305SWOT Assignment – CSUSB.docx
MARKETING PRINCIPLESMKTG 305SWOT Assignment – CSUSB.docxMARKETING PRINCIPLESMKTG 305SWOT Assignment – CSUSB.docx
MARKETING PRINCIPLESMKTG 305SWOT Assignment – CSUSB.docxalfredacavx97
 

Similar to MedicalResearch.com: Medical Research Exclusive Interviews Jan 23 2015 (20)

A study on effect of cme programs in improving doctors medical practice
A study on effect of cme programs in improving doctors medical practiceA study on effect of cme programs in improving doctors medical practice
A study on effect of cme programs in improving doctors medical practice
 
Electronic Media Use in Academic Medical Center Patient Recruitment
Electronic Media Use in Academic Medical Center Patient RecruitmentElectronic Media Use in Academic Medical Center Patient Recruitment
Electronic Media Use in Academic Medical Center Patient Recruitment
 
How to improve clinical trial recruitment
How to improve clinical trial recruitment How to improve clinical trial recruitment
How to improve clinical trial recruitment
 
1 front cover-preface and table of contents
1 front cover-preface and table of contents1 front cover-preface and table of contents
1 front cover-preface and table of contents
 
MEMOTEXT - University of Toronto Masters in Health Informatics guest lecture ...
MEMOTEXT - University of Toronto Masters in Health Informatics guest lecture ...MEMOTEXT - University of Toronto Masters in Health Informatics guest lecture ...
MEMOTEXT - University of Toronto Masters in Health Informatics guest lecture ...
 
Results of an Online Survey of Stakeholders Regarding Barriers and Solutions ...
Results of an Online Survey of Stakeholders Regarding Barriers and Solutions ...Results of an Online Survey of Stakeholders Regarding Barriers and Solutions ...
Results of an Online Survey of Stakeholders Regarding Barriers and Solutions ...
 
A dissertation report on analysis of patient satisfaction max polyclinic by ...
A  dissertation report on analysis of patient satisfaction max polyclinic by ...A  dissertation report on analysis of patient satisfaction max polyclinic by ...
A dissertation report on analysis of patient satisfaction max polyclinic by ...
 
mComply_Brochure_v1.0c.pdf
mComply_Brochure_v1.0c.pdfmComply_Brochure_v1.0c.pdf
mComply_Brochure_v1.0c.pdf
 
A Leading Patient Experience Survey Platform by MedStatix - White Labeled for...
A Leading Patient Experience Survey Platform by MedStatix - White Labeled for...A Leading Patient Experience Survey Platform by MedStatix - White Labeled for...
A Leading Patient Experience Survey Platform by MedStatix - White Labeled for...
 
Cme model of dmims (du) wardha the 10 point action program for learning and q...
Cme model of dmims (du) wardha the 10 point action program for learning and q...Cme model of dmims (du) wardha the 10 point action program for learning and q...
Cme model of dmims (du) wardha the 10 point action program for learning and q...
 
Implement a Direct-to-Patient Approach to Increase Patient Engagement and Ret...
Implement a Direct-to-Patient Approach to Increase Patient Engagement and Ret...Implement a Direct-to-Patient Approach to Increase Patient Engagement and Ret...
Implement a Direct-to-Patient Approach to Increase Patient Engagement and Ret...
 
Health Informatics- Module 4-Chapter 3.pptx
Health Informatics- Module 4-Chapter 3.pptxHealth Informatics- Module 4-Chapter 3.pptx
Health Informatics- Module 4-Chapter 3.pptx
 
Importance of research in the feild of medical science
Importance of research in the feild of medical scienceImportance of research in the feild of medical science
Importance of research in the feild of medical science
 
A SURVEY ON FACTORS INFLUENCING QUALITY MANAGEMENT WITH REFERENCE TO NURSING ...
A SURVEY ON FACTORS INFLUENCING QUALITY MANAGEMENT WITH REFERENCE TO NURSING ...A SURVEY ON FACTORS INFLUENCING QUALITY MANAGEMENT WITH REFERENCE TO NURSING ...
A SURVEY ON FACTORS INFLUENCING QUALITY MANAGEMENT WITH REFERENCE TO NURSING ...
 
Patient satisfaction survey.docx
Patient satisfaction survey.docxPatient satisfaction survey.docx
Patient satisfaction survey.docx
 
Example Dissertation Proposal Defense Power Point Slide
Example Dissertation Proposal Defense Power Point SlideExample Dissertation Proposal Defense Power Point Slide
Example Dissertation Proposal Defense Power Point Slide
 
Your Patients Are Here: Where to Recruit & How to Retain Highly Engaged Patients
Your Patients Are Here: Where to Recruit & How to Retain Highly Engaged PatientsYour Patients Are Here: Where to Recruit & How to Retain Highly Engaged Patients
Your Patients Are Here: Where to Recruit & How to Retain Highly Engaged Patients
 
Growing By Understanding
Growing By UnderstandingGrowing By Understanding
Growing By Understanding
 
MARKETING PRINCIPLESMKTG 305SWOT Assignment – CSUSB.docx
MARKETING PRINCIPLESMKTG 305SWOT Assignment – CSUSB.docxMARKETING PRINCIPLESMKTG 305SWOT Assignment – CSUSB.docx
MARKETING PRINCIPLESMKTG 305SWOT Assignment – CSUSB.docx
 
MADLENARTICLES
MADLENARTICLESMADLENARTICLES
MADLENARTICLES
 

More from Marie Benz MD FAAD

MedicalResearch.com: Medical Research Exclusive Interviews August 8 2015
MedicalResearch.com:  Medical Research Exclusive Interviews August 8 2015MedicalResearch.com:  Medical Research Exclusive Interviews August 8 2015
MedicalResearch.com: Medical Research Exclusive Interviews August 8 2015Marie Benz MD FAAD
 
MedicalResearch.com: Medical Research Exclusive Interviews July 24 2015
MedicalResearch.com:  Medical Research Exclusive Interviews July 24 2015MedicalResearch.com:  Medical Research Exclusive Interviews July 24 2015
MedicalResearch.com: Medical Research Exclusive Interviews July 24 2015Marie Benz MD FAAD
 
MedicalResearch.com: Medical Research Exclusive Interviews July 16 2015
MedicalResearch.com:  Medical Research Exclusive Interviews July 16 2015MedicalResearch.com:  Medical Research Exclusive Interviews July 16 2015
MedicalResearch.com: Medical Research Exclusive Interviews July 16 2015Marie Benz MD FAAD
 
MedicalResearch.com: Medical Research Exclusive Interviews July 9 2015
MedicalResearch.com:  Medical Research Exclusive Interviews July 9 2015MedicalResearch.com:  Medical Research Exclusive Interviews July 9 2015
MedicalResearch.com: Medical Research Exclusive Interviews July 9 2015Marie Benz MD FAAD
 
MedicalResearch.com: Medical Research Exclusive Interviews July 2 2015
MedicalResearch.com:  Medical Research Exclusive Interviews July 2 2015MedicalResearch.com:  Medical Research Exclusive Interviews July 2 2015
MedicalResearch.com: Medical Research Exclusive Interviews July 2 2015Marie Benz MD FAAD
 
Medical research slideshare_june_18_2015
Medical research slideshare_june_18_2015 Medical research slideshare_june_18_2015
Medical research slideshare_june_18_2015 Marie Benz MD FAAD
 
Medical research slideshare_june_18_2015
Medical research slideshare_june_18_2015 Medical research slideshare_june_18_2015
Medical research slideshare_june_18_2015 Marie Benz MD FAAD
 
MedicalResearch.com: Medical Research Exclusive Interviews June 11 2015
MedicalResearch.com:  Medical Research Exclusive Interviews June 11 2015MedicalResearch.com:  Medical Research Exclusive Interviews June 11 2015
MedicalResearch.com: Medical Research Exclusive Interviews June 11 2015Marie Benz MD FAAD
 
MedicalResearch.com: Medical Research Exclusive Interviews May 26 2015
MedicalResearch.com:  Medical Research Exclusive Interviews May 26 2015MedicalResearch.com:  Medical Research Exclusive Interviews May 26 2015
MedicalResearch.com: Medical Research Exclusive Interviews May 26 2015Marie Benz MD FAAD
 
MedicalResearch.com: Medical Research Exclusive Interviews May 12 2015
MedicalResearch.com:  Medical Research Exclusive Interviews May 12 2015MedicalResearch.com:  Medical Research Exclusive Interviews May 12 2015
MedicalResearch.com: Medical Research Exclusive Interviews May 12 2015Marie Benz MD FAAD
 
Medical research slideshare_may_6_2015
Medical research slideshare_may_6_2015 Medical research slideshare_may_6_2015
Medical research slideshare_may_6_2015 Marie Benz MD FAAD
 
MedicalResearch.com: Medical Research Exclusive Interviews April 28 2015
MedicalResearch.com:  Medical Research Exclusive Interviews April 28  2015MedicalResearch.com:  Medical Research Exclusive Interviews April 28  2015
MedicalResearch.com: Medical Research Exclusive Interviews April 28 2015Marie Benz MD FAAD
 
MedicalResearch.com: Medical Research Exclusive Interviews March 17 2015
MedicalResearch.com:  Medical Research Exclusive Interviews March 17 2015MedicalResearch.com:  Medical Research Exclusive Interviews March 17 2015
MedicalResearch.com: Medical Research Exclusive Interviews March 17 2015Marie Benz MD FAAD
 
MedicalResearch.com: Medical Research Exclusive Interviews March 12 2015
MedicalResearch.com:  Medical Research Exclusive Interviews March 12 2015MedicalResearch.com:  Medical Research Exclusive Interviews March 12 2015
MedicalResearch.com: Medical Research Exclusive Interviews March 12 2015Marie Benz MD FAAD
 
MedicalResearch.com: Medical Research Exclusive Interviews March 5 2015
MedicalResearch.com:  Medical Research Exclusive Interviews March 5  2015MedicalResearch.com:  Medical Research Exclusive Interviews March 5  2015
MedicalResearch.com: Medical Research Exclusive Interviews March 5 2015Marie Benz MD FAAD
 

More from Marie Benz MD FAAD (15)

MedicalResearch.com: Medical Research Exclusive Interviews August 8 2015
MedicalResearch.com:  Medical Research Exclusive Interviews August 8 2015MedicalResearch.com:  Medical Research Exclusive Interviews August 8 2015
MedicalResearch.com: Medical Research Exclusive Interviews August 8 2015
 
MedicalResearch.com: Medical Research Exclusive Interviews July 24 2015
MedicalResearch.com:  Medical Research Exclusive Interviews July 24 2015MedicalResearch.com:  Medical Research Exclusive Interviews July 24 2015
MedicalResearch.com: Medical Research Exclusive Interviews July 24 2015
 
MedicalResearch.com: Medical Research Exclusive Interviews July 16 2015
MedicalResearch.com:  Medical Research Exclusive Interviews July 16 2015MedicalResearch.com:  Medical Research Exclusive Interviews July 16 2015
MedicalResearch.com: Medical Research Exclusive Interviews July 16 2015
 
MedicalResearch.com: Medical Research Exclusive Interviews July 9 2015
MedicalResearch.com:  Medical Research Exclusive Interviews July 9 2015MedicalResearch.com:  Medical Research Exclusive Interviews July 9 2015
MedicalResearch.com: Medical Research Exclusive Interviews July 9 2015
 
MedicalResearch.com: Medical Research Exclusive Interviews July 2 2015
MedicalResearch.com:  Medical Research Exclusive Interviews July 2 2015MedicalResearch.com:  Medical Research Exclusive Interviews July 2 2015
MedicalResearch.com: Medical Research Exclusive Interviews July 2 2015
 
Medical research slideshare_june_18_2015
Medical research slideshare_june_18_2015 Medical research slideshare_june_18_2015
Medical research slideshare_june_18_2015
 
Medical research slideshare_june_18_2015
Medical research slideshare_june_18_2015 Medical research slideshare_june_18_2015
Medical research slideshare_june_18_2015
 
MedicalResearch.com: Medical Research Exclusive Interviews June 11 2015
MedicalResearch.com:  Medical Research Exclusive Interviews June 11 2015MedicalResearch.com:  Medical Research Exclusive Interviews June 11 2015
MedicalResearch.com: Medical Research Exclusive Interviews June 11 2015
 
MedicalResearch.com: Medical Research Exclusive Interviews May 26 2015
MedicalResearch.com:  Medical Research Exclusive Interviews May 26 2015MedicalResearch.com:  Medical Research Exclusive Interviews May 26 2015
MedicalResearch.com: Medical Research Exclusive Interviews May 26 2015
 
MedicalResearch.com: Medical Research Exclusive Interviews May 12 2015
MedicalResearch.com:  Medical Research Exclusive Interviews May 12 2015MedicalResearch.com:  Medical Research Exclusive Interviews May 12 2015
MedicalResearch.com: Medical Research Exclusive Interviews May 12 2015
 
Medical research slideshare_may_6_2015
Medical research slideshare_may_6_2015 Medical research slideshare_may_6_2015
Medical research slideshare_may_6_2015
 
MedicalResearch.com: Medical Research Exclusive Interviews April 28 2015
MedicalResearch.com:  Medical Research Exclusive Interviews April 28  2015MedicalResearch.com:  Medical Research Exclusive Interviews April 28  2015
MedicalResearch.com: Medical Research Exclusive Interviews April 28 2015
 
MedicalResearch.com: Medical Research Exclusive Interviews March 17 2015
MedicalResearch.com:  Medical Research Exclusive Interviews March 17 2015MedicalResearch.com:  Medical Research Exclusive Interviews March 17 2015
MedicalResearch.com: Medical Research Exclusive Interviews March 17 2015
 
MedicalResearch.com: Medical Research Exclusive Interviews March 12 2015
MedicalResearch.com:  Medical Research Exclusive Interviews March 12 2015MedicalResearch.com:  Medical Research Exclusive Interviews March 12 2015
MedicalResearch.com: Medical Research Exclusive Interviews March 12 2015
 
MedicalResearch.com: Medical Research Exclusive Interviews March 5 2015
MedicalResearch.com:  Medical Research Exclusive Interviews March 5  2015MedicalResearch.com:  Medical Research Exclusive Interviews March 5  2015
MedicalResearch.com: Medical Research Exclusive Interviews March 5 2015
 

Recently uploaded

Aspirin presentation slides by Dr. Rewas Ali
Aspirin presentation slides by Dr. Rewas AliAspirin presentation slides by Dr. Rewas Ali
Aspirin presentation slides by Dr. Rewas AliRewAs ALI
 
Artifacts in Nuclear Medicine with Identifying and resolving artifacts.
Artifacts in Nuclear Medicine with Identifying and resolving artifacts.Artifacts in Nuclear Medicine with Identifying and resolving artifacts.
Artifacts in Nuclear Medicine with Identifying and resolving artifacts.MiadAlsulami
 
Bangalore Call Girls Hebbal Kempapura Number 7001035870 Meetin With Bangalor...
Bangalore Call Girls Hebbal Kempapura Number 7001035870  Meetin With Bangalor...Bangalore Call Girls Hebbal Kempapura Number 7001035870  Meetin With Bangalor...
Bangalore Call Girls Hebbal Kempapura Number 7001035870 Meetin With Bangalor...narwatsonia7
 
Call Girls Service Pune Vaishnavi 9907093804 Short 1500 Night 6000 Best call ...
Call Girls Service Pune Vaishnavi 9907093804 Short 1500 Night 6000 Best call ...Call Girls Service Pune Vaishnavi 9907093804 Short 1500 Night 6000 Best call ...
Call Girls Service Pune Vaishnavi 9907093804 Short 1500 Night 6000 Best call ...Miss joya
 
Bangalore Call Girls Marathahalli 📞 9907093804 High Profile Service 100% Safe
Bangalore Call Girls Marathahalli 📞 9907093804 High Profile Service 100% SafeBangalore Call Girls Marathahalli 📞 9907093804 High Profile Service 100% Safe
Bangalore Call Girls Marathahalli 📞 9907093804 High Profile Service 100% Safenarwatsonia7
 
Russian Call Girls in Pune Tanvi 9907093804 Short 1500 Night 6000 Best call g...
Russian Call Girls in Pune Tanvi 9907093804 Short 1500 Night 6000 Best call g...Russian Call Girls in Pune Tanvi 9907093804 Short 1500 Night 6000 Best call g...
Russian Call Girls in Pune Tanvi 9907093804 Short 1500 Night 6000 Best call g...Miss joya
 
Call Girls Cuttack Just Call 9907093804 Top Class Call Girl Service Available
Call Girls Cuttack Just Call 9907093804 Top Class Call Girl Service AvailableCall Girls Cuttack Just Call 9907093804 Top Class Call Girl Service Available
Call Girls Cuttack Just Call 9907093804 Top Class Call Girl Service AvailableDipal Arora
 
Vip Call Girls Anna Salai Chennai 👉 8250192130 ❣️💯 Top Class Girls Available
Vip Call Girls Anna Salai Chennai 👉 8250192130 ❣️💯 Top Class Girls AvailableVip Call Girls Anna Salai Chennai 👉 8250192130 ❣️💯 Top Class Girls Available
Vip Call Girls Anna Salai Chennai 👉 8250192130 ❣️💯 Top Class Girls AvailableNehru place Escorts
 
VIP Call Girls Pune Vani 9907093804 Short 1500 Night 6000 Best call girls Ser...
VIP Call Girls Pune Vani 9907093804 Short 1500 Night 6000 Best call girls Ser...VIP Call Girls Pune Vani 9907093804 Short 1500 Night 6000 Best call girls Ser...
VIP Call Girls Pune Vani 9907093804 Short 1500 Night 6000 Best call girls Ser...Miss joya
 
VIP Call Girls Indore Kirti 💚😋 9256729539 🚀 Indore Escorts
VIP Call Girls Indore Kirti 💚😋  9256729539 🚀 Indore EscortsVIP Call Girls Indore Kirti 💚😋  9256729539 🚀 Indore Escorts
VIP Call Girls Indore Kirti 💚😋 9256729539 🚀 Indore Escortsaditipandeya
 
Call Girls Service Navi Mumbai Samaira 8617697112 Independent Escort Service ...
Call Girls Service Navi Mumbai Samaira 8617697112 Independent Escort Service ...Call Girls Service Navi Mumbai Samaira 8617697112 Independent Escort Service ...
Call Girls Service Navi Mumbai Samaira 8617697112 Independent Escort Service ...Call girls in Ahmedabad High profile
 
Bangalore Call Girls Majestic 📞 9907093804 High Profile Service 100% Safe
Bangalore Call Girls Majestic 📞 9907093804 High Profile Service 100% SafeBangalore Call Girls Majestic 📞 9907093804 High Profile Service 100% Safe
Bangalore Call Girls Majestic 📞 9907093804 High Profile Service 100% Safenarwatsonia7
 
(👑VVIP ISHAAN ) Russian Call Girls Service Navi Mumbai🖕9920874524🖕Independent...
(👑VVIP ISHAAN ) Russian Call Girls Service Navi Mumbai🖕9920874524🖕Independent...(👑VVIP ISHAAN ) Russian Call Girls Service Navi Mumbai🖕9920874524🖕Independent...
(👑VVIP ISHAAN ) Russian Call Girls Service Navi Mumbai🖕9920874524🖕Independent...Taniya Sharma
 
Call Girl Number in Panvel Mumbai📲 9833363713 💞 Full Night Enjoy
Call Girl Number in Panvel Mumbai📲 9833363713 💞 Full Night EnjoyCall Girl Number in Panvel Mumbai📲 9833363713 💞 Full Night Enjoy
Call Girl Number in Panvel Mumbai📲 9833363713 💞 Full Night Enjoybabeytanya
 
VIP Service Call Girls Sindhi Colony 📳 7877925207 For 18+ VIP Call Girl At Th...
VIP Service Call Girls Sindhi Colony 📳 7877925207 For 18+ VIP Call Girl At Th...VIP Service Call Girls Sindhi Colony 📳 7877925207 For 18+ VIP Call Girl At Th...
VIP Service Call Girls Sindhi Colony 📳 7877925207 For 18+ VIP Call Girl At Th...jageshsingh5554
 
Russian Escorts Girls Nehru Place ZINATHI 🔝9711199012 ☪ 24/7 Call Girls Delhi
Russian Escorts Girls  Nehru Place ZINATHI 🔝9711199012 ☪ 24/7 Call Girls DelhiRussian Escorts Girls  Nehru Place ZINATHI 🔝9711199012 ☪ 24/7 Call Girls Delhi
Russian Escorts Girls Nehru Place ZINATHI 🔝9711199012 ☪ 24/7 Call Girls DelhiAlinaDevecerski
 
Book Paid Powai Call Girls Mumbai 𖠋 9930245274 𖠋Low Budget Full Independent H...
Book Paid Powai Call Girls Mumbai 𖠋 9930245274 𖠋Low Budget Full Independent H...Book Paid Powai Call Girls Mumbai 𖠋 9930245274 𖠋Low Budget Full Independent H...
Book Paid Powai Call Girls Mumbai 𖠋 9930245274 𖠋Low Budget Full Independent H...Call Girls in Nagpur High Profile
 
Call Girls Service Jaipur Grishma WhatsApp ❤8445551418 VIP Call Girls Jaipur
Call Girls Service Jaipur Grishma WhatsApp ❤8445551418 VIP Call Girls JaipurCall Girls Service Jaipur Grishma WhatsApp ❤8445551418 VIP Call Girls Jaipur
Call Girls Service Jaipur Grishma WhatsApp ❤8445551418 VIP Call Girls Jaipurparulsinha
 
CALL ON ➥9907093804 🔝 Call Girls Baramati ( Pune) Girls Service
CALL ON ➥9907093804 🔝 Call Girls Baramati ( Pune)  Girls ServiceCALL ON ➥9907093804 🔝 Call Girls Baramati ( Pune)  Girls Service
CALL ON ➥9907093804 🔝 Call Girls Baramati ( Pune) Girls ServiceMiss joya
 

Recently uploaded (20)

Aspirin presentation slides by Dr. Rewas Ali
Aspirin presentation slides by Dr. Rewas AliAspirin presentation slides by Dr. Rewas Ali
Aspirin presentation slides by Dr. Rewas Ali
 
Artifacts in Nuclear Medicine with Identifying and resolving artifacts.
Artifacts in Nuclear Medicine with Identifying and resolving artifacts.Artifacts in Nuclear Medicine with Identifying and resolving artifacts.
Artifacts in Nuclear Medicine with Identifying and resolving artifacts.
 
Bangalore Call Girls Hebbal Kempapura Number 7001035870 Meetin With Bangalor...
Bangalore Call Girls Hebbal Kempapura Number 7001035870  Meetin With Bangalor...Bangalore Call Girls Hebbal Kempapura Number 7001035870  Meetin With Bangalor...
Bangalore Call Girls Hebbal Kempapura Number 7001035870 Meetin With Bangalor...
 
Call Girls Service Pune Vaishnavi 9907093804 Short 1500 Night 6000 Best call ...
Call Girls Service Pune Vaishnavi 9907093804 Short 1500 Night 6000 Best call ...Call Girls Service Pune Vaishnavi 9907093804 Short 1500 Night 6000 Best call ...
Call Girls Service Pune Vaishnavi 9907093804 Short 1500 Night 6000 Best call ...
 
Bangalore Call Girls Marathahalli 📞 9907093804 High Profile Service 100% Safe
Bangalore Call Girls Marathahalli 📞 9907093804 High Profile Service 100% SafeBangalore Call Girls Marathahalli 📞 9907093804 High Profile Service 100% Safe
Bangalore Call Girls Marathahalli 📞 9907093804 High Profile Service 100% Safe
 
Russian Call Girls in Pune Tanvi 9907093804 Short 1500 Night 6000 Best call g...
Russian Call Girls in Pune Tanvi 9907093804 Short 1500 Night 6000 Best call g...Russian Call Girls in Pune Tanvi 9907093804 Short 1500 Night 6000 Best call g...
Russian Call Girls in Pune Tanvi 9907093804 Short 1500 Night 6000 Best call g...
 
Call Girls Cuttack Just Call 9907093804 Top Class Call Girl Service Available
Call Girls Cuttack Just Call 9907093804 Top Class Call Girl Service AvailableCall Girls Cuttack Just Call 9907093804 Top Class Call Girl Service Available
Call Girls Cuttack Just Call 9907093804 Top Class Call Girl Service Available
 
Vip Call Girls Anna Salai Chennai 👉 8250192130 ❣️💯 Top Class Girls Available
Vip Call Girls Anna Salai Chennai 👉 8250192130 ❣️💯 Top Class Girls AvailableVip Call Girls Anna Salai Chennai 👉 8250192130 ❣️💯 Top Class Girls Available
Vip Call Girls Anna Salai Chennai 👉 8250192130 ❣️💯 Top Class Girls Available
 
VIP Call Girls Pune Vani 9907093804 Short 1500 Night 6000 Best call girls Ser...
VIP Call Girls Pune Vani 9907093804 Short 1500 Night 6000 Best call girls Ser...VIP Call Girls Pune Vani 9907093804 Short 1500 Night 6000 Best call girls Ser...
VIP Call Girls Pune Vani 9907093804 Short 1500 Night 6000 Best call girls Ser...
 
VIP Call Girls Indore Kirti 💚😋 9256729539 🚀 Indore Escorts
VIP Call Girls Indore Kirti 💚😋  9256729539 🚀 Indore EscortsVIP Call Girls Indore Kirti 💚😋  9256729539 🚀 Indore Escorts
VIP Call Girls Indore Kirti 💚😋 9256729539 🚀 Indore Escorts
 
Call Girls Service Navi Mumbai Samaira 8617697112 Independent Escort Service ...
Call Girls Service Navi Mumbai Samaira 8617697112 Independent Escort Service ...Call Girls Service Navi Mumbai Samaira 8617697112 Independent Escort Service ...
Call Girls Service Navi Mumbai Samaira 8617697112 Independent Escort Service ...
 
Bangalore Call Girls Majestic 📞 9907093804 High Profile Service 100% Safe
Bangalore Call Girls Majestic 📞 9907093804 High Profile Service 100% SafeBangalore Call Girls Majestic 📞 9907093804 High Profile Service 100% Safe
Bangalore Call Girls Majestic 📞 9907093804 High Profile Service 100% Safe
 
(👑VVIP ISHAAN ) Russian Call Girls Service Navi Mumbai🖕9920874524🖕Independent...
(👑VVIP ISHAAN ) Russian Call Girls Service Navi Mumbai🖕9920874524🖕Independent...(👑VVIP ISHAAN ) Russian Call Girls Service Navi Mumbai🖕9920874524🖕Independent...
(👑VVIP ISHAAN ) Russian Call Girls Service Navi Mumbai🖕9920874524🖕Independent...
 
Call Girl Number in Panvel Mumbai📲 9833363713 💞 Full Night Enjoy
Call Girl Number in Panvel Mumbai📲 9833363713 💞 Full Night EnjoyCall Girl Number in Panvel Mumbai📲 9833363713 💞 Full Night Enjoy
Call Girl Number in Panvel Mumbai📲 9833363713 💞 Full Night Enjoy
 
VIP Service Call Girls Sindhi Colony 📳 7877925207 For 18+ VIP Call Girl At Th...
VIP Service Call Girls Sindhi Colony 📳 7877925207 For 18+ VIP Call Girl At Th...VIP Service Call Girls Sindhi Colony 📳 7877925207 For 18+ VIP Call Girl At Th...
VIP Service Call Girls Sindhi Colony 📳 7877925207 For 18+ VIP Call Girl At Th...
 
Russian Escorts Girls Nehru Place ZINATHI 🔝9711199012 ☪ 24/7 Call Girls Delhi
Russian Escorts Girls  Nehru Place ZINATHI 🔝9711199012 ☪ 24/7 Call Girls DelhiRussian Escorts Girls  Nehru Place ZINATHI 🔝9711199012 ☪ 24/7 Call Girls Delhi
Russian Escorts Girls Nehru Place ZINATHI 🔝9711199012 ☪ 24/7 Call Girls Delhi
 
Book Paid Powai Call Girls Mumbai 𖠋 9930245274 𖠋Low Budget Full Independent H...
Book Paid Powai Call Girls Mumbai 𖠋 9930245274 𖠋Low Budget Full Independent H...Book Paid Powai Call Girls Mumbai 𖠋 9930245274 𖠋Low Budget Full Independent H...
Book Paid Powai Call Girls Mumbai 𖠋 9930245274 𖠋Low Budget Full Independent H...
 
Call Girls Service Jaipur Grishma WhatsApp ❤8445551418 VIP Call Girls Jaipur
Call Girls Service Jaipur Grishma WhatsApp ❤8445551418 VIP Call Girls JaipurCall Girls Service Jaipur Grishma WhatsApp ❤8445551418 VIP Call Girls Jaipur
Call Girls Service Jaipur Grishma WhatsApp ❤8445551418 VIP Call Girls Jaipur
 
CALL ON ➥9907093804 🔝 Call Girls Baramati ( Pune) Girls Service
CALL ON ➥9907093804 🔝 Call Girls Baramati ( Pune)  Girls ServiceCALL ON ➥9907093804 🔝 Call Girls Baramati ( Pune)  Girls Service
CALL ON ➥9907093804 🔝 Call Girls Baramati ( Pune) Girls Service
 
Russian Call Girls in Delhi Tanvi ➡️ 9711199012 💋📞 Independent Escort Service...
Russian Call Girls in Delhi Tanvi ➡️ 9711199012 💋📞 Independent Escort Service...Russian Call Girls in Delhi Tanvi ➡️ 9711199012 💋📞 Independent Escort Service...
Russian Call Girls in Delhi Tanvi ➡️ 9711199012 💋📞 Independent Escort Service...
 

MedicalResearch.com: Medical Research Exclusive Interviews Jan 23 2015

  • 1. MedicalResearch.com Exclusive Interviews with Medical Research and Health Care Researchers from Major and Specialty Medical Research Journals and Meetings Editor: Marie Benz, MD info@medicalresearch.com January 22 2015 For Informational Purposes Only: Not for Specific Medical Advice.
  • 2. Medical Disclaimer | Terms and Conditions • The contents of the MedicalResearch.com Site, such as text, graphics, images, and other material contained on the MedicalResearch.com Site ("Content") are for informational purposes only. The Content is not intended to be a substitute for professional medical advice, diagnosis, or treatment. Always seek the advice of your physician or other qualified health provider with any questions you may have regarding a medical condition. Never disregard professional medical advice or delay in seeking it because of something you have read on the Hemodialysis.com Site! • If you think you may have a medical emergency, call your doctor or 911 immediately. MedicalResearch.com does not recommend or endorse any specific tests, physicians, products, procedures, opinions, or other information that may be mentioned on the Site. Reliance on any information provided by MedicalResearch.com or other Eminent Domains Inc (EDI) websites, EDI employees, others appearing on the Site at the invitation of MedicalResearch.com or EDI, or other visitors to the Site is solely at your own risk. • The Site may contain health- or medical-related materials that are sexually explicit. If you find these materials offensive, you may not want to use our Site. The Site and the Content are provided on an "as is" basis. Read more interviews on MedicalResearch.com
  • 3. Text Messaging Can Boost Medical Research Clinical Trial Recruitment MedicalResearch.com Interview with: Moe Alsumidaie MBA MSF President & Chief Scientific Officer Annex Clinical • MedicalResearch: What is the background for this study? • Response: SUMMARY: • A real-world case study measuring the impact of Short Messaging System (SMS) or “Text Messaging” on clinical trial patient recruitment using an interactive two-way patient engagement platform by Mosio, Inc., which provides clinical research services designed to increase patient recruitment, engagement and retention, found that use of text messaging alone can be an effective means of patient engagement that results in clinical trial patient enrollment. • Patient recruitment, retention and medication adherence continue to be challenges in conducting effective clinical trials. While clinical trials often rely on email recruitment, recent studies suggest that only 22% of emails are read.1 Alternatively, 98% of text messages are read1 and 90% of text messages are read within the first three minutes of receipt.2 • Recent research has evaluated the impact of Short Messaging System (SMS) or “Text Messaging” in healthcare settings, such as appointment reminders and medication adherence. Results have demonstrated that SMS intervention significantly improved patient behavioral outcomes: patients who received SMS reminders were more likely to show up to appointments on time,3 and patients who received SMS reminders were more adherent to medications.4 However, only limited research is available on the effect of SMS on clinical trial subject enrollment. • Johnson County Clin-Trials (JCCT), a clinical research facility that specializes in executing 10-15 vaccine clinical trials per year, was facing issues with enrolling patients rapidly in a tight time frame using email. To access a more effective strategy to better engage patients, JCCT employed two-way SMS/text messaging solutions, and this study assessed the impact of SMS/text messaging on patient recruitment and enrollment. Read the rest of the interviews on MedicalResearch.com Content NOT an endorsement of efficacy and NOT intended as specific medical advice.
  • 4. Text Messaging Can Boost Medical Research Clinical Trial Recruitment MedicalResearch.com Interview with: Moe Alsumidaie MBA MSF President & Chief Scientific Officer Annex Clinical • MedicalResearch: What are the main findings? • Response: Overall, SMS/text messaging alone produced an immediate response that resulted in more optimized clinical trial enrollment outcomes in comparison to JCCT’s previous experiences using email. JCCT remarked that the response was five times the response compared to e-mail and that enrollment results exceeded the trial sponsor’s expectations. • Use of two-way SMS/text messaging achieved a 1% increase in research subjects enrolled for every 1.5% increase in text messages sent. Over an 8-week enrollment period, a total of 1,541 text messages sent resulted in screening of 795 patients and enrollment of 265 patients. Read the rest of the interviews on MedicalResearch.com Content NOT an endorsement of efficacy and NOT intended as specific medical advice.
  • 5. Text Messaging Can Boost Medical Research Clinical Trial Recruitment MedicalResearch.com Interview with: Moe Alsumidaie MBA MSF President & Chief Scientific Officer Annex Clinical • MedicalResearch: What should clinicians and patients take away from your report? • Response: This case study suggests that two-way text messaging through a strategic and interactive patient engagement platform is an effective tool to captivate potential study participants by reaching them on the mobile devices they carry. JCCT found that patients responded positively to receiving SMS/Text Messages. • In this case, we have found that two-way SMS/Text Messaging solutions enable researchers to enroll patients in clinical trials. It is important to emphasize that IT systems utilization in clinical trials require validated systems that are HIPAA and 21 CFR Part 11 compliant; the two-way SMS system we used was validated for clinical trial use. • It is also important to note that FCC regulations require that any SMS/text message recipient opt in to receiving SMS from the deliverer. Many study sites have patient email lists; I recommend that study sites consider building their own SMS/text message database, so that they can send mobile communications to patients and comply with FCC regulations. Taking the effort now can save study sites time in the long run, while boosting enrollment performance, and available technologies can facilitate the process. • For example, the Mosio platform used by JCCT in the case study includes technology and tools to assist with developing communications content that can convert patients to opt in to receiving SMS/text messages, such as opt-in web widgets, opt-in wording templates in consent forms, and opt-in email blast templates to send out to existing patient databases. • Further, I want to indicate that many patients tend to respond to SMS/text messages by texting back with questions and more information. Mosio’s technological platform is capable of having interactive and automated conversations with patients, which can be used to qualify patients while enhancing patient engagement and reducing the work burden on study sites. Read the rest of the interviews on MedicalResearch.com Content NOT an endorsement of efficacy and NOT intended as specific medical advice.
  • 6. Text Messaging Can Boost Medical Research Clinical Trial Recruitment MedicalResearch.com Interview with: Moe Alsumidaie MBA MSF President & Chief Scientific Officer Annex Clinical • MedicalResearch: What recommendations do you have for future research as a result of this study? • Response: This real-world case study, conducted during an actual vaccine clinical trial, strongly suggests that a bilateral two-way SMS/text messaging platform can boost trial recruitment, with implications for cost savings, study timing and quality of results. Future prospective studies should compare SMS/text messaging with other common recruiting approaches via random subject assignment to groups to quantify levels of response with each approach and its resulting effectiveness and cost efficiency. • New approaches will be key in meeting trial retention, compliance and adherence challenges. In rigorous investigational product trials, non-adherence rates of 20%-30% require a 50% increase in sample size to maintain equivalent statistical power, while a 50% non-adherence rate requires a 200% increase in sample size.5 • Conducting additional studies to evaluate the impact of SMS/Text Messages on medication adherence and other areas such as patient retention, study compliance, appointment reminders, and patient reported outcomes data collection will bring us closer to finding useful and cost effective solutions in clinical trials. Read the rest of the interviews on MedicalResearch.com Content NOT an endorsement of efficacy and NOT intended as specific medical advice.
  • 7. Text Messaging Can Boost Medical Research Clinical Trial Recruitment MedicalResearch.com Interview with: Moe Alsumidaie MBA MSF President & Chief Scientific Officer Annex Clinical • References: • Frost & Sullivan 2010, Epsilon 2009 • http://www.tatango.com/blog/90-of-text-messages-are-read-within-3-minutes/ • Use of mobile telephone short message service as a reminder: the effect on patient attendance, Sumanth Prasad and Richa Anand, International Dental Journal 2012; 62: 21-26 • Sahar Khonsari, et al., Effect of a reminder system using an automated short messaging service on medication adherence following acute coronary syndrome. Eur J Cardiovasc Nurs. published 02, February 2014 • Smith, Dorothy PharmD, Patient Nonadherencein Clinical Trials: Could There Be a Link to Post Marketing Patient Safety?, Consumer Health Information Corporation, October2011 • Citation: • Text Messaging Enhances Clinical Trial Enrollment with Mobile Solutions from Mosio Applied Clinical Trials • Publish date: Nov 3, 2014 Moe Alsumidaie Read the rest of the interviews on MedicalResearch.com Content NOT an endorsement of efficacy and NOT intended as specific medical advice.
  • 8. Endocrine Society Guidelines For Weight Loss Medications in Obesity MedicalResearch.com interview with: Caroline M. Apovian, MD Chair of the Endocrine Society task force that developed “Pharmacological Management of Obesity: An Endocrine Society Clinical Practice Guideline” Boston University School of Medicine Boston Medical Center • MedicalResearch: What is the background for this report? • Dr. Apovian: The Food and Drug Administration has approved four new anti-obesity drugs – lorcaserin, phentermine/topiramate, naltrexone/bupropion and liraglutide – in the past two years. To help clinicians navigate this changing landscape, the Endocrine Society developed its Clinical Practice Guideline to provide strategies for prescribing drugs to manage obesity and promote weight loss. • MedicalResearch: What are the main findings? • Dr. Apovian: In the Clinical Practice Guideline, the Endocrine Society recommends that diet, exercise and behavioral modifications be part of all obesity management approaches. Other tools such as weight loss medications and bariatric surgery can be combined with behavioral changes to reduce food intake and increase physical activity, in appropriate patients. Patients who have been unable to successfully lose weight and maintain a goal weight may be candidates for prescription medication if they meet the criteria on the drug’s label as well as BMI criteria (BMI greater than or equal to 30 or greater than or equal to 27 with at least one comorbidity). Read the rest of the interviews on MedicalResearch.com Content NOT an endorsement of efficacy and NOT intended as specific medical advice.
  • 9. Endocrine Society Guidelines For Weight Loss Medications in Obesity MedicalResearch.com interview with: Caroline M. Apovian, MD Chair of the Endocrine Society task force that developed “Pharmacological Management of Obesity: An Endocrine Society Clinical Practice Guideline” Boston University School of Medicine Boston Medical Center • Other recommendations from the CPG include: • If a patient responds well to a weight loss medication and loses 5 percent or more of their body weight after three months, the medication can be continued. If the medication is ineffective or the patient experiences side effects, the prescription should be stopped and an alternative medication or approach considered. • Since some diabetes medications are associated with weight gain, people with diabetes who are obese or overweight should be given medications that promote weight loss or have no effect on weight as first- and second-line treatments. Doctors should discuss medications’ potential effects on weight with patients. • Certain types of medication – angiotensin converting enzyme inhibitors, angiotensin receptor blockers and calcium channel blockers – should be used as a first-line treatment for high blood pressure in obese people. These are effective blood pressure treatments that are less likely to contribute to weight gain than an alternative medication, beta-adrenergic blockers. • When patients need medications that can have an impact on weight such as antidepressants, antipsychotic drugs and medications for treating epilepsy, they should be fully informed and provided with estimates of each option’s anticipated effect on weight. Doctors and patients should engage in a shared-decision making process to evaluate the options. • In patients with uncontrolled high blood pressure or a history of heart disease, the medications phentermine and diethylpropion should not be used. Read the rest of the interviews on MedicalResearch.com Content NOT an endorsement of efficacy and NOT intended as specific medical advice.
  • 10. Endocrine Society Guidelines For Weight Loss Medications in Obesity MedicalResearch.com interview with: Caroline M. Apovian, MD Chair of the Endocrine Society task force that developed “Pharmacological Management of Obesity: An Endocrine Society Clinical Practice Guideline” Boston University School of Medicine Boston Medical Center • MedicalResearch: What should clinicians and patients take away from your report? • Dr. Apovian: Lifestyle changes should always be a central part of any weight loss strategy. Medications do not work by themselves, but they can help people maintain a healthy diet by reducing the appetite. Adding a medication to a lifestyle modification program is likely to result in greater weight loss. • Citation: • Pharmacological Management of Obesity: An Endocrine Society Clinical Practice Guideline • Caroline M. Apovian, Louis J. Aronne, Daniel H. Bessesen, Marie E. McDonnell, M. Hassan Murad, Uberto Pagotto, Donna H. Ryan, and Christopher D. Still • DOI: http://dx.doi.org/10.1210/jc.2014-3415 Received: September 03, 2014 Accepted: December 08, 2014 Published Online: January 15, 2015 Read the rest of the interviews on MedicalResearch.com Content NOT an endorsement of efficacy and NOT intended as specific medical advice.
  • 11. ICD-9 Codes Do Not Identify All Strokes in Atrial Fibrillation Patients MedicalResearch.com Interview with: Jonathan Thigpen, PharmD Assistant Professor Clinical and Administrative Sciences Notre Dame of Maryland University School of Pharmacy • Medical Research: What is the background for this study? What are the main findings? Dr. Thigpen: This effort assessed the accuracy of International Classification of Disease 9th Edition (ICD-9) stroke codes in identifying valid stroke events in a cohort of atrial fibrillation (AF) patients. The initial electronic search yielded 1,812 events across three stroke centers (Boston Medical Center, Geisinger Health System, and University of Alabama). All ICD-9 identified stroke events were vetted through manual chart review with final adjudication by a stroke neurologist. Atrial fibrillation was verified by evidence via electrocardiogram at stroke admission, 6 months prior to, or 90 days after stroke admission. • In addition to assessing the accuracy of the stroke codes alone, we also assessed the accuracy of stroke and Atrial fibrillation codes combined as well as the accuracy of stroke codes when seeking for stroke associated with Atrial fibrillation. These additional steps give readers insight as to the accuracy and reliability of using ICD-9 codes alone to create a stroke plus AF cohort. We feel that this effort is extremely important given the increasing reliance on ICD-9 codes as a means of identifying stroke events and covariates in research, especially research using administrative data. The positive predictive value (PPV) of stroke codes alone was 94.2%. PPVs did not differ across clinical site or by type of event (ischemic vs. intracranial hemorrhage). PPV of stroke codes did differ by event coding position (primary vs. other; 97.2% vs. 83.7%) and by ischemic stroke code (433 vs. 434; 85.2% vs. 94.4%). When combined with validation of Atrial fibrillation codes, the PPV of stroke codes decreased to 82.2%. After excluding ischemic stroke due to a different mechanism (eg, vascular procedure, tumor, sepsis) the PPV dropped further to 72.8%. As a separate exercise, manual review confirmed 33 (7.2%) ischemic strokes in 458 events coded as “without infarction”. Read the rest of the interviews on MedicalResearch.com Content NOT an endorsement of efficacy and NOT intended as specific medical advice.
  • 12. ICD-9 Codes Do Not Identify All Strokes in Atrial Fibrillation Patients MedicalResearch.com Interview with: Jonathan Thigpen, PharmD Assistant Professor Clinical and Administrative Sciences Notre Dame of Maryland University School of Pharmacy • Medical Research: What should clinicians and patients take away from your report? • Dr. Thigpen: These results indicate that ICD-9 stroke codes alone have limited use in identifying acute strokes in the setting of active Atrial fibrillation. Manual verification of stroke is needed to confirm stroke events in the setting of AF so as to reduce potential bias. • Medical Research: What recommendations do you have for future research as a result of this study? • Dr. Thigpen: Future research could build on our study in the following ways. • Firstly, there is limited knowledge concerning the accuracy of ICD-10 stroke codes. Secondly, our study (and other previous literature) indicates several screening methods that tend to lead to higher coding accuracies (ie, only using stroke codes in the primary position). Further optimizing ICD-9 screening methods so as to limit the inaccuracies of stroke codes will potentially help better establish ICD stroke codes as a reliable tool for case and covariate ascertainment. • Citation: • Validity of International Classification of Disease Codes to Identify Ischemic Stroke and Intracranial Hemorrhage Among Individuals With Associated Diagnosis of Atrial Fibrillation • AHA publication:Jonathan L. Thigpen, Chrisly Dillon, Kristen B. Forster, Lori Henault, Emily K. Quinn, Yorghos Tripodis, Peter B. Berger, Elaine M. Hylek, and Nita A. Limdi • Circ Cardiovasc Qual Outcomes. 2015;CIRCOUTCOMES.113.000371published online before print January 13 2015, doi:10.1161/CIRCOUTCOMES.113.000371 Read the rest of the interviews on MedicalResearch.com Content NOT an endorsement of efficacy and NOT intended as specific medical advice.
  • 13. Walking Program Insufficient To Reduce Risk of Falls in Elderly MedicalResearch.com Interview with: Alexander Voukelatos BSc, BA, MA(psych), PhD Healthy Populations Program Manager Health Promotion Sydney Local Health District and Conjoint Lecturer School of Public Health and Community Medicine University of NSW • Medical Research: What is the background for this study? • Response: Falls in older people has been a significant public health issue in high income countries for several decades now. We know that if current trends continue, given that more people will be living for longer, falls will be an even bigger issue in the not too distant. Falls are not an inevitable part of ageing, and in fact many falls can be prevented relatively simply by increasing physical activity. • For over 15 years we’ve known that physical activity is one of the most effective ways of reducing the risk of falls in older people living in the community; since the publication of the first Cochrane review on Interventions for preventing falls in the elderly by Gillespie and colleagues. [1] I know that Health Departments here, in Australia, and in New Zealand – as I suspect has been the case in many high income countries – have invested a lot of resources over the past few decades into reducing falls-related hospital admissions in older people. Much of this going into promoting and funding physical activity programs for older people. • However, this investment has had very little if any impact on falls-related hospital admissions in older people. There may be several reasons we haven’t seen any difference in these rates. In New South Wales – Australia’s most populous state – we know that physical activity rates amongst older people have actually risen by about 15% between 1998 and 2005,[2] the most popular activity by far being walking,[3] yet we haven’t see any corresponding change in falls- related hospitalization rates. Perhaps there has not been enough time for these programs to have made an impact on hospitalization rates, or perhaps the change in physical activity levels is insufficient to make an impact on these rates. Read the rest of the interviews on MedicalResearch.com Content NOT an endorsement of efficacy and NOT intended as specific medical advice.
  • 14. Walking Program Insufficient To Reduce Risk of Falls in Elderly MedicalResearch.com Interview with: Alexander Voukelatos BSc, BA, MA(psych), PhD Healthy Populations Program Manager Health Promotion Sydney Local Health District and Conjoint Lecturer School of Public Health and Community Medicine University of NSW • Another possibility could be that while we’ve seen an increase in physical activity in older people perhaps its not the kind of physical activity that results in a reduction in falls. Sherrington and colleagues [4] reviewed effective physical activity interventions for preventing falls in older people and found they had several elements in common: a) the physical activity included balance challenging exercise i.e. exercises taking participants to the limits of their stability, b) at least 50 hours of accumulated activity was needed, and c) no walking was included in the exercises. So we know not all types of physical activity will be equally effective in reducing the risk of falling. There is some disagreement in the literature about walking. • There are several studies that included walking as part of the intervention and showed a reduction in falls in older people. Other studies supported the conclusions made by Sherrington that walking is not associated with a reduction in falls. All of these studies included walking as a component of an intervention which makes it difficult to figure out what effect walking specifically has on falls rates. • This is were our study comes in. We wanted to investigate the effectiveness of a walking program on falls in older people, specifically sedentary older people, who we presumed would get the most benefit from becoming more physically active. • We developed a walking program specifically for sedentary older people, that they could do themselves in their own time, at their preferred locale. The aim of the program was to get participants walking for at least 150 minutes per week at a brisk pace. The program comprised of four parts: the first part focused on increasing the frequency of walks, the second part focused on getting participants to walk for at least 150 minutes per week, followed by walking for 150 minutes at a brisk pace, while the final part focused on supporting participants in maintaining their walking levels and incorporating walking as part of their daily activities. Read the rest of the interviews on MedicalResearch.com Content NOT an endorsement of efficacy and NOT intended as specific medical advice.
  • 15. Walking Program Insufficient To Reduce Risk of Falls in Elderly MedicalResearch.com Interview with: Alexander Voukelatos BSc, BA, MA(psych), PhD Healthy Populations Program Manager Health Promotion Sydney Local Health District and Conjoint Lecturer School of Public Health and Community Medicine University of NSW • Medical Research: What are the main findings? • Response: Overall, we found no evidence that walking is associated with a reduction (or increase) in falls rates. The walking program did however increase participants’ levels of general physical activity as well as walking levels. However, on closer inspection there were some interesting trends. Note that by trends I mean there was no statistical support for the following conclusions but there was some consistency in the data that suggests there maybe something going on (something for future research). The data indicated that for participants aged 65-74 years walking had non- significant decrease in falls, whereas for participants aged 75 years or more walking had a non-significant increase in the the risk of falls. The key words here are non-significant. This study was not powered to examine the data stratified by age group. However these trends may be explained if we consider age-group a proxy for frailty (of course this is not conclusive as there will be some 65-74 year olds who are frail, and indeed some people over 75 years who are not frail) For the more frail participants (i.e. 75+ year olds) walking actually increased their risk of falls by exposing them to environmental falls risk factors – eg. uneven surfaces, slippery paths – which they were too frail to negotiate. On the other hand the less frail participants (65-74 year olds) even though they were exposed to more environmental falls hazards they were able to negotiate the hazards and in time reduce their risk of falling. These two opposing effects may have cancelled each other out thus resulting in the null effect for the overall study. This is purely speculative however. Read the rest of the interviews on MedicalResearch.com Content NOT an endorsement of efficacy and NOT intended as specific medical advice.
  • 16. Walking Program Insufficient To Reduce Risk of Falls in Elderly MedicalResearch.com Interview with: Alexander Voukelatos BSc, BA, MA(psych), PhD Healthy Populations Program Manager Health Promotion Sydney Local Health District and Conjoint Lecturer School of Public Health and Community Medicine University of NSW • Medical Research: What should clinicians and patients take away from your report? • Response: The take away message from this study is that walking is insufficient activity to reduce the risk of falls. If a clinician or anyone else is concerned about someone’s risk of falling walking is probably not going to do much to address this. • However, walking is still a highly effective and very important activity for addressing many chronic diseases in older people. And walking can still play an important role in reducing falls as a way of increasing activity levels for sedentary older people who, once they become more active, can then participate in proven falls prevention activities such as tai chi. • I would also recommend that clinicians use their judgment in prescribing walking for people over 75 years of age. Read the rest of the interviews on MedicalResearch.com Content NOT an endorsement of efficacy and NOT intended as specific medical advice.
  • 17. Walking Program Insufficient To Reduce Risk of Falls in Elderly MedicalResearch.com Interview with: Alexander Voukelatos BSc, BA, MA(psych), PhD Healthy Populations Program Manager Health Promotion Sydney Local Health District and Conjoint Lecturer School of Public Health and Community Medicine University of NSW • Medical Research: What recommendations do you have for future research as a result of this study? • Response: This study shows that the relationship between walking and falls is not straight forward. Future studies might consider planning a stratified analysis based on age-groups, or level of frailty to see if age is a modifying variable. Also future studies could use different types of walking such as obstacle walking or hiking/bushwalking which present many falls hazards to walker – the idea being that if the walker can learn to safely negotiate these high- level falls hazards then that will confer a beneficial effect in negotiating more moderate falls hazards typically found in their local environment. • [1] Gillespie LD, Gillespie WJ, Cumming R, Lamb S, Rowe BH. Interventions for preventing falls in the elderly (Cochrane Review). The Cochrane Library 1997, Issue 4. [2] Chau J, Smith B, Chey T, Merom D, Bauman A. Trends in population levels of sufficient physical activity in NSW, 1998 to 2005: Summary report. Sydney, NSW: NSW Centre for Physical Activity and Health (CPAH); January 2007. Report No. CPAH06-001b [3] Merom D, Pye V, Macniven R, Van der Ploeg H, Milat A, Sherrington C, Lord S, Bauman A. Prevalence and correlates of participation in fall prevention exercise/physical activity by older adults. Prev Med 2012; 55(5): 613-7. [4] Sherrington C, Tiedemann A, Fairhall N, Close JCT, Lord SR. Exercise to prevent falls in older adults: an updated meta-analysis and best practice recommendations. NSW Public Health Bull 2011, 22 (3-4): 78-83. Read the rest of the interviews on MedicalResearch.com Content NOT an endorsement of efficacy and NOT intended as specific medical advice.
  • 18. Walking Program Insufficient To Reduce Risk of Falls in Elderly MedicalResearch.com Interview with: Alexander Voukelatos BSc, BA, MA(psych), PhD Healthy Populations Program Manager Health Promotion Sydney Local Health District and Conjoint Lecturer School of Public Health and Community Medicine University of NSW • Citation: • The impact of a home-based walking programme on falls in older people: the Easy Steps randomised controlled trial Alexander Voukelatos, Dafna Merom, Catherine Sherrington, Chris Rissel, Robert G. Cumming, and Stephen R. Lord Age Ageing first published online January 8, 2015 doi:10.1093/ageing/afu186 Read the rest of the interviews on MedicalResearch.com Content NOT an endorsement of efficacy and NOT intended as specific medical advice.
  • 19. Eczema and Atopic Dermatitis Linked To Increased Cardiovascular Risk MedicalResearch.com Interview with: Dr. Jonathan L. Silverberg MD PhD MPH Assistant Professor in Dermatology, Medical Social Sciences and Preventive Medicine Northwestern University, Chicago, Illinois • Medical Research: What is the background for this study? What are the main findings? Dr. Silverberg: There is a growing body of literature supporting an association between psoriasis and increased cardiovascular risk. We hypothesized that these associations are not specific to psoriasis. Rather, they likely occur in other chronic inflammatory skin disorders, namely eczema. We studied two large-scale US population-based studies and found that adults with eczema were more likely to smoke cigarettes, drink alcohol and were less physically active. In turn, they also have higher rates of obesity, high blood pressure, prediabetes and type 2 diabetes and high cholesterol. Of note, eczema was associated with these disorders even after controlling for smoking, alcohol consumption and physical activity. This suggests that chronic inflammation and/or other factors related to eczema may also drive increased cardiovascular risk. • Medical Research: What should clinicians and patients take away from your report? • Dr. Silverberg: There are several take home points: • First, smoking, alcohol and physical activity are all modifiable cardiovascular risk factors. Adults with eczema should be screened for these behaviors and clinical interventions should be employed early and often to reduce their excess cardiovascular risk. Second, better systemic treatments are needed to address the systemic inflammation of moderate to severe eczema. Clinicians should avoid using systemic steroids, which predictably increase weight gain, blood pressure and blood sugar. Read the rest of the interviews on MedicalResearch.com Content NOT an endorsement of efficacy and NOT intended as specific medical advice.
  • 20. Eczema and Atopic Dermatitis Linked To Increased Cardiovascular Risk MedicalResearch.com Interview with: Dr. Jonathan L. Silverberg MD PhD MPH Assistant Professor in Dermatology, Medical Social Sciences and Preventive Medicine Northwestern University, Chicago, Illinois • Medical Research: What recommendations do you have for future research as a result of this study? • Dr. Silverberg: Future studies are needed to identify the mechanisms of increased cardiovascular risk in atopic dermatitis and the most effective clinical approaches to reducing this risk. Further, more research is needed for the development of safer and more effective systemic treatments of atopic dermatitis. • Citation: • Eczema and cardiovascular risk factors in 2 US adult population studies • Silverberg JI, Greenland P J Allergy Clin Immunol. 2015 Jan 8. pii: S0091-6749(14)01677-7. doi: 10.1016/j.jaci.2014.11.023. [Epub ahead of print] Read the rest of the interviews on MedicalResearch.com Content NOT an endorsement of efficacy and NOT intended as specific medical advice.
  • 21. Expedited Partner Therapy May Decrease Population Levels of Gonorrhea and Chlamydia MedicalResearch.com Interview with: Matthew Golden MD, MPH Director, PHSKC HIV/STD Program Professor of Medicine, University of Washington Harborview Medical Center • Medical Research: What is the background for this study? What are the main findings? Dr. Golden: Gonorrhea and chlamydial infection are the most common reportable infections in the United States and, in women, are associated with pelvic inflammatory disease, ectopic pregnancy, infertility and chronic pelvic pain. One way to decrease the number of cases of gonorrhea and chlamydia is to increase our success in treating the sex partners of persons diagnosed with these infections. Expedited partner therapy (EPT) – treating partners without requiring them to first undergo a medical evaluation – is one way to increase partner treatment. This usually involves giving people medication to give to their partners. Prior randomized trials have found that EPT decreases patients’ risk of becoming reinfected. • We conducted a community-level randomized trial to evaluate whether making free Expedited partner therapy available to medical providers would increase the use of Expedited Partner Therapy and decrease gonorrhea and chlamydial infections at the population level. We found that a public health program that made Expedited partner therapy widely available could dramatically increase medical providers use of EPT. Although our final result was not statistically significant, our findings suggest that the program likely decreased both gonorrhea and chlamydial infection by about 10% at the population-level. Read the rest of the interviews on MedicalResearch.com Content NOT an endorsement of efficacy and NOT intended as specific medical advice.
  • 22. Expedited Partner Therapy May Decrease Population Levels of Gonorrhea and Chlamydia MedicalResearch.com Interview with: Matthew Golden MD, MPH Director, PHSKC HIV/STD Program Professor of Medicine, University of Washington Harborview Medical Center • Medical Research: What should clinicians and patients take away from your report? • Dr. Golden: Expedited partner therapy, which was previously shown to be an effective intervention in increasing partner treatment and decreasing patients’ risk of reinfection, can also have a population-level effect. • Medical Research: What recommendations do you have for future research as a result of this study? • Dr. Golden: Health departments need to develop and test ways to better promote Expedited partner therapy use. • Citation: • Uptake and Population-Level Impact of Expedited Partner Therapy (EPT) on Chlamydia trachomatis And Neisseria gonorrhoeae : The Washington State Community-Level Randomized Trial of EPT. • Matthew R. Golden et al Published: January 15, 2015 DOI: 10.1371/journal.pmed.1001777 Read the rest of the interviews on MedicalResearch.com Content NOT an endorsement of efficacy and NOT intended as specific medical advice.
  • 23. Mortality Risks High In Elderly Patients After Carotid Artery Stenting MedicalResearch.com Interview with: Soko Setoguchi, MD DrPH Assistant Professor of Medicine Harvard Medical School and Harvard School of Public Health Director of Safety and Outcome Research in Cardiology Associate Physician in the Division of Pharmacoepidemiology and Pharmacoeconomics Brigham and Women’s Hospital • Medical Research: What is the background for this study? What are the main findings? Dr. Setoguchi: Medicare made a decision to cover Carotid Artery Stenting (CAS) in 2005 after publication of SAPPHIRE, which demonstrated the efficacy of Carotid Artery Stenting (CAS) vs Carotid endarterectomy (CEA) in high risk patients for CEA. Despite the data showing increased carotid artery stenting dissemination following the 2005 National Coverage Determination, peri-procedural and long-term outcomes have not been described among Medicare beneficiaries, who are quite different from trial patients, older and with more comorbidities in general population. • Understanding the outcomes in these population is particularly important in the light of more recent study, the Carotid Revascularization Endarterectomy versus Stenting Trial (CREST), which established CAS as a safe and efficacious alternative to CEA among non-high-surgical risk patients that also expanded the clinical indication of carotid artery stenting. • Another motivation to study ‘real world outcomes in the general population is expected differences in the proficiency of physicians peforming stenting in trial setting vs. real world practice setting. SAPPHIRE and CREST physicians were enrolled only after having demonstrated CAS proficiency with low complication rates whereas hands-on experience and patient outcomes among real-world physicians and hospitals is likely to be more diverse. • We found that unadjusted mortality risks over study period of 5 years with a mean of 2 years of follow-up in our population was 32%. Much higher mortality risks observed among certain subgroups with older age, symptomatic patients and non-elective hospitalizations. Read the rest of the interviews on MedicalResearch.com Content NOT an endorsement of efficacy and NOT intended as specific medical advice.
  • 24. Mortality Risks High In Elderly Patients After Carotid Artery Stenting MedicalResearch.com Interview with: Soko Setoguchi, MD DrPH Assistant Professor of Medicine Harvard Medical School and Harvard School of Public Health Director of Safety and Outcome Research in Cardiology Associate Physician in the Division of Pharmacoepidemiology and Pharmacoeconomics Brigham and Women’s Hospital • Medical Research: What should clinicians and patients take away from your report? • Dr. Setoguchi: Mortality risks in older Medicare patients who underwent Carotid Artery Stenting were high. The benefit of Carotid Artery Stenting in older patients, especially those who are symptomatic from carotid stenosis and who may undergo urgent procedure, is likely diminished. Therefore, careful clinical assessment of risk benefit should guide the choice of therapy in older patients. • Factors such as patient’s age, symptomatic status and urgent hospitalization that we found associated with higher mortality can guide physicians and patient to make decision to consider/perform Carotid Artery Stenting. However, such guidance should not overwrite a decision based on careful clinical assessments about the prognosis of patient and patient preferences. Read the rest of the interviews on MedicalResearch.com Content NOT an endorsement of efficacy and NOT intended as specific medical advice.
  • 25. Mortality Risks High In Elderly Patients After Carotid Artery Stenting MedicalResearch.com Interview with: Soko Setoguchi, MD DrPH Assistant Professor of Medicine Harvard Medical School and Harvard School of Public Health Director of Safety and Outcome Research in Cardiology Associate Physician in the Division of Pharmacoepidemiology and Pharmacoeconomics Brigham and Women’s Hospital • Medical Research: What recommendations do you have for future research as a result of this study? • Dr. Setoguchi: Our study did not have comparable data to SAPPHIRE or CREST to directly apply the same proficiency criteria used in these studies. The modified criteria we applied likely underestimated the number of providers who would have been included SAPHIRE or CREST. However, our data demonstrated that the real-world providers are more diverse in their proficiency than those who were enrolled in these trials. More studies with high quality data are needed to carefully assess provider’s proficiency in real-world settings and provide guidance on the necessary level of proficiency for better outcomes after Carotid Artery Stenting. • Citation: • Jalbert JJ, Nguyen LL, Gerhard-Herman MD, et al. Outcomes After Carotid Artery Stenting in Medicare Beneficiaries, 2005 to 2009. JAMA Neurol. Published online January 12, 2015. doi:10.1001/jamaneurol.2014.3638. Read the rest of the interviews on MedicalResearch.com Content NOT an endorsement of efficacy and NOT intended as specific medical advice.
  • 26. Periodontitis Linked To Larger Myocardial Infarct Size MedicalResearch.com Interview with: Dr. Francisco Mesa Department of Periodontics, School of Dentistry, University of Granada, Spain • Medical Research: What is the background for this study? What are the main findings? • Dr. Mesa: The size of an acute myocardial infarct (AMI) is one of the determinants of its severity, i.e., the degree of myocardial necrosis. This necrosis is indicated by peak troponin I levels in the blood. Among the acute myocardial infarct patients in our study, mediated regression analysis demonstrated that troponin I levels were higher, i.e., the infarct size was larger, in those with chronic periodontitis. • Medical Research: What should clinicians and patients take away from your report? • Dr. Mesa: The primary message for dentists is that their contribution to the healthcare of patients is not limited to oral problems alone, adding value to their profession. Their knowledge and action can have important systemic repercussions, implying the need for maximum rigor in the diagnosis and treatment of chronic periodontitis. Around 75-80% of adult cases of chronic periodontitis can be treated and controlled by the general dentist. A key message for patients would be to maintain correct oral hygiene habits and pursue what could be called a “Cardio-Perio-Healthy Life Style”, given the large number of risk factors shared by cardiovascular and periodontal diseases. Read the rest of the interviews on MedicalResearch.com Content NOT an endorsement of efficacy and NOT intended as specific medical advice.
  • 27. Periodontitis Linked To Larger Myocardial Infarct Size MedicalResearch.com Interview with: Dr. Francisco Mesa Department of Periodontics, School of Dentistry, University of Granada, Spain • Medical Research: What recommendations do you have for future research as a result of this study? • Dr. Mesa: Periodontics has acquired a dimension that could not have been imagined a few years ago, when the loss of teeth was the sole complication of periodontal disease. It is now considered to be a risk factor for systemic diseases (e.g., AMI and low-weight or premature births) and to represent a major public health problem. This important research line in Periodontal Medicine will continue to contribute scientific evidence on the influence of periodontitis on different diseases. Further research is warranted on the inclusion of periodontitis in risk scores for AMI and other pathologies associated with endothelial dysfunction, such as erectile dysfunction or dementia. Cardiologists are increasingly aware of the relationship of chronic periodontitis with atherosclerosis and AMI. In 2012, the European Society of Cardiology introduced periodontitis into its cardiovascular disease prevention guidelines, and the American Heart Association has acknowledged its influence on cardiovascular risk, although it has called for more follow-up studies to elucidate the causal nature of this relationship. • Citation: • Marfil-Álvarez, F. Mesa, A. Arrebola-Moreno, J.A. Ramírez-Hernández, A. Magán-Fernández, F. O’Valle, P. Galindo-Moreno, and A. Catena • Acute Myocardial Infarct Size Is Related to Periodontitis Extent and Severity J DENT RES October 2014 93: 993-998, first published on August 19, 2014 doi:10.1177/002203451454822 Read the rest of the interviews on MedicalResearch.com Content NOT an endorsement of efficacy and NOT intended as specific medical advice.
  • 28. New Drug For Hyperkalemia Selectively Traps Potassium MedicalResearch.com Interview with: David K. Packham, M.B., B.S., M.D Royal Melbourne Hospital Melbourne Renal Research Group VIC 3073, Australia, • Medical Research: What is the background for this study? What are the main findings? Dr. Packham: ZS-9 represents a new mechanism of action for addressing hyperkalemia. Unlike traditional nonspecific organic polymer cationexchangers, ZS-9 is a non-absorbed, inorganic crystalline potassium-selective cation exchanger that traps excess potassium in the gastrointestinal tract. It has been evaluated in three prospective, randomized, double-blind, placebo-controlled studies with over 1100 patients to date, representing the largest ever clinical development program for hyperkalemia. • ZS-003 was the first of two pivotal Phase 3 studies that evaluated the safety and efficacy of ZS-9 in patients with hyperkalemia. In ZS-003, treatment of patients with an oral suspension of ZS-9 (2.5, 5, or 10 grams, three times a day) resulted in statistically significant and clinically meaningful reductions in serum potassium, compared with placebo, during the “acute phase” (first 48 hours), with 99 percent of patients achieving normal potassium levels with the highest 10 gram dose. During the next 12 days of the trial (the “maintenance phase”), ZS- 9 (5 or 10 grams) given once daily could maintain the corrected potassium levels achieved during the acute phase. In contrast, patients who were randomized back to placebo after achieving normal potassium reverted back to hyperkalemia. The tolerability profile has been favorable, with adverse event rates from ZS-9 similar to that of placebo. Read the rest of the interviews on MedicalResearch.com Content NOT an endorsement of efficacy and NOT intended as specific medical advice.
  • 29. New Drug For Hyperkalemia Selectively Traps Potassium MedicalResearch.com Interview with: David K. Packham, M.B., B.S., M.D Royal Melbourne Hospital Melbourne Renal Research Group VIC 3073, Australia, • Medical Research: What should clinicians and patients take away from your report? • Dr. Packham: Hyperkalemia increases the risk of mortality and limits use of life-saving renin angiotensin-aldosterone inhibitors (RAASi) in patients with heart failure (HF), chronic kidney disease (CKD), and diabetes mellitus (DM). Current therapies for hyperkalemia include loop diurectics and nonspecific organic polymer resins (eg, sodium polystyrene sulfonate); however, these have inconsistent, questionable efficacy and carry significant gastrointestinal intolerability. From our studies, ZS-9 has been observed to rapidly and predictably lower and maintain normal serum potassium levels, while demonstrating a favorable safety and tolerability profile. ZS-9 offers potential promise for a safe and effective new therapy which will enable life saving cardio- and renoprotective RAAS therapies in patients with heart failure, CKD, and DM. Furthermore, other observations suggest that ZS-9 has may improve serum bicarbonate, thus demonstrating potential to improve metabolic acidosis, a key factor in the progression of kidney disease. Read the rest of the interviews on MedicalResearch.com Content NOT an endorsement of efficacy and NOT intended as specific medical advice.
  • 30. New Drug For Hyperkalemia Selectively Traps Potassium MedicalResearch.com Interview with: David K. Packham, M.B., B.S., M.D Royal Melbourne Hospital Melbourne Renal Research Group VIC 3073, Australia, • Medical Research: What recommendations do you have for future research as a result of this study? • Dr. Packham: There are 4 potential areas for future research under consideration: • Emergency department setting for acute therapy of hyperkalemia: Subgroup analyses of our studies have consistently shown that the potassium lowering effect of ZS-9 is rapid (within an hour) and has greatest potency among patients with the severest cases of hyperkalemia (ie, serum potassium ≥6mEq/l). Based on our observations in these studies, we believe that ZS-9 has potential to be an alternative therapy to emergent dialysis in emergency or hospital settings. • Long term safety and efficacy: ZS-005 is an ongoing study that is evaluating ZS-9 as long-term (≥12-month) therapy for patients who require chronic treatment for hyperkalemia. • Dialysis patients: The ability to address hyperkalemia on non-dialysis days, reduce the pre-dialysis potassium levels to enable lower potassium baths, and liberate diet are exciting considerations for a non- polymer based therapy, such as ZS-9, which does not adversely interact with polymer phosphate binders. • Transplant patients: Anti-rejection medications carry the risk of hyperkalemia and threaten the ability to optimize transplant medications. Maintenance of normal potassium in transplant patients with ZS-9 may permit optimal use of transplant medications. • ACEi, ARB, and Mineralocorticoid Dose Optimization: Hyperkalemia is often the rate-limiting side effect of these important RAAS medications. Clinical studies of ZS-9 have demonstrated that its effect is independent of concomitant use of RAAS therapies. Dose optimization may be permitted for such therapies that are no longer limited because of hyperkalemia. • Citation: • Sodium Zirconium Cyclosilicate in Hyperkalemia • David K. Packham, M.B., B.S., M.D., Henrik S. Rasmussen, M.D., Ph.D., Philip T. Lavin, Ph.D., Mohamed A. El-Shahawy, M.D., M.P.H., Simon D. Roger, M.D., Geoffrey Block, M.D., Wajeh Qunibi, M.D., Pablo Pergola, M.D., Ph.D., and Bhupinder Singh, M.D. • N Engl J Med 2015; 372:222-231 January 15, 2015 DOI: 10.1056/NEJMoa1411487 Read the rest of the interviews on MedicalResearch.com Content NOT an endorsement of efficacy and NOT intended as specific medical advice.
  • 31. New Cancer Fighting Strategy Attacks Telomere Pathway In Difficult Tumors MedicalResearch.com Interview with: Lee Zou, Ph.D. Professor of Pathology, Harvard Medical School The Jim & Ann Orr MGH Research Scholar Associate Scientific Director, MGH Cancer Center • Medical Research: What is the background for this study? What are the main findings? Dr. Lee Zou: Cancer cells must rely on telomerase or the alternative lengthening of telomere (ALT) pathway to maintain telomeres and bypass replicative senescence. The ALT pathway is active in about 10-15% of human cancers, and it is particularly prevalent in specific cancer types, such as osteosarcoma, glioblastoma, and neuroendocrine pancreatic tumors. ALT is a recombination-mediated process. Whether the reliance of cancer cells on alternative lengthening of telomere can be exploited therapeutically was not known. • In our study, we discovered that the ATR kinase is a key regulator of alternative lengthening of telomere. We found that ATR inhibitors disrupt ALT effectively. Furthermore, we found that ATR inhibitors selectively kill ALT-positive cancer cells in a panel of caner cell lines. These findings have suggested the first rational therapeutic strategy for the treatment of ALT- positive cancer. Read the rest of the interviews on MedicalResearch.com Content NOT an endorsement of efficacy and NOT intended as specific medical advice.
  • 32. New Cancer Fighting Strategy Attacks Telomere Pathway In Difficult Tumors MedicalResearch.com Interview with: Lee Zou, Ph.D. Professor of Pathology, Harvard Medical School The Jim & Ann Orr MGH Research Scholar Associate Scientific Director, MGH Cancer Center • Medical Research: What should clinicians and patients take away from your report? • Dr. Lee Zou: Many of the ALT-positive cancers are difficult-to-treat cancers. Our findings have provided clinicians with a new therapeutic strategy to explore in preclinical and clinical studies. In particular, several ATR inhibitors have been recently developed and are entering clinical trials. I hope that our findings will motivate clinician to investigate the efficacy of alternative lengthening of telomere inhibitors in the treatment of ALT-positive cancers. • Our findings have provided cancer patients with new hope. However, it is important to note that further preclinical and clinical work is still needed to validate and refine this new therapeutic strategy. Read the rest of the interviews on MedicalResearch.com Content NOT an endorsement of efficacy and NOT intended as specific medical advice.
  • 33. New Cancer Fighting Strategy Attacks Telomere Pathway In Difficult Tumors MedicalResearch.com Interview with: Lee Zou, Ph.D. Professor of Pathology, Harvard Medical School The Jim & Ann Orr MGH Research Scholar Associate Scientific Director, MGH Cancer Center • Medical Research: What recommendations do you have for future research as a result of this study? • Dr. Lee Zou: I hope that our findings will motivate clinicians and patients to explore this new therapeutic strategy is clinical trials. Our work also suggest that DNA recombination could be a good therapeutic target in certain cancers. For future research, it would be interesting to identify additional cancers that are dependent on DNA recombination and to target them with ATR inhibitors. • Citation: • Science. 2015 Jan 16;347(6219):273-7. doi: 10.1126/science.1257216. • Alternative lengthening of telomeres renders cancer cells hypersensitive to ATR inhibitors. • Flynn RL1, Cox KE2, Jeitany M3, Wakimoto H4, Bryll AR2, Ganem NJ2, Bersani F5, Pineda JR3, Suvà ML6, Benes CH7, Haber DA5, Boussin FD3, Zou L8. Read the rest of the interviews on MedicalResearch.com Content NOT an endorsement of efficacy and NOT intended as specific medical advice.
  • 34. Breast Cancer Patients May Benefit From Detailed Nutrition Monitoring MedicalResearch.com Interview with: Cecilia Cesa Schiavon Department of Nutrition, Federal University of Santa Catarina Florianópolis, Santa Catarina, Brazil • Medical Research: What is the background for this study? What are the main findings? Response: The study was based on a nutritional intervention for patients undergoing treatment for breast cancer. The intervention took place right after the surgical procedure and lasted about a year, until the end of chemotherapy. The patients were submitted to a special methodology of intervention, aimed at increasing fruit and vegetable intake and reducing red and processed meat, following the World Cancer Research Fund and the American Institute for Cancer Research in the document entitled Food, Nutrition, Physical Activity and the prevention of Cancer: A Global Perspective”. • The main findings show that women undergoing breast cancer treatment may benefit from immediate, individualized, and detailed nutrition monitoring through appropriate nutrition education. • Medical Research: What should clinicians and patients take away from your report? • Response: I think that clinicians and patients can consider the possibility of improving the quality of the diet of patients, even during breast cancer treatment. And it is also a big motivation to know that a healthy diet can contribute to a reduction of the side effects of chemotherapy, and in consequence this can help reduce the risk of breast cancer recurrence. Read the rest of the interviews on MedicalResearch.com Content NOT an endorsement of efficacy and NOT intended as specific medical advice.
  • 35. Breast Cancer Patients May Benefit From Detailed Nutrition Monitoring MedicalResearch.com Interview with: Cecilia Cesa Schiavon Department of Nutrition, Federal University of Santa Catarina Florianópolis, Santa Catarina, Brazil • Medical Research: What recommendations do you have for future research as a result of this study? • Response: We suggest a study with more time and more resources to have a larger sample size and to include patients from other health centers. • Additionally, the ideal situation would be to be able to supply the patients with fruits and vegetables to guarantee easy access and, consequently, monitor the consumption more closely. • Citation: • J Nutr Educ Behav. 2015 Jan-Feb;47(1):2-9. doi: 10.1016/j.jneb.2014.09.005. • Nutrition education intervention for women with breast cancer: effect on nutritional factors and oxidative stress. • Schiavon CC1, Vieira FG1, Ceccatto V1, de Liz S1, Cardoso AL1, Sabel C2, Gonzalez-Chica DA1, da Silva EL2, Galvan D1, Crippa CG3, Di Pietro PF4. Read the rest of the interviews on MedicalResearch.com Content NOT an endorsement of efficacy and NOT intended as specific medical advice.
  • 36. Healthy Diet Lowers Diabetes Risk in All Ethnic Groups MedicalResearch.com Interview with: Jinnie J. Rhee Department of Epidemiology, Harvard School of Public Health, Boston, MA • Medical Research: What is the background for this study? What are the main findings? Response: The goal of this study was to see if the dietary determinants of type 2 diabetes observed in predominantly white populations were similar to those in other racial and ethnic groups. We created a dietary diabetes risk reduction score using eight different dietary factors found to be associated with risk of type 2 diabetes, where a higher score indicates a healthier overall diet (A higher score included low intakes trans fat, sugar-sweetened beverages, and red and processed meats; lower glycemic index; and higher intakes of cereal fiber, nuts, and coffee; and higher polyunsaturated to saturated fat ratio). We found a protective association of similar magnitude between a healthy overall diet and type 2 diabetes risk in all racial and ethnic groups. However, in terms of the actual number of preventable cases, a healthier diet conferred even greater benefit for minority women because they were initially at higher risk than white women. • This study is significant because diabetes is a rapidly growing epidemic in most parts of the world, but most previous studies of diet and diabetes have been conducted in populations of European origin. This analysis was very powerful because it combined two large populations with a total of 156,030 women who were followed for up to 28 years with many repeated assessments of diet. This allowed us to conduct detailed analyses within specific racial and ethnic groups. Read the rest of the interviews on MedicalResearch.com Content NOT an endorsement of efficacy and NOT intended as specific medical advice.
  • 37. Healthy Diet Lowers Diabetes Risk in All Ethnic Groups MedicalResearch.com Interview with: Jinnie J. Rhee Department of Epidemiology, Harvard School of Public Health, Boston, MA • Medical Research: What should clinicians and patients take away from your report? • Response: These findings suggest that a healthy overall diet can play an important role in the prevention of type 2 diabetes, particularly in high-risk groups. The take-home message is that regardless of one’s race and ethnicity, a healthy diet can play an important role in preventing diabetes, and in fact, the benefit may be greater for minority women who already have elevated risks of diabetes. A healthier diet, especially that of lower glycemic foods and lower intakes of sugar sweetened beverages and red and processed meats, and higher intakes of cereal fiber and coffee should be encouraged. Read the rest of the interviews on MedicalResearch.com Content NOT an endorsement of efficacy and NOT intended as specific medical advice.
  • 38. Healthy Diet Lowers Diabetes Risk in All Ethnic Groups MedicalResearch.com Interview with: Jinnie J. Rhee Department of Epidemiology, Harvard School of Public Health, Boston, MA • Medical Research: What recommendations do you have for future research as a result of this study? • Response: This study marks a big step forward in diabetes research because it shows that a dietary pattern found to be beneficial in a population mainly European origin can be applicable to most of the world’s population of women. Future research should focus on how this knowledge can be applied into practice. This includes finding ways to address health disparities that are influenced by race, income, and other socioeconomic factors • Citation: • Jinnie J. Rhee, Josiemer Mattei, Michael D. Hughes, Frank B. Hu, and Walter C. Willett • Dietary Diabetes Risk Reduction Score, Race and Ethnicity, and Risk of Type 2 Diabetes in Women Diabetes Care published ahead of print January 15, 2015, doi:10.2337/dc14-1986 Read the rest of the interviews on MedicalResearch.com Content NOT an endorsement of efficacy and NOT intended as specific medical advice.
  • 39. Buddy System May You Swap Bad Habits For Good Ones MedicalResearch.com Interview with: Jane Wardle University College London • Medical Research: What is the background for this study? What are the main findings? Dr. Wardle: Previous studies have shown that couples tend to have similar health behaviours to one another, but no studies had compared having a partner who takes up a healthy behaviour (e.g. quits smoking) with having one whose behaviour is consistently healthy (e.g. never smoked). Nor have there been other studies in the older age group – our participants were over 60 on average. We used data from 3722 couples participating in the English Longitudinal Study of Ageing (ELSA) to explore this issue for three behaviours: smoking, physical activity, and weight loss. For each behaviour, we found that when one partner changed their behaviour, the other partner was more likely to make a positive change, and the effect was stronger than having a partner whose behaviour was consistently healthy (i.e. never smoked/always exercised). • Medical Research: What should clinicians and patients take away from your report? • Dr. Wardle: We know from decades of research that information alone is not enough to get everyone to take up a healthy lifestyle. Changing together may make things easier. Perhaps clinicians could encourage people to bring their spouse in too. Your nearest and dearest may be your best buddy – but where that’s not an option, perhaps another buddy system could help people to swap their bad habits for healthier ones. Read the rest of the interviews on MedicalResearch.com Content NOT an endorsement of efficacy and NOT intended as specific medical advice.
  • 40. Buddy System May You Swap Bad Habits For Good Ones MedicalResearch.com Interview with: Jane Wardle University College London • Medical Research: What recommendations do you have for future research as a result of this study? • Dr. Wardle: This study is observational – in that we compared couples where the partner changed with couples where they didn’t. Future research needs to test this in a trial – can we harness the partner effect in treatment. And can we identify what is important about the partner effect to translate to other potential buddies? • Citation: • Sarah E. Jackson, Andrew Steptoe, Jane Wardle. The Influence of Partner’s Behavior on Health Behavior Change. JAMA Internal Medicine, 2015; DOI: 10.1001/jamainternmed.2014.7554 Read the rest of the interviews on MedicalResearch.com Content NOT an endorsement of efficacy and NOT intended as specific medical advice.
  • 41. Hardening of Aorta Linked To Poor Cognitive Performance MedicalResearch.com Interview with: Jose Gutierrez MD, MPH Assistant Professor of Neurology Division of Stroke and Cerebrovascular Disease Columbia University Medical Center NY, NY • Medical Research: What is the background for this study? What are the main findings? Dr. Gutierrez: There is growing interest in the effects of vascular health in cognition. The prevailing thought is that vascular disease leads to worse cognition due to direct structural damage of the brain, as in the case of brain infarcts, microhemorrhages or white matter hyperintensities, which are themselves associated with traditional cardiovascular risk factors such as hypertension, diabetes, smoking etc. Arterial stiffness, particularly of the aorta, has gained interest among researchers as predictors of vascular disease and worse cognition, but it is not clear whether arterial stiffness in the absence of traditional definition of vascular disease may be associated with worse cognition. • We investigated in a representative sample of the US among adults 60 years or older who underwent cognitive testing with the Digit Symbol Subtraction test and who also had other measures of vascular disease, including blood workup, blood pressure measurement and Pulse pressure. We hypothesized that indirect measures of arterial stiffness such as ABI > 1.3 or pulse pressure would be associated with worse cognition, even among those without any clinical vascular disease or traditional vascular risk factors. We Included 2573 US adults in the sample, segregated those with any self-reported vascular disease or vascular risk factors and we found that among those without vascular disease or risk factors, an ABI > 1.3 and increased intra-visit blood pressure variability were predictors of worse cognitive performance compared with those without these indicators. Among participants with both indirect markers of arterial stiffness, their cognitive performance was worse that having only one of them suggesting additive effects of these two variables. Read the rest of the interviews on MedicalResearch.com Content NOT an endorsement of efficacy and NOT intended as specific medical advice.
  • 42. Hardening of Aorta Linked To Poor Cognitive Performance MedicalResearch.com Interview with: Jose Gutierrez MD, MPH Assistant Professor of Neurology Division of Stroke and Cerebrovascular Disease Columbia University Medical Center NY, NY • Medical Research: What should clinicians and patients take away from your report? • Dr. Gutierrez: This results could be used by clinicians to screen patients without apparent vascular disease for arterial stiffness, and eventually, if positive, refer them for more accurate measure of arterial stiffness such as carotid-femoral pulse wave velocity. Increasing awareness of the vascular component of cognition. • Medical Research: What recommendations do you have for future research as a result of this study? • Dr. Gutierrez: We need to know whether medications that can alter aortic stiffness may be beneficial to preserves cognition among these individuals. Physical exercise appears to have a protective effects against cognitive decline, so it would be important to see whether exercise decreases aortic stiffness, thus mediating the effects on cognition. It would be desirable to investigate further if certain antihypertensives may be preferable in patient with increased aortic stiffness. Ideally, a patient should come to our office and be screen for vascular disease. If positive, then aggressively controls their vascular risk factors. If negative, then proceed with measure of aortic stiffness that may lead to individualized therapies among those not typically consider to have vascular disease. Evidently, all these algorithm should be tested in clinical trials. • Citation: • Gutierrez J, Marshall RS, Lazar RM. Indirect Measures of Arterial Stiffness and Cognitive Performance in Individuals Without Traditional Vascular Risk Factors or Disease. JAMA Neurol. Published online January 19, 2015. doi:10.1001/jamaneurol.2014.3873. • Email * Read the rest of the interviews on MedicalResearch.com Content NOT an endorsement of efficacy and NOT intended as specific medical advice.
  • 43. Is Strict Salt Limit Necessary For Older Adults? MedicalResearch.com Interview with: Andreas Kalogeropoulos, MD MPH PhD Assistant Professor of Medicine (Cardiology) Emory University School of Medicine Emory Clinical Cardiovascular Research Institute Atlanta GA 30322 • Medical Research: What is the background for this study? What are the main findings? • Dr. Kalogeropoulos: There is ongoing debate on how low should we go when it comes to dietary sodium (salt) restriction recommendations. In this study, we examined the association between self-reported dietary sodium intake and 10-year risk for death, cardiovascular disease, and heart failure in approximately 2,600 adults 71-80 years old. The subjects (women: 51.2%; white: 61.7%; black: 38.3%) were participants of the community-based Health, Aging, and Body Composition Study, which is sponsored by NIH and focuses on aging processes, i.e. was not specifically designed to address the issue of dietary salt intake. Also, it is important to note that salt intake was self-reported (not objectively measured) using a food frequency questionnaire, which underestimates salt intake. Keeping these limitations in mind, we did not observe a significant association between self-reported sodium intake and 10-year mortality, cardiovascular disease, and heart failure. Ten-year mortality was lower in the group reporting 1500–2300 mg daily sodium intake (30.7%) compared to those reporting daily intake less than 1500 mg (33.8%) or over 2300 mg (35.2%); however, this difference was not statistically significant. The 10-year event rates for cardiovascular disease (28.5%, 28.2%, and 29.7%) and heart failure (15.7%, 14.3%, and 15.5%) were also comparable across the <1500-mg, 1500-2300-mg, and >2300- mg dietary sodium intake groups. Read the rest of the interviews on MedicalResearch.com Content NOT an endorsement of efficacy and NOT intended as specific medical advice.
  • 44. Is Strict Salt Limit Necessary For Older Adults? MedicalResearch.com Interview with: Andreas Kalogeropoulos, MD MPH PhD Assistant Professor of Medicine (Cardiology) Emory University School of Medicine Emory Clinical Cardiovascular Research Institute Atlanta GA 30322 • Medical Research: What should clinicians and patients take away from your report? • Dr. Kalogeropoulos: The current CDC guidelines for dietary sodium intake, i.e. <2300 mg for the general population (corresponding to 6 grams or a teaspoon of salt) with <1500 mg reserved for high risk groups, including older adults, are probably the best approach until more data are available. Although our findings do not support the strict (<1500 mg daily) sodium restriction recommendation for the general population of older adults, there was no signal of harm either. However, a more conservative approach to sodium restriction (that is, targeting <2300 mg/d) might be appropriate for older adults until more data become available. Of note, our findings do not apply to older adults with pre-existing cardiovascular disease or heart failure. • Patients should not interpret our findings as a “license” to consume more salt. First, average dietary salt intake is already too high in most Western or westernized communities and well above the average reported intake in our study. Second, there is no question whether moderating salt intake provides health benefits. We know that going from high-salt (e.g. 12 g or two teaspoons a day) to low-salt (6 g or one teaspoon a day) diet is good for health. We are now trying to fine-tune the most appropriate level of “low” that provides the most benefit, i.e., would that be 6 g a day or lower? Third, high salt intake (i.e., far more than a teaspoon daily) was associated with a trend towards increased mortality in our study, too; we just did not have enough participants in this category to be statistically confident about these trends. Therefore, patients should follow the current guidelines and consult with their healthcare provider as the best approach. Read the rest of the interviews on MedicalResearch.com Content NOT an endorsement of efficacy and NOT intended as specific medical advice.
  • 45. Is Strict Salt Limit Necessary For Older Adults? MedicalResearch.com Interview with: Andreas Kalogeropoulos, MD MPH PhD Assistant Professor of Medicine (Cardiology) Emory University School of Medicine Emory Clinical Cardiovascular Research Institute Atlanta GA 30322 • Medical Research: What recommendations do you have for future research as a result of this study? • Dr. Kalogeropoulos: Our study is not the first to cast doubt on the strict (<1500 mg daily) sodium intake recommendation — although it is the first one to look at older adults specifically. Considering the special case of older adults, in whom comorbidities, inadequate caloric intake, and medication interactions are additional concerns with very low sodium intake, we probably need to test the effect of sodium restriction on outcomes explicitly in this special population before implementing a generalized recommendation for very low (<1500 mg/d) sodium intake. Of note, the Institute of Medicine has highlighted the lack of data on the appropriate level of sodium restriction for older adults and other special populations, including patients with pre-existing heart failure. In the latter, a limited number of studies to date has not corroborated the traditional recommendation for sodium restriction, highlighting the need for more rigorous studies and more definitive evidence in special populations. • Citation: • Kalogeropoulos AP, Georgiopoulou VV, Murphy RA, et al. Dietary Sodium Content, Mortality, and Risk for Cardiovascular Events in Older Adults: The Health, Aging, and Body Composition (Health ABC) Study. JAMA Intern Med. Published online January 19, 2015. doi:10.1001/jamainternmed.2014.6278. Read the rest of the interviews on MedicalResearch.com Content NOT an endorsement of efficacy and NOT intended as specific medical advice.
  • 46. VA Study Examines Variability of Costs and Mortality of Stent Surgery MedicalResearch.com Interview with: Dr. P. Michael Ho, MD PhD Denver Veteran Affairs Medical Center, University of Colorado, Denver, Section of Cardiology Denver, Colorado 80220. • Medical Research: What is the background for this study? What are the main findings? • Dr. Ho: There is increasing interest in measuring health care value, particularly as the healthcare system moves towards accountable care. Value in health care focuses on measuring outcomes achieved relative to costs for a cycle of care. Attaining high value care – good clinical outcomes at low costs – is of interest to patients, providers, health systems, and payers. To date, value assessments have not been operationalized and applied to specific patient populations. We focused on percutaneous coronary intervention (PCI) because it is an important aspect of care for patients with ischemic heart disease, is commonly performed and is a costly procedure. In this study, we evaluated 1-year risk-adjusted mortality and 1-year risk-standardized costs of care for all patients who underwent PCI in the VA healthcare system from 2008 to 2010. • We found that median one-year unadjusted hospital mortality rate was 6.13% (interquartile range 4.51% to 7.34% across hospitals). Four hospitals were significantly above the one-year risk standardized median mortality rate, with median mortality ratios ranging from 1.23 to 1.28; no hospitals were significantly below median mortality. Median 1-year total unadjusted hospital costs were $46,302 (IQR of $37,291 to $57,886) per patient. There were 16 hospitals above and 19 hospitals below the risk standardized average cost, with risk standardized ratios ranging from 0.45 to 2.09 reflecting much larger magnitude of variability in costs compared to mortality. These findings suggest that there are opportunities to improve PCI healthcare by reducing costs without compromising outcomes. This approach of evaluating outcomes and costs together may be a model for other health systems and accountable care organizations interested in operationalizing value measurement. Read the rest of the interviews on MedicalResearch.com Content NOT an endorsement of efficacy and NOT intended as specific medical advice.
  • 47. VA Study Examines Variability of Costs and Mortality of Stent Surgery MedicalResearch.com Interview with: Dr. P. Michael Ho, MD PhD Denver Veteran Affairs Medical Center, University of Colorado, Denver, Section of Cardiology Denver, Colorado 80220. • Medical Research: What should clinicians and patients take away from your report? • Dr. Ho: These findings are more relevant to hospitals, health care systems, and policy makers. Most of the variation across hospitals of longitudinal PCI care occurs as a result of differences in costs with much less variation in patient outcomes. This suggests that value of PCI care for the VA may be enhanced through reducing unnecessary variation in costs of care, without compromising patient outcomes. • Medical Research: What recommendations do you have for future research as a result of this study? • Dr. Ho: Future research will be needed to identify specific reasons for the cost variations identified, one of which may be regional differences in practice patterns. For inpatient care, there may be structural factors related to staffing of cath labs and/or supply costs (e.g., coronary stents) which are negotiated by individual hospitals. For outpatient care, there will be opportunities to explore differences in follow-up care, some of which may be related to intensity of care provided, frequency of cardiac testing and/or need for non-cardiac related care. We are planning on conducting qualitative interviews at hospitals to identify reasons for the cost variations found in this study. • Citation: • 1-Year Risk-Adjusted Mortality and Costs of Percutaneous Coronary Intervention in the Veterans Health Administration: Insights From the VA CART Program • P. Michael Ho, MD, PhD, Colin I. O’Donnell, MS, Steven M. Bradley, MD, MPH Gary K. Grunwald, PhD Christian Helfrich, PhD Michael Chapko, PhD Chuan-Fen Liu, PhD Thomas M. Maddox, MD, MSc Thomas T. Tsai, MD, MSc Robert L. Jesse, MD, PhD, Stephan D. Fihn, MD, MPH John S. Rumsfeld, MD, PhD • Journal of the American College of Cardiology Volume 65, Issue 3, 27 January 2015, Pages 236–242 Read the rest of the interviews on MedicalResearch.com Content NOT an endorsement of efficacy and NOT intended as specific medical advice.
  • 48. Sitting Time Linked To Increased Heart Disease and Cancer Mortality MedicalResearch.com Interview with: David Alter, MD, PhD FRCPC Senior Scientist Toronto Rehabilitation Institute-University Health Network and Institute for Clinical Evaluative Sciences Research Director, Cardiac Rehabilitation and Secondary Prevention Program Toronto Rehabilitation Institute Medical Research: What is the background for this study? What are the main findings? Dr. Alter: We knew going into the study that exercise was an important lifestyle factor that improved health. We also knew from studies that sedentary time was associated with deleterious health-effects. What we didn’t know was whether the health-outcome effects of sedentary time and exercise were really one and the same (i.e., albeit opposite ends of the same spectrum) or alternatively, whether the health effects of each were independent of one another. We explored over 9000 published studies to quantify the health-outcome effects associated with sedentary behaviour and extracted only those which took into account both sedentary time and exercise. We found a consistent association between sedentary time and a host of health outcomes independent of exercise. Specifically, after controlling for an individual’s exercising behaviour, sitting-time was associated with a 15-20% higher risk of death, heart-disease, death from heart disease, cancer-incidence, and death from cancer. Sitting time was also independently associated with a marked (i.e., 90% increase) in the risk for diabetes after controlling for exercise. In short, sedentary times and exercise are each independently associated with health outcomes. We hypothesize that the two may have different mechanism, and may require different therapeutic strategies. But, the health-outcome implications of both are each important in their own right. Read the rest of the interviews on MedicalResearch.com Content NOT an endorsement of efficacy and NOT intended as specific medical advice.
  • 49. Sitting Time Linked To Increased Heart Disease and Cancer Mortality MedicalResearch.com Interview with: David Alter, MD, PhD FRCPC Senior Scientist Toronto Rehabilitation Institute-University Health Network and Institute for Clinical Evaluative Sciences Research Director, Cardiac Rehabilitation and Secondary Prevention Program Toronto Rehabilitation Institute • Medical Research: What should clinicians and patients take away from your report? • Dr. Alter: A call to action. Several indeed: • First, from a public-health perspective, the signal between sedentary time and deleterious health- outcomes is one that is consistent for multiple diseases and health outcomes. Traditionally, public health and health promotion messaging has focused on exercise alone (among other healthy lifestyle behaviours like diet and smoking-cessation etc.). However, we believe that our study justifies the need for a modification in public-health and health promotion messaging – We must speak about the benefits of both “Exercise” and “Avoidance of excessive sitting” – A two-pronged message – both are important, but distinct…. “An hour per day of exercise does not give the right of passage to sit for the remaining 23” – It’s time we incorporate the sedentary message into public health. • Second, while we need more scientifically-rigorous research on how best to decrease sitting times, there are things we can do as patients and health care providers in the meantime as the science of sitting times evolves. “Breaking cultural norms” is a challenge. However, we need to begin by providing greater specificity and granularity to our recommendations. The generic “sit less” “exercise more” is often lost on us. We know this already. What we often struggle with, however, is the “how”. While admittedly anecdotal, I’ve had success by setting out a step-by-step plan for my patients. I set forth tangible and achievable tasks that patients can follow which promote self-monitoring, goal-set, and feedback. • How do I do this for my patients? I begin by having patients monitor their sleeping times, sitting times, and exercise times per day using either a simple app or log-book diary I provide to them. Generally, one week of monitoring serves as a starting point. This task does two things: It engages patients into monitoring their sitting time behaviours to draw awareness to the issue, and serves as baseline set of measures against which I compare when monitoring progress. . Then, I have patients set a goal – “how many fewer minutes per day should you sit over the next week?” And, “how will you achieve this?” Read the rest of the interviews on MedicalResearch.com Content NOT an endorsement of efficacy and NOT intended as specific medical advice.
  • 50. Sitting Time Linked To Increased Heart Disease and Cancer Mortality MedicalResearch.com Interview with: David Alter, MD, PhD FRCPC Senior Scientist Toronto Rehabilitation Institute-University Health Network and Institute for Clinical Evaluative Sciences Research Director, Cardiac Rehabilitation and Secondary Prevention Program Toronto Rehabilitation Institute • Some practical tips? Begin by having the patient setting a timer at work to remind them on the half-hour, to get up and stretch from their sitting position and stroll for 2 minutes; There are a host of other activities…..For example, every commercial break on TV, do the same; watch the last 5-10 minutes of a show standing than sitting….great for sports….Have patients multitask during their leisure time. For example, I am currently typing this email while on my elliptical…spelling errors aside, it feels like a “healthy way to message” in more ways than one. Finally, I have patients return where I will provide them with feedback. What health benefits did they achieve over the last week or two by sitting less? This, in my experience, is VERY IMPORTANT. What patients (and providers) don’t realize is how much can be achieved with just little changes. For example, we burn 2 times as many calories standing as we do sitting. For an average 70 kg male, that is an incremental difference of 70 Kcal/hour. Walking will burn even more than standing- Doing the math, you realize that you can burn quite a number of calories per day through these simple strategies. Cumulatively over a week, month, or year, the numbers add up. • There are many potential approaches when feeding back such information – – the aim is for patients to internalize the health-benefits of this behaviour change. There are all sorts of software apps and tools that can help… And more will undoubtedly emerge in the coming years. • The bottom line: reducing sitting time is a distinct priority from our 30-45 minutes per day requirement for moderate to vigorous exercise – -both need to be done. Both need to be promoted. Both should be thought of as “health hygiene”. Read the rest of the interviews on MedicalResearch.com Content NOT an endorsement of efficacy and NOT intended as specific medical advice.
  • 51. Sitting Time Linked To Increased Heart Disease and Cancer Mortality MedicalResearch.com Interview with: David Alter, MD, PhD FRCPC Senior Scientist Toronto Rehabilitation Institute-University Health Network and Institute for Clinical Evaluative Sciences Research Director, Cardiac Rehabilitation and Secondary Prevention Program Toronto Rehabilitation Institute • Medical Research: What recommendations do you have for future research as a result of this study? • Dr. Alter: More research is needed. Indeed, we have barely scratched the surface here. This underscores another call to action – – that to our research community. First, our study demonstrated that the measurement of sitting times varied from study to study. Definitions were all over the map in the studies explored. Therefore, we need a more uniform and consistent definition so that we are “speaking the same language” when embarking on further research. Second, we need to better understand the mechanisms by which sedentary behaviours lead to poorer health, and how this differs from exercise and it’s health promoting attributes. Plausible mechanisms are needed to help build the credibility of the message – – especially for clinicians and scientists. Teasing out the mechanisms will also give way to new approaches and solutions. So, we need to look “underneath the hood” and dig deeper into mechanisms. Third, we need to better understand how best to “intervene” in order to change the sedentary behaviours of individuals and what effects if any, such therapeutic interventions have on health outcomes. Solutions need be integrative and interwoven within an individual’s 12 hour waking day period- – In so doing, we also need to find better and more innovative ways of quantifying the health-benefits of sitting-less – – These will not be as aesthetic as weight loss. Nor might they feel as intensive as exercise. We need to incorporate health metrics which connect with patients in a way that motivates them from pushing on and sustaining their reductions in sitting times – – after all, solutions may be quite simple and intuitive – but will they resonate with patients? Finally, our approach to solutions may differ from individual to individual For example, our study suggested that the deleterious effects of sitting time were most pronounced among those who did not participate in any regular exercise. On the flip side, the adverse health effects of sitting may diminished in magnitude among those who do participate avidly in regular exercise. This suggests that our research priorities may necessitate a better understanding of the population – – who are the sedentary non-avid exercisers in the population? Do they need different approaches to their otherwise sedentary but avid-exercise counterparts? Read the rest of the interviews on MedicalResearch.com Content NOT an endorsement of efficacy and NOT intended as specific medical advice.
  • 52. Sitting Time Linked To Increased Heart Disease and Cancer Mortality MedicalResearch.com Interview with: David Alter, MD, PhD FRCPC Senior Scientist Toronto Rehabilitation Institute-University Health Network and Institute for Clinical Evaluative Sciences Research Director, Cardiac Rehabilitation and Secondary Prevention Program Toronto Rehabilitation Institute • In short, there is a lot more work to be done. We are hoping that this study serves as a call to action for all! • Citation: • Biswas A, Oh PI, Faulkner GE, Bajaj RR, Silver MA, Mitchell MS, et al. Sedentary Time and Its Association With Risk for Disease Incidence, Mortality, and Hospitalization in Adults: A Systematic Review and Meta-analysis. Ann Intern Med. 2015;162:123-132. doi:10.7326/M14- 1651 Read the rest of the interviews on MedicalResearch.com Content NOT an endorsement of efficacy and NOT intended as specific medical advice.
  • 53. High-Intensity Statin Therapy Underutilized After Heart Attack MedicalResearch.com Interview with: Dr. Robert S. Rosenson, MD Professor, Cardiology Icahn School of Medicine at Mount Sinai Cardiovascular Institute New York, New York 10029 • Medical Research: What is the background for this study? What are the main findings? Dr. Rosenson: High intensity statin therapy is evidence-based and guideline directed for patients with acute coronary syndromes. In a 5 percent random sample of Medicare patients, we investigated the utilization of high vs low-moderate dosage statin in older adjusts who were admitted with an acute myocardial infarction of severe myocardial ischemia requiring hospitalization for a revascularization procedure (PCI or CABG). • We report that only 27 percent of hospitalized patients received high-intensity statin therapy based on their first outpatient fill for a statin medication. The most important determinant for the utilization of statin therapy is the dosage of the statin previously prescribed as an outpatient. When patients were started on a high-intensity statin, the continued use diminished in the ensuing year • Medical Research: What should clinicians and patients take away from your report? • Dr. Rosenson: Evidence-based guidelines recommend in hospital initiation of high-intensity statin for patients with acute coronary syndromes, and this therapy should be continued long-term. It is important to understand that we do not treat these patients to a LDL cholesterol target as statins have other salutary effects that extend beyond cholesterol lowering. Read the rest of the interviews on MedicalResearch.com Content NOT an endorsement of efficacy and NOT intended as specific medical advice.
  • 54. High-Intensity Statin Therapy Underutilized After Heart Attack MedicalResearch.com Interview with: Dr. Robert S. Rosenson, MD Professor, Cardiology Icahn School of Medicine at Mount Sinai Cardiovascular Institute New York, New York 10029 • Medical Research: What recommendations do you have for future research as a result of this study? • Dr. Rosenson: We have plans to evaluate treatment patterns and the impact on cardiovascular events. • Citation: • Underutilization of High-Intensity Statin Therapy After Hospitalization for Coronary Heart Disease • Robert S. Rosenson, MD , Shia T. Kent, PhD Todd M. Brown, MD Michael E. Farkouh, MD Emily B. Levitan, PhD†, Huifeng Yun, MD, PhD Pradeep Sharma, MS Monika M. Safford, MD Meredith Kilgore, PhD Paul Muntner, PhD Vera Bittner, MD • Journal of the American College of Cardiology • Volume 65, Issue 3, 27 January 2015, Pages 270–277 Read the rest of the interviews on MedicalResearch.com Content NOT an endorsement of efficacy and NOT intended as specific medical advice.
  • 55. Key Clock Genes Linked To Alcohol Use and Abuse MedicalResearch.com Interview with: Timo Partonen MD, Research Professor National Institute for Health and Welfare Helsinki, Finland • Medical Research: What is the background for this study? What are the main findings? Response: Alcohol-use disorders are often comorbid conditions with mood and anxiety disorders. Clinical studies have demonstrated that there are abnormalities in circadian rhythms and intrinsic clocks in patients with alcohol-use disorders. Circadian clock gene variants are therefore a fruitful target of interest. • The main findings are that variants of key clock genes, namely those of ARNTL, ARNTL2, PER1 and PER2, have association with alcohol consumption, with alcohol abuse, or with alcohol dependence. It is of interest that variants of a fifth clock gene of key importance, that is those of CLOCK, are associated with alcohol-use disorders only if comorbid with depressive disorders. • Medical Research: What should clinicians and patients take away from your report? • Response: It is too early to give any implication for clinical practice on the basis of these findings. So, the current take-home message to clinicians or patients is that research on the topic has advanced, but clinical applications still remain to be seen. • Medical Research: What recommendations do you have for future research as a result of this study? • Response: Characterization of the functional relevance of these genetic variants and the mechanistic basis of these associations need to be elucidated in order to have the first insight of the potential, if any, small-molecule medication options for treatment of alcohol-use disorders. • Citation: • Clock Genes in Human Alcohol Abuse and Comorbid Conditions • Timo Partonen Alcohol Available online 16 January 2015 Read the rest of the interviews on MedicalResearch.com Content NOT an endorsement of efficacy and NOT intended as specific medical advice.
  • 56. Punitive Approach To Drug Dependence May Be Counterproductive MedicalResearch.com Interview with: Katharine A. Neill PhD Alfred C. Glassell III Postdoctoral Fellow in Drug Policy Baker Institute, Rice University Houston, TX 77005 • Medical Research: What is the background for this study? What are the main findings? Dr. Neill: This study is a historical review of drug policy in 20th century United States. It examines drug policy decisions and consequences through a socio-political lens and argues that the prominence of the law-and-order approach to dealing with drug offenders–that emphasizes punishment and incarceration over prevention and treatment–is a result of the construction of drug offenders as social deviants that threaten society. This punitive model has been especially harmful because it has occurred to the detriment of harm reduction approaches to drug use that have greater potential to negate the negative individual and public health consequences of drug use. • Medical Research: What should clinicians and patients take away from your report? • Dr. Neill: For clinicians who see drug dependence as a disease, the negative constructions of drug users is a big reason why harm reduction efforts–which may include education and prevention campaigns, expanded treatment services for individuals seeking help, prevention of dangerous drug use such as driving under the influence, and safe addiction maintenance for individuals who do not or cannot stop using via needle-exchange programs–have not received the same kind of funding as law enforcement approaches. Not only do powerful public safety interests have an interest in maintaining a punishment-over-treatment focus, but the general public’s perceptions of drug users as deviant and dangerous makes the funding of harm reduction efforts politically unpopular, especially programs such as needle-exchange. I suspect that many patients who have sought help for their drug dependence have had direct experiences with this kind of stigma. Read the rest of the interviews on MedicalResearch.com Content NOT an endorsement of efficacy and NOT intended as specific medical advice.
  • 57. Punitive Approach To Drug Dependence May Be Counterproductive MedicalResearch.com Interview with: Katharine A. Neill PhD Alfred C. Glassell III Postdoctoral Fellow in Drug Policy Baker Institute, Rice University Houston, TX 77005 • Medical Research: What recommendations do you have for future research as a result of this study? • Dr. Neill: Several things have to happen to refocus attention on drug dependence as a disease that requires medical treatment rather than a deviant behavior that deserves punishment. Changing public perceptions of drug offenders is critical. This is happening in small ways already. Drug war fatigue and a new crop of prescription opiate and heroin users that is whiter and more female than previous users has facilitated more calls for drug treatment and less focus on incarceration. But it is important that drug dependence is viewed as a disease regardless of who the users are. Research can help with this; studies that look at the effectiveness of drug treatment at preventing relapses, the public health benefits of needle- exchange programs, and the negative consequences of incarceration on drug users’ future drug use and recidivism rates, are all examples of important issues that can be explored more. It is also critical to have education campaigns that bring awareness to the public about the true nature of drug addiction. • Citation: • World Medical & Health Policy Vol 6 Issue 4 • Katharine A. Neill Article first published online: 21 DEC 2014 • DOI: 10.1002/wmh3.123 Read the rest of the interviews on MedicalResearch.com Content NOT an endorsement of efficacy and NOT intended as specific medical advice.